You are on page 1of 334

www.ck12.

org

MCR3U - Textbook (Final)

Brendon May
CK-12
David Gabel
Jen Kershaw
Raja Almukahhal
Larame Spence
Mara Landers
Nick Fiori
Art Fortgang
Melissa Vigil
Brenda Meery
Kaitlyn Spong
Lori Jordan
Kate Dirga
Andrew Gloag
Eve Rawley
Anne Gloag
Melissa Kramer
Mark Spong
Bradley Hughes
Larry Ottman
Andrea Hayes
Bill Zahner

ii
Say Thanks to the Authors
Click http://www.ck12.org/saythanks
(No sign in required)
www.ck12.org

EDITOR
Kaitlyn Spong
To access a customizable version of this book, as well as other
interactive content, visit www.ck12.org
AUTHORS
Brendon May
CK-12
David Gabel
Jen Kershaw
CK-12 Foundation is a non-profit organization with a mission to
Raja Almukahhal
reduce the cost of textbook materials for the K-12 market both in
Larame Spence
the U.S. and worldwide. Using an open-source, collaborative, and
Mara Landers
web-based compilation model, CK-12 pioneers and promotes the
Nick Fiori
creation and distribution of high-quality, adaptive online textbooks
Art Fortgang
that can be mixed, modified and printed (i.e., the FlexBook®
Melissa Vigil
textbooks).
Brenda Meery
Kaitlyn Spong
Copyright © 2020 CK-12 Foundation, www.ck12.org
Lori Jordan
Kate Dirga
The names “CK-12” and “CK12” and associated logos and the
Andrew Gloag
terms “FlexBook®” and “FlexBook Platform®” (collectively
Eve Rawley
“CK-12 Marks”) are trademarks and service marks of CK-12
Anne Gloag
Foundation and are protected by federal, state, and international
Melissa Kramer
laws.
Mark Spong
Bradley Hughes
Any form of reproduction of this book in any format or medium,
Larry Ottman
in whole or in sections must include the referral attribution link
Andrea Hayes
http://www.ck12.org/saythanks (placed in a visible location) in
Bill Zahner
addition to the following terms.

Except as otherwise noted, all CK-12 Content (including CK-12


Curriculum Material) is made available to Users in accordance
with the Creative Commons Attribution-Non-Commercial 3.0
Unported (CC BY-NC 3.0) License (http://creativecommons.org/
licenses/by-nc/3.0/), as amended and updated by Creative Com-
mons from time to time (the “CC License”), which is incorporated
herein by this reference.

Complete terms can be found at http://www.ck12.org/about/


terms-of-use.

Printed: February 18, 2020

iv
www.ck12.org Contents

Contents

1 Introduction to Functions 1
1.1 Functions and Relations . . . . . . . . . . . . . . . . . . . . . . . . . . . . . . . . . . . . . . . 2
1.2 Domain and Range of a Function . . . . . . . . . . . . . . . . . . . . . . . . . . . . . . . . . . 7
1.3 Function Families . . . . . . . . . . . . . . . . . . . . . . . . . . . . . . . . . . . . . . . . . . 10
1.4 Function Notation . . . . . . . . . . . . . . . . . . . . . . . . . . . . . . . . . . . . . . . . . . 16
1.5 Inverse Functions . . . . . . . . . . . . . . . . . . . . . . . . . . . . . . . . . . . . . . . . . . 23
1.6 Vertical and Horizontal Translations . . . . . . . . . . . . . . . . . . . . . . . . . . . . . . . . 29
1.7 Stretching and Reflecting Transformations . . . . . . . . . . . . . . . . . . . . . . . . . . . . . 34
1.8 Combining Transformations . . . . . . . . . . . . . . . . . . . . . . . . . . . . . . . . . . . . . 39
1.9 References . . . . . . . . . . . . . . . . . . . . . . . . . . . . . . . . . . . . . . . . . . . . . . 45

2 Algebraic Expressions 47
2.1 Addition and Subtraction of Polynomials . . . . . . . . . . . . . . . . . . . . . . . . . . . . . . 48
2.2 Multiplying Polynomials . . . . . . . . . . . . . . . . . . . . . . . . . . . . . . . . . . . . . . 53
2.3 Factoring Polynomials . . . . . . . . . . . . . . . . . . . . . . . . . . . . . . . . . . . . . . . . 57
2.4 Simplifying Rational Expressions . . . . . . . . . . . . . . . . . . . . . . . . . . . . . . . . . . 92
2.5 Multiplying and Dividing Rational Expressions . . . . . . . . . . . . . . . . . . . . . . . . . . 94
2.6 Adding and Subtracting Rational Expressions . . . . . . . . . . . . . . . . . . . . . . . . . . . 107
2.7 References . . . . . . . . . . . . . . . . . . . . . . . . . . . . . . . . . . . . . . . . . . . . . . 120

3 Quadratic Functions 122


3.1 Properties of Quadratic Functions . . . . . . . . . . . . . . . . . . . . . . . . . . . . . . . . . . 123
3.2 Vertex Form of a Quadratic Equation . . . . . . . . . . . . . . . . . . . . . . . . . . . . . . . . 130
3.3 Inverse of Quadratic Functions . . . . . . . . . . . . . . . . . . . . . . . . . . . . . . . . . . . 133
3.4 Simplifying Square Roots . . . . . . . . . . . . . . . . . . . . . . . . . . . . . . . . . . . . . . 138
3.5 Methods for Solving Quadratic Functions . . . . . . . . . . . . . . . . . . . . . . . . . . . . . 141
3.6 Families of Quadratic Functions . . . . . . . . . . . . . . . . . . . . . . . . . . . . . . . . . . 146
3.7 The Discriminant . . . . . . . . . . . . . . . . . . . . . . . . . . . . . . . . . . . . . . . . . . 151
3.8 Linear-Quadratic Systems . . . . . . . . . . . . . . . . . . . . . . . . . . . . . . . . . . . . . . 157
3.9 References . . . . . . . . . . . . . . . . . . . . . . . . . . . . . . . . . . . . . . . . . . . . . . 161

4 Exponential Functions 162


4.1 Product Rules for Exponents . . . . . . . . . . . . . . . . . . . . . . . . . . . . . . . . . . . . 163
4.2 Quotient Rules for Exponents . . . . . . . . . . . . . . . . . . . . . . . . . . . . . . . . . . . . 167
4.3 Power Rule for Exponents . . . . . . . . . . . . . . . . . . . . . . . . . . . . . . . . . . . . . . 170
4.4 Zero and Negative Exponents . . . . . . . . . . . . . . . . . . . . . . . . . . . . . . . . . . . . 173
4.5 Fractional Exponents . . . . . . . . . . . . . . . . . . . . . . . . . . . . . . . . . . . . . . . . 180
4.6 Exponential Expressions . . . . . . . . . . . . . . . . . . . . . . . . . . . . . . . . . . . . . . 187
4.7 Graphs of Exponential Functions . . . . . . . . . . . . . . . . . . . . . . . . . . . . . . . . . . 190
4.8 Graphing Transformed Exponential Functions . . . . . . . . . . . . . . . . . . . . . . . . . . . 199
4.9 Applications of Exponential Functions . . . . . . . . . . . . . . . . . . . . . . . . . . . . . . . 210
4.10 References . . . . . . . . . . . . . . . . . . . . . . . . . . . . . . . . . . . . . . . . . . . . . . 214

v
Contents www.ck12.org

5 Trigonometric Functions and Applications 215


5.1 Sine, Cosine, Tangent . . . . . . . . . . . . . . . . . . . . . . . . . . . . . . . . . . . . . . . . 216
5.2 Solving Right Triangles . . . . . . . . . . . . . . . . . . . . . . . . . . . . . . . . . . . . . . . 221
5.3 Special Right Triangles . . . . . . . . . . . . . . . . . . . . . . . . . . . . . . . . . . . . . . . 224
5.4 Reference Angles and Angles in the Unit Circle . . . . . . . . . . . . . . . . . . . . . . . . . . 229
5.5 Signs of Trigonometric Ratios on the Unit Circle . . . . . . . . . . . . . . . . . . . . . . . . . . 236
5.6 Trigonometric Ratios on the Unit Circle . . . . . . . . . . . . . . . . . . . . . . . . . . . . . . 241
5.7 Introduction to Trig Identities . . . . . . . . . . . . . . . . . . . . . . . . . . . . . . . . . . . . 246
5.8 Laws of Sines and Cosines . . . . . . . . . . . . . . . . . . . . . . . . . . . . . . . . . . . . . 250
5.9 Ambiguity and 3D Trigonometry . . . . . . . . . . . . . . . . . . . . . . . . . . . . . . . . . . 255
5.10 Sinusoidal Functions . . . . . . . . . . . . . . . . . . . . . . . . . . . . . . . . . . . . . . . . 265
5.11 Transforming Trigonometric Functions . . . . . . . . . . . . . . . . . . . . . . . . . . . . . . . 276
5.12 Modelling Sinusoidal Functions . . . . . . . . . . . . . . . . . . . . . . . . . . . . . . . . . . . 283
5.13 Trigonometric Applications . . . . . . . . . . . . . . . . . . . . . . . . . . . . . . . . . . . . . 288
5.14 References . . . . . . . . . . . . . . . . . . . . . . . . . . . . . . . . . . . . . . . . . . . . . . 292

6 Discrete Functions 294


6.1 Arithmetic Sequences and Series . . . . . . . . . . . . . . . . . . . . . . . . . . . . . . . . . . 295
6.2 Geometric Sequences and Series . . . . . . . . . . . . . . . . . . . . . . . . . . . . . . . . . . 304
6.3 Simple and Compound Interest . . . . . . . . . . . . . . . . . . . . . . . . . . . . . . . . . . . 312
6.4 Annuities (Future Value) . . . . . . . . . . . . . . . . . . . . . . . . . . . . . . . . . . . . . . 319
6.5 Annuities (Present Value) . . . . . . . . . . . . . . . . . . . . . . . . . . . . . . . . . . . . . . 323
6.6 References . . . . . . . . . . . . . . . . . . . . . . . . . . . . . . . . . . . . . . . . . . . . . . 327

vi
www.ck12.org Chapter 1. Introduction to Functions

C HAPTER
1 Introduction to Functions
Chapter Outline
1.1 F UNCTIONS AND R ELATIONS
1.2 D OMAIN AND R ANGE OF A F UNCTION
1.3 F UNCTION FAMILIES
1.4 F UNCTION N OTATION
1.5 I NVERSE F UNCTIONS
1.6 V ERTICAL AND H ORIZONTAL T RANSLATIONS
1.7 S TRETCHING AND R EFLECTING T RANSFORMATIONS
1.8 C OMBINING T RANSFORMATIONS
1.9 R EFERENCES

1
1.1. Functions and Relations www.ck12.org

1.1 Functions and Relations

Suppose you wanted to get cash for your $250 stash of nickels, dimes and quarters that you had accumulated over
many years. To see where you could get the best deal, you went online to check out what banks would give you for
your coins, and you got the following redemption information:
Bank A: full cash value for customers with an account; otherwise 91.1% of your coins’ value.
Bank B: full cash value for customers with an account if the value is put on a merchant’s gift card that has use fees;
otherwise 92% of your coin’s value.
Bank C: full cash value for customers with an account if the value is $100 or less; otherwise 99% of the coins’ value.
When you told your friends about the redemption information, one of them said: “Oh, really! The money you’ll get
for your coins is not a function of their actual value.” Was your friend correct? Why or why not?

Functions and the Vertical Line Test

Consider two situations shown in the boxes below:

TABLE 1.1:
Situation 1: Situation 2:
You are selling raffle tickets for a school fundraiser. You collect data from several students in your class on
Each ticket costs $3.00 their ages and their heights: (18, 6500 ), (17, 6400 ), (18,
6700 ), (18, 6800 ), (17,6600 )

In the first situation, let the variable x represent the number of raffle tickets that you sell, and let y represent the
amount of money you make. If you sell x raffle tickets, you will make y = 3x dollars; there is one and only one
number representing your profit. Notice that you can use the number of raffle tickets you sell to predict how much
money you will make. This is an example of a function.
Now consider the second situation. Can you similarly use the data to predict specific height, based on age? No, this
is not the case in the second situation. For example, if a student is 18 years old, there are multiple heights that the
student could be. This situation is not a function.
A function is a relationship between an independent variable, the input, and a dependent variable, the output, where
each input value of the independent variable, corresponds to one and only one output value of the dependent
variable.
It is important to note that both situations above are relations. A relation is simply a relationship between two sets of
numbers or data. For example, in the second situation, we created a relationship between students’ ages and heights,
just by writing each student’s information as an ordered pair.
Functions may be presented in many ways. Some of the most common ways to represent functions include: sets of
ordered pairs (e.g., in a table), written or equation rules, and graphs.
The notation used to show that there is a functional relationship between the independent and dependent variables
is called functional notation. The typical notation for a function is f (x), which is another way of representing the
dependent variable y in an equation. The function y = 3x would be represented using function notation as follows:

f (x) = 3x

2
www.ck12.org Chapter 1. Introduction to Functions

The table below shows the different ways to represent a relation:

TABLE 1.2:
Representation Example
Set of ordered pairs (1,3), (2,6), (3,9), (4,12) (a subset of the ordered pairs
for this function)
Equation

y = 3x or f (x) = 3x
Graph

In the first representation above, we are given a set of ordered pairs. To verify that this is a function, we must ensure
that each x-value is associated with a single y-value. In this example, the first number in each pair (the x-value) is
different, so we can be certain that there are no cases where a particular x is associated with more than one y.
In the second representation, the equation of a line, it is apparent that any number put in place of x will result in a
different y, since the x number is simply being multiplied by 3.
The third representation above is a graph. A quick and effective visual to decide if a graph is a function is by doing
a “vertical line test”. If all possible vertical lines only cross the relation in one place, then the relation is a function.
If a vertical line can be drawn anywhere on the graph such that the line crosses the relation in two places, then the
relation is not a function.

Vertical Line Test

A graphed relation is a function if there are no vertical lines that intersect the graphed relation in more than one
point.
Are each of these graphed relations a function?

3
1.1. Functions and Relations www.ck12.org

1.

2.

For graph 1, by drawing a vertical line (the red line) through the graph, we can see that the vertical line intersects
the circle more than once. Therefore, this graph is NOT a function.
For graph 2, No matter where a vertical line is drawn through the graph, there will be only one intersection.
Therefore, this graph is a function.

MEDIA
Click image to the left or use the URL below.
URL: http://www.ck12.org/flx/render/embeddedobject/79151

4
www.ck12.org Chapter 1. Introduction to Functions

Examples

Example 1

Earlier, you were asked about getting cash for a $250 stash of coins and if the data you received from the banks
represent a function. Your friend thinks not but you are not sure.
If we were to organize the information received from the banks into ordered pairs (x, y), it might look something
like:
Bank A: ($250, $250), ($250, $250*0.911)
Bank B: ($250, $250-fees), ($250, $250*0.92)
Bank C: ($250, $250*0.99)
Each x value, the independent variable, represents the real value of coins, in this case $250, and each y value, the
dependent variable, represents the money the bank will give you for the coins.
Since there are many different y values for the one x value, the above relationships definitely are not a function. Your
friend was right.

Example 2

Determine if each relation is a function:

1. (2, 4), (3, 9), (5, 11), (5, 12)


2. Function defined as:

(2, 4), (3, 9), (5, 11), (5, 12) is not a function because 5 is paired with 11 and with 12.
The graph displaying a relation is a function because every x is paired with only one y. A vertical line through the
graph will always only encounter a single point.

For the following examples, determine if the relation is a function.

Example 3

(2, 0)(4, −1)(2.1, 4)(1, 4)(4, −1)


Don’t be fooled! This is a function, there is only one unique output for each input. The fact that both x values 2.1
and 1 are associated with y value 4 does not mean that 2.1 and 1 don’t have a specific associated value. Also, not
matter how close two x’s (2 and 2.1, for instance) may be, if they are not exactly the same, they don’t affect the
definition of a function.

5
1.1. Functions and Relations www.ck12.org

Example 4

y = 4x
Any value chosen for x has one and only one associated value for y (4 times as big) and therefore is a function.

Example 5

x = |y|
This is not a function. This graph looks like a “<”, with the point on the origin. Any value chosen for x will have 2
associated y values. For instance: 4 = |−4| and 4 = |4|.

6
www.ck12.org Chapter 1. Introduction to Functions

1.2 Domain and Range of a Function

Learning Objectives

In this concept, you will learn to identify the domain and range of a simple linear function.

Joanna and Macy are helping with a research project about how fish waste is affecting the local environment. They
know that fish length is related to the amount of waste produced by each fish. They also know that fish in the fish
farm can be anywhere from 2 inches to 30 inches. The amount of waste (in mL) per day is approximately equal to
.62 times the fish length.
Joanna and Macy are making a chart that captures what they know. Each fish length corresponds to a volume of
waste. So far they recorded fish lengths of 2.8, 11.5, 10.7, 19.3, and 29. The corresponding amounts of waste are
1.736, 7.13, 6.634, 11.966, and 17.98.
Can you calculate the total daily amount of waste in the fish farm by creating a function and stating the domain range
of that function?
In this concept, you will learn to identify the domain and range of a simple linear function.

Identifying the Domain and Range of a Function

A function is a rule that maps elements of one set, the input, to elements of another set, the output. In a function,
an element of the input can be mapped to only one element in the output set. This means you cannot have one input
being assigned to two different outputs.
One way to represent a function is as a set of ordered pairs. The first element in the ordered pair is an element of the
input and it is assigned to the second element of the ordered pair, the output.

7
1.2. Domain and Range of a Function www.ck12.org

While the rule that does this assigning might not be apparent, you can still represent the function as a set of ordered
pairs.
Let’s look at an example of a function being described by a set of ordered pairs. Notice that braces, {}, are used to
surround the set of ordered pairs.

{(0, 5), (1, 6), (2, 7), (3, 8)}

In (1, 6), 1 is an element in the input and it is being assigned to an element, 6, in the output. Each element, in the
input (0, 1, 2, 3) is being assigned to only one element in the output (5, 6, 7, 8), so this list of ordered pairs does
represent a function.
Let’s take a look at another set of ordered pairs and determine if it represents a function or not and list the elements
of the input and elements of the output.

{(2, 4), (5, 3), (6, 7), (2, 8)}

The elements of the input are (2, 5, 6) and the elements of the output are (4, 3, 7, 8). However this set of ordered pairs
does not represent a function. It violates one of the conditions of a function. The element 2, from the input is being
assigned to two different outputs, 4 and 8. This doesn’t work for functions.
More formally, the domain of a function is the set of all the elements in the input and the range is the set of all the
elements in the output.
Let’s look at another example.
Identify the domain and range of this function (or a rule that assigns an element of the input to an element of the
output) represented as the following set of ordered pairs.

{(0, −10), (2, −8), (4, −6), (6, −4)}

The domain is the set of all the elements in the input, or all of the first elements of the ordered pairs.
The answer is the domain of this function is {0, 2, 4, 6}.
The range is the set of all the elements in the output, or all of the second elements in the ordered pair.
The answer is the range of this function is {−10, −8, −6, −4}.

Examples

Example 1

Earlier, you were given a problem about Joanna and Macy, who are researching how waste from local fish farms is
impacting the environment.
They have recorded fish lengths of 2.8, 11.5, 10.7, 19.3, and 29. The corresponding amounts of waste are 1.736,
7.13, 6.634, 11.966, and 17.98.
This information can be represented as a set or ordered pairs.

{(2.8, 1.736), (11.5, 7.13), (10.7, 6.634), (19.3, 11.966), (29, 17.98)}

8
www.ck12.org Chapter 1. Introduction to Functions

The five entries Joanna and Macy recorded represent a function with the input being the fish length and the output
being amount of waste.
The domain of this function is the fish lengths, or {2.8, 11.5, 10.7, 19.3, 29}, and the range is the waste amount in
mL, or {1.736, 7.13, 6.634, 11.966, 17.98}.

Example 2

Write the domain and range of this function, represented as a set of ordered pairs.

{(1, 3)(3, 9)(4, 6)(5, 12)}

The first value of each ordered pair is the domain of the function.
The second value of each ordered pair is the range of the function.
The answer is the domain is {1, 3, 4, 5} and the range is {3, 9, 6, 12}.
Identify the domain and range of each function, represented as a set of ordered pairs.

Example 3

{(1, 3)(2, 4)(5, 7)(9, 11)}

The first value of each ordered pair is the domain of the function.
The second value of each ordered pair is the range of the function.
The domain is {1, 2, 5, 9} and the range is {3, 4, 7, 11}.

Example 4

{(8, 12)(9, 22)(4, 7)(2, 5)}

The first value of each ordered pair is the domain of the function.
The second value of each ordered pair is the range of the function.
The domain is {8, 9, 4, 2}, and the range is {12, 22, 7, 5}.

Example 5

{(8, 9)(3, 5)(7, 6)(10, 12)}

The first value of each ordered pair is the domain of the function.
The second value of each ordered pair is the range of the function.
The domain is {8, 3, 7, 10}, and the range is {9, 5, 6, 12}.

9
1.3. Function Families www.ck12.org

1.3 Function Families

Learning Objectives

Here you will learn about evaluating more complex functions involving powers and roots by identifying the ’family’
each function belongs to in order to simplify the general form of the function’s graph.
Often, the most challenging part of completing a math problem is just getting started. Once you have a ’feel’ for
how a particular problem should be solved, crunching the numbers is generally not very difficult.
An understanding of function families can be a big help with this, as it gives you an idea of what a more complex
function should look like once it has been graphed, just by identifying the parent or most simplified version of the
function.
Do you know the parent functions for the Square, Cube, Square Root, and Reciprocal function families?

Square and Cube Function Families

Function Family: Square Functions

A square function is a 2nd degree equation, meaning it has an x2 .


The graph of every square function is a parabola. A parabola has a vertex, and an axis of symmetry. The graph
below shows these aspects of the graph of y = x2 - 3.

Function Family: Cubic Functions

A cube function is a third-degree equation: x3 and which does not contain negative or fractional exponents. In
general, the graphs of cube functions have a particular shape, illustrated by the graph shown here:

10
www.ck12.org Chapter 1. Introduction to Functions

Cubic functions have a similar shape. However, only some cubic functions will have a relative maximum and
minimum. For example, the graph of y = (x - 2)3 - 5x shown above, has a relative maximum around x = 0.7, and a
relative minimum around x = 3.3. The shape of the cubic graph means that we can predict end behavior: one end
will approach ∞, and the other will approach −∞.
It is important to note here that the cubic function grows faster than an associated quadratic function. For example,
y = x3 grows faster than y = x2 .

Function Family: Square Root Functions



Consider the parent of the family, y = x. The domain of the function is limited to real numbers ≥ 0, as the square
root of a negative number is not a real number. Similarly, the range of the function is limited to real numbers ≥
0. This may seem√confusing if you think of squares having two roots. For example, 9 has two roots: 3, and -3.
However, for y = x, we have to define the function value as the principal root, which means the positive root.

The function y = x is shown below:

The same kind of limitations of domain and range will exist for any square root function.

Function Family: Reciprocal Functions

The function y = 1x has a rather surprising graph. First, the domain cannot include 0, as the fraction 10 is undefined.
The range also does not include 0, as a fraction can only be zero if the numerator is zero, and the numerator of y = 1x

11
1.3. Function Families www.ck12.org

is always 1.
In order to understand what these limitations mean for the graph, we will consider function values near x = 0 and y =
0. First, consider very small values of x. For example, consider x = 0.001. This yields y = 1x = 0.001
1
= 1000. As we
get closer and closer to x = 0, the function values approach ∞. On the other side of the x-axis the function values will
approach −∞. We can see this behavior in the graph as a vertical asymptote: the graph is asymptotic to the y-axis.
We can also see in the graph that as x approaches +∞ or −∞, the function values approach 0. The exclusion of y =
0 from the range means that the function is asymptotic to the x-axis.

MEDIA
Click image to the left or use the URL below.
URL: http://www.ck12.org/flx/render/embeddedobject/187142

MEDIA
Click image to the left or use the URL below.
URL: http://www.ck12.org/flx/render/embeddedobject/187146

Examples

Example 1

Earlier, you were given a question about identifying the parent functions of various function families.

12
www.ck12.org Chapter 1. Introduction to Functions

Can you identify the parent functions of the square, cube, square root, and reciprocal functions now?
Square: y = x2
Cube: y = x3

Square Root: y = x
Reciprocal: y = 1/x
Learning the function families is one of the fastest way to graph complex equations. Using parent functions and
transformations (which are detailed in another set of lessons), you can graph very complex equations rather easily.

Example 2


Graph the function y = 3 − x + 1, identify the parent function, and state the domain and range of the function.

The parent function is y = x

From the graph you can see that the function does not take on any x values above 3. (Why not?) Therefore the
domain is limited to real numbers ≤ 3. The function’s lowest value is 1, so the range is limited to all real numbers
≥1.
It is important to note that while the graph of a square root function might look as if it has horizontal asymptote, it
does not. The function values will grow without bound (though relatively slowly!).

Example 3

2
Graph the function f (x) = x−3 , identify the parent function, and identify horizontal and vertical asymptotes.
The parent function is y = 1/x

13
1.3. Function Families www.ck12.org

The graph is asymptotic to the x-axis (y = 0) and to the line x = 3.

Example 4

Identify the parent function within the following set of functions. Graph the set of functions using a graphing
calculator. Identify similarities and differences of the set.

TABLE 1.3:
f (x) = x2 − 10 f (x) = x2 − 1 f (x) = x2
f (x) = x2 + 3 f (x) = x2 + 9

14
www.ck12.org Chapter 1. Introduction to Functions

a. Parent Function: The parent function of this group of quadratic functions is the most basic function in the set:
f (x) = x2

b. Similarities: width, shape, end behavior, and degree

c. Differences: x and y intercepts

Example 5

Identify the parent function within the following set of functions. Graph the set of functions using a graphing
calculator. Identify similarities and differences of the set.

TABLE 1.4:
f (x) = (x + 9)3 f (x) = (x + 2)3 f (x) = x3
f (x) = (x − 4)3 f (x) = (x − 8)3

a. Parent Function: The parent function of this group of quartic functions is the most basic function in the set:
y = x3

b. Similarities: end behavior, domain and range, direction, and width

c. Differences: x and y intercepts, increasing and decreasing intervals

15
1.4. Function Notation www.ck12.org

1.4 Function Notation

Learning Objectives

Here you’ll learn how to use function notation when working with functions.
Suppose the value V of a digital camera t years after it was bought is represented by the function V (t) = 875 − 50t.

• Can you determine the value of V (4) and explain what the solution means in the context of this problem?
• Can you determine the value of t when V (t) = 525 and explain what this situation represents?
• What was the original cost of the digital camera?

Function Notation

A function is like a machine that shows how an input is changed to create an output. For example, a function
that triples the input and subtracts one from it to create the output, would convert x into 3(x) − 1. If that function
were named f , and 3 is fed into the machine, 3(3) − 1 = 8 comes out. In other words, function f of 3 = 3(3) − 1, or,
in standard function notation, f (3) = 3(3) − 1 = 8.

16
www.ck12.org Chapter 1. Introduction to Functions

When naming a function the symbol f (x) is often used. The symbol f (x) is pronounced as “ f of x.” This means that
the equation is a function that is written in terms of the variable x. An example of such a function is f (x) = 3x + 4.
Functions can also be written using a letter other than f and a variable other than x. For example, v(t) = 2t 2 − 5 and
d(h) = 4h − 3. In addition to representing a function as an equation, you can also represent a function:

• As a graph
• As ordered pairs
• As a table of values
• As an arrow or mapping diagram

When a function is represented as an equation, an ordered pair can be determined by evaluating various values of
the assigned variable. Suppose f (x) = 3x − 4. To calculate f (4), substitute:

f (4) = 3(4) − 4
f (4) = 12 − 4
f (4) = 8

Graphically, if f (4) = 8, this means that the point (4, 8) is a point on the graph of the line.

Let’s practice function notation by substituting the following values of x into the equation f (x) =
x2 2x5 :
1. f (2)
2. f (−7)
3. f (1.4)

To determine the value of the function for the assigned values of the variable, substitute the values into the
function.

17
1.4. Function Notation www.ck12.org

f (x) = x2 + 2x + 5 f (x) = x2 + 2x + 5 f (x) = x2 + 2x + 5


↓ ↓ & ↓ ↓ & ↓ ↓ &
2 2 2
f (2) = (2) + 2(2) + 5 f (−7) = (−7) + 2(−7) + 5 f (1.4) = (1.4) + 2(1.4) + 5
f (2) = 4 + 4 + 5 f (−7) = 49 − 14 + 5 f (1.4) = 1.96 + 2.8 + 5
f (2) = 13 f (−7) = 40 f (1.4) = 9.76

Functions can also be represented as mapping rules.

Now, if g : x → 5 − 2x , let’s find the following in simplest form:

1. g(y)

g(y) = 5 − 2y

2. g(y − 3)

g(y − 3) = 5 − 2(y − 3) = 5 − 2y + 6 = 11 − 2y

3. g(2y)

g(2y) = 5 − 2(2y) = 5 − 4y

Examples

Example 1

Earlier, you were told that the value V of a digital camera t years after it was bought is represented by the function
V (t) = 875 − 50t. We want to:

• Determine the value of V (4) and explain what the solution means to this problem.
• Determine the value of t when V (t) = 525 and explain what this situation represents.
• What was the original cost of the digital camera?

The camera is valued at $675, 4 years after it was purchased.

V (t) = 875 − 50t


V (4) = 875 − 50(4)
V (4) = 875 − 200
V (4) = $675

The digital camera has a value of $525, 7 years after it was purchased.

18
www.ck12.org Chapter 1. Introduction to Functions

V (t) = 875 − 50t Let V (t) = 525


525 = 875 − 50t Solve the equation
525 − 875 = 875 − 875 − 50t
− 350 = −50t
−350 −50t
=
−50 −50
7=t

The original cost of the camera was $875.

V (t) = 875 − 50t Let t = 0.


V (0) = 875 − 50(0)
V (0) = 875 − 0
V (0) = $875

Example 2

If f (x) = 3x2 − 4x + 6 find:

• f (−3)
• f (a − 2)

f (x) = 3x2 − 4x + 6 Substitute (−3) for x in the function.


f (−3) = 3(−3)2 − 4(−3) + 6 Perform the indicated operations.
f (−3) = 3(9) + 12 + 6 Simplify
f (−3) = 27 + 12 + 6
f (−3) = 45
f (−3) = 45

f (x) = 3x2 − 4x + 6
f (a − 2) = 3(a − 2)2 − 4(a − 2) + 6 Write (a − 2)2 in expanded form.
f (a − 2) = 3(a − 2)(a − 2) − 4(a − 2) + 6 Perform the indicated operations.
f (a − 2) = (3a − 6)(a − 2) − 4(a − 2) + 6
f (a − 2) = 3a2 − 6a − 6a + 12 − 4a + 8 + 6 Simplify
2
f (a − 2) = 3a − 16a + 26
f (a − 2) = 3a2 − 16a + 26

19
1.4. Function Notation www.ck12.org

Example 3

m+3 12
If f (m) = 2m−5 find ‘m’ if f (m) = 13

m+3
f (m) =
2m − 5
12 m+3
= Solve the equation for m.
13 2m − 5
12 m+3
(13)(2m − 5) = (13)(2m − 5)
13 2m − 5
12 m+3
(13)(2m

 − 5) = (13) (2m
− 5)


13
 2m−

5
(2m − 5)12 = (13)m + 3
24m − 60 = 13m + 39
24m − 60+60 = 13m + 39+60
24m = 13m + 99
24m−13m = 13m−13m + 99
11m = 99
11m 99
=
11 11
9
11m

 99


=
11

 11


m=9

Example 4

The emergency brake cable in a truck parked on a steep hill breaks and the truck rolls down the hill. The distance in
feet, d, that the truck rolls is represented by the function d = f (t) = 0.5t 2 .
How far will the truck roll after 9 seconds? How long will it take the truck to hit a tree which is at the bottom of the
hill 600 feet away? Round your answer to the nearest second.
d = f (t) = 0.52

d = f (t) = 0.52 Substitute 9 for t.


2
f (9) = 0.5(9) Perform the indicated operations.
f (9) = 0.5(81)
f (9) = 40.5 f eet

20
www.ck12.org Chapter 1. Introduction to Functions

After 9 seconds, the truck will roll 40.5 feet.

d = f (t) = 0.5t 2 Substitute 600 for d.


2
600 = 0.5t Solve for t.
600 0.5t 2
=
0.5 0.5
1200
600
  2
0.5t
=
0.5
 0.5

1200 = t 2
√ √
1200 = t 2
34.64 seconds ≈ t

The truck will hit the tree in approximately 35 seconds.

Example 5

2a−3
Let P(a) = a+2 .
First, evaluate:

• P(0)
• P(1) 
• P − 12

Then find a value of ‘a’ where P(a) does not exist, find P(a − 2) in simplest form, and find ‘a’ if P(a) = −5
Evaluating for 0, 1, and − 21 :

2a − 3 2a − 3 2a − 3
P(a) = P(a) = P(a) =
a+2 a+2 a+2
2 − 12 − 3
  
2(0) − 3 2(1) − 3 1
P(0) = P(1) = P − =
− 12 + 2

(0) + 2 (1) + 2 2
 
  1 2 − 1 − 3
−3 2−3 1 2
P(0) = P(1) = P − =
2 1+2 2 − 12 + 42
 
−1 1 −1 − 3
P(1) = P − = 3
3 2 2
 
1 3
P − = −4 ÷
2 2
   
1 2
P − = −4
2 3
 
1 −8
P − =
2 3

Finding where the function is undefined:


The function will not exist if the denominator equals zero because division by zero is undefined.

21
1.4. Function Notation www.ck12.org

a+2 = 0
a+2−2 = 0−2
a = −2

2a−3
Therefore, if a = −2, then P(a) = a+2 does not exist.
Finding P(a − 2):

2a − 3
P(a) =
a+2
2(a − 2) − 3
P(a − 2) = Substitue a − 2 for a
(a − 2) + 2
2a − 4 − 3
P(a − 2) = Remove parentheses
a−2+2
2a − 7
P(a − 2) = Combine like terms
a
2a 7
P(a − 2) = − Express the fraction as two separate fractions and reduce.
a a
7
P(a − 2) = 2 −
a

Finding ‘a’ if P(a) = −5:

2a − 3
P(a) =
a+2
2a − 3
−5 = Let P(a) = −5
a + 2 
2a − 3
− 5(a + 2) = (a + 2) Multiply both sides by (a + 2)
a+2
 
2a − 3
− 5a − 10 = (
a+2) Simplify
a+2
− 5a − 10 = 2a − 3 Solve the linear equation
− 5a − 10 − 2a = 2a − 2a − 3 Move 2a to the left by subtracting
− 7a − 10 = −3 Simplify
− 7a − 10 + 10 = −3 + 10 Move 10 to the right side by addition
− 7a = 7 Simplify
−7a 7
= Divide both sides by -7 to solve for a.
−7 −7
a = −1

22
www.ck12.org Chapter 1. Introduction to Functions

1.5 Inverse Functions

Learning Objectives

Here you’ll learn how to find the inverse of a relation and function.
2
A planet’s maximum distance from the sun (in astronomical units) is given by the formula d = p 3 , were p is the
period (in years) of the planet’s orbit around the sun. What is the inverse of this function?

Inverse Functions

By now, you are probably familiar with the term “inverse”. Multiplication and division are inverses of each other.
More examples are addition and subtraction and the square and square root. We are going to extend this idea to
functions. An inverse relation maps the output values to the input values to create another relation. In other words,
we switch the x and y values. The domain of the original relation becomes the range of the inverse relation and the
range of the original relation becomes the domain of the inverse relation.
Let’s find the inverse mapping of S = {(6, −1), (−2, −5), (−3, 4), (0, 3), (2, 2)}.
Here, we will find the inverse of this relation by mapping it over the line y = x. As was stated above in the definition,
the inverse relation switched the domain and range of the original function. So, the inverse of this relation, S, is S−1
(said “s inverse”) and will flip all the x and y values.
S−1 = {(−1, 6), (−5, −2), (4, −3), (3, 0), (2, 2)}
If we plot the two relations on the x − y plane, we have:

The blue points are all the points in S and the red points are all the points in S−1 . Notice that the points in S−1 are a
reflection of the points in S over the line, y = x. All inverses have this property.

23
1.5. Inverse Functions www.ck12.org

If we were to fold the graph on y = x, each inverse point S−1 should lie on the original point from S. The point (2, 2)
lies on this line, so it has no reflection. Any value on this line will remain the same.
Domain of S: x ∈ {6, −2, −3, 0, 2}
Range of S: y ∈ {−1, −5, 4, 3, 2}
Domain of S0 : x ∈ {−1, −5, 4, 3, 2}
Range of S0 : y ∈ {6, −2, −3, 0, 2}
By looking at the domains and ranges of S and S−1 , we see that they are both functions (no x-values repeat). When
the inverse of a function is also a function, we say that the original function is a one-to-one function. Each value
maps one unique value onto another unique value.
Now, let’s find the inverse of f (x) = 32 x − 1.
This is a linear function. Let’s solve by doing a little investigation. First, draw the line along with y = x on the same
set of axes.

Notice the points on the function (blue line). Map these points over y = x by switching their x and y values. You
could also fold the graph along y = x and trace the reflection.

24
www.ck12.org Chapter 1. Introduction to Functions

The red line in the graph to the right is the inverse of f (x) = 23 x − 1. Using slope triangles between (-1, 0) and (1,
3), we see that the slope is 32 . Use (-1, 0) to find the y-intercept.

3
f −1 (x) = x + b
2
3
0 = (−1) + b
2
3
=b
2

The equation of the inverse, read “ f inverse”, is f −1 (x) = 32 x + 32 .


You may have noticed that the slopes of f and f −1 are reciprocals of each other. This will always be the case for
linear functions. Also, the x-intercept of f becomes the y-intercept of f −1 and vice versa.
Alternate Method: There is also an algebraic approach to finding the inverse of any function. Let’s repeat this
example using algebra.
Step 1: Change f (x) to y.
y = 23 x − 1
Step 2: Switch the x and y. Change y to y−1 for the inverse.
x = 32 y−1 − 1
Step 3: Solve for y0 .

2
x = y−1 − 1
3  
3 3 2 −1
(x + 1) = · y
2 2 3
3 3
x + = y−1
2 2

25
1.5. Inverse Functions www.ck12.org

The algebraic method will work for any type of function.



Finally, let’s determine if g(x) = x − 2 and f (x) = x2 + 2 are inverses of each other.
There are two different ways to determine if two functions are inverses of each other. The first, is to find f −1 and
g−1 and see if f −1 = g and g−1 = f .

p
x= y−1 − 2 x = (y−1 )2 + 2
x2 = y−1 − 2 and x − 2 = (y−1 )2

x2 + 2 = y−1 = g−1 (x) ± x − 2 = y−1 = f −1 (x)

√ √
Notice the ± sign in front of the square root for f −1 . That means that g−1 is x − 2 and − x − 2.
Therefore, f −1 is not really a function because it fails the vertical line test. However, if you were to take each
part separately, individually, they are functions. You can also think about reflecting f (x) over y = x. It would be a
parabola on its side, which is not a function.

The inverse of g would then be only half of the parabola, see below. Despite the restrictions on the domains, f and
g are inverses of each other.

26
www.ck12.org Chapter 1. Introduction to Functions

Examples

Example 1

2
Earlier, you were asked to find the inverse of the function d = p 3 .
2
In the function d = p 3 , d is the equivalent of y and p is the equivalent of x. So rewrite the equation and follow the
step-by-step process illustrated above.
2
y = x3 ,
Switch the x and y. Change y to y−1 for the inverse.
2
x = (y−1 ) 3
Solve for y0 .

2
x = (y−1 ) 3
3 2 3
x 2 = (y−1 ) 3 · 2
3
x 2 = y−1

3
Now replace y and x with d and p. The inverse d is p 2 .

Example 2

Find the inverse of g(x) = − 34 x + 12 algebraically.

27
1.5. Inverse Functions www.ck12.org

3
y = − x + 12
4
3
x = − y−1 + 12
4
3
x − 12 = − y−1
4
4
− (x − 12) = y−1
3
4
g−1 (x) = − x + 16
3

Example 3

Find the inverse of f (x) = 2x3 + 5 algebraically. Is the inverse a function?

y = 2x3 + 5
x = 2(y−1 )3 + 5
x − 5 = 2(y−1 )3
x−5
= (y−1 )3
2 r
−1 3 x−5
f (x) =
2

Yes, f −1 is a function. Plot in your graphing calculator if you are unsure and see if it passes the vertical line test.

28
www.ck12.org Chapter 1. Introduction to Functions

1.6 Vertical and Horizontal Translations

Learning Objectives

Here you will learn about graphing more complex types of functions easily by applying horizontal and vertical shifts
to the graphs of parent functions. If you are not familiar with parent functions or function families, it would be a
good idea to review the lessons on those topics before proceeding.
Horizontal and vertical transformations are two of the many ways to convert the basic parent functions in a function
family into their more complex counterparts.
What vertical and/or horizontal shifts must be applied to the parent function of y = x2 in order to graph g(x) =
(x − 3)2 + 4?

Vertical and Horizontal Transformations

Have you ever tried to draw a picture of a rabbit, or cat, or dog? Unless you are talented, even the most common
animals can be a bit of a challenge to draw accurately (or even recognizably!). One trick that can help even the
most "artistically challenged" to create a clearly recognizable basic sketch is demonstrated in nearly all "learn to
draw" courses: start with basic shapes. By starting your sketch with simple circles, ellipses, rectangles, etc., the
basic outline of the more complex figure is easily arrived at, then details can be added as necessary, but the figure is
already recognizable for what it is.
The same trick works when graphing equations. By learning the basic shapes of different types of function graphs,
and then adjusting the graphs with different types of transformations, even complex graphs can be sketched rather
easily. This lesson will focus on two particular types of transformations: vertical shifts and horizontal shifts.
We can express the application of vertical shifts this way:
Formally: For any function f (x), the function g(x) = f (x) + c has a graph that is the same as f (x), shifted c units
vertically. If c is positive, the graph is shifted up. If c is negative, the graph is shifted down.
Informally: Adding a positive number after the x outside the parentheses shifts the graph up, adding a negative (or
subtracting) shifts the graph down.
We can express the application of horizontal shifts this way:
Formally: given a function f (x), and a constant a > 0, the function g(x) = f (x - a) represents a horizontal shift a units
to the right from f (x). The function h(x) = f (x + a) represents a horizontal shift a units to the left.
Informally: Adding a positive number after the x inside the parentheses shifts the graph left, adding a negative (or
subtracting) shifts the graph right.

MEDIA
Click image to the left or use the URL below.
URL: http://www.ck12.org/flx/render/embeddedobject/187147

29
1.6. Vertical and Horizontal Translations www.ck12.org

MEDIA
Click image to the left or use the URL below.
URL: http://www.ck12.org/flx/render/embeddedobject/187148

Examples

Example 1

Earlier, you were given a question about applying vertical and/or horizontal shifts to a parent function in order to
graph a different function in the same function family.
What transformations must be applied to y = x2 , in order to graph g(x) = (x − 3)2 + 4?
The graph of g(x) = (x − 3)2 + 4 is the graph of y = x2 shifted 3 units to the right, and 4 units up.

Example 2

What must be done to the graph of y = x2 to convert it into the graphs of y = x2 - 3, and y = x2 + 4?

At first glance, it may seem that the graphs have different widths. For example, it might look like y = x2 + 4,
the uppermost of the three parabolas, is thinner than the other two parabolas. However, this is not the case. The
parabolas are congruent.
If we shifted the graph of y = x2 up four units, we would have the exact same graph as y = x2 + 4. If we shifted y =
x2 down three units, we would have the graph of y = x2 - 3.

30
www.ck12.org Chapter 1. Introduction to Functions

Example 3

Identify the transformation(s) involved in converting the graph of f (x) = |x| into g(x) = |x - 3|.
From the examples of vertical shifts above, you might think that the graph of g(x) is the graph of f(x), shifted 3 units
to the left. However, this is not the case. The graph of g(x) is the graph of f (x), shifted 3 units to the right.
The direction of the shift makes sense if we look at specific function values.

TABLE 1.5:
x g(x) = abs(x - 3)
0 3
1 2
2 1
3 0
4 1
5 2
6 3

From the table we can see that the vertex of the graph is the point (3, 0). The function values on either side of x = 3
are symmetric, and greater than 0.

Example 4

What transformations must be applied to y = x2 , in order to graph g(x) = (x + 2)2 − 2?

31
1.6. Vertical and Horizontal Translations www.ck12.org

The graph of g(x) = (x + 2)2 − 2 is the graph of y = x2 shifted 2 units to the left, and 2 units down.

Example 5

Use the parent function f(x) = x2 to graph f(x) = x2 + 3.


The function f(x) = x2 is a parabola with the vertex at (0, 0).
Adding outside the parenthesis shifts the graph vertically.
Therefore, f(x) = x2 + 3 will be a parabola with the vertex 3 units up.

Example 6

Use the parent function f(x) = |x| to graph f(x) = |x - 4|.


The graph of the absolute value function family parent function f(x) = |x| is a large "V" with the vertex at the origin.
Adding or subtracting inside the parenthesis results in horizontal movement.
Recall that the horizontal shift is right for negative numbers, and left for positive numbers.
Therefore f(x) = |x - 4| is a large "V" with the vertex 4 units to the right of the origin.

32
www.ck12.org Chapter 1. Introduction to Functions

33
1.7. Stretching and Reflecting Transformations www.ck12.org

1.7 Stretching and Reflecting Transformations

Learning Objectives

Here you will explore how to take the parent function in a function family and either stretch it horizontally or
vertically, or reflect it to change its direction. If you are not already familiar with function families, it would be a
good idea to review them first, and then return here to apply those concepts.
Understanding how changes in the equation of a function result in stretching and/or reflecting the graph of the
function is a great way to take some of the mystery out of graphing more complicated equations. By recognizing the
family to which a more complex equation belongs, and then identifying what changes have been made to the parent
of that family, the graph of even quite detailed functions can be made much more understandable.
See if you can identify what parts of the equation: y = − 15 x2 represent either a stretch or a reflection of the parent
function y = x2 before the examples in this section.

Stretching and Reflecting Transformations

Stretching and Compressing Graphs

If we multiply a function by a coefficient, the graph of the function will be stretched or compressed.
Given a function f(x), we can formalize compressing and stretching the graph of f(x) as follows:

• A function g(x) represents a vertical stretch of f (x) if g(x) = cf (x) and c > 1.

• A function g(x) represents a vertical compression of f (x) if g(x) = cf (x) and 0 < c < 1.

• A function h(x) represents a horizontal compression of f (x) if h(x) = f (cx) and c > 1.

• A function h(x) represents a horizontal stretch of f (x) if h(x) = f (cx) and 0 < c < 1.

Notice that a vertical compression or a horizontal stretch occurs when the coefficient is a number between 0 and 1.

Reflecting Graphs Over the y-axis and x-axis

Consider the graphs of the functions y = x2 and y = -x2 , shown below.

34
www.ck12.org Chapter 1. Introduction to Functions

The graph of y = -x2 represents a reflection of y = x2 , over the x-axis. That is, every function value of y = -x2 is the
negative of a function value of y = x2 . In general, g(x) = -f (x) has a graph that is the graph of f (x), reflected over the
x-axis.

MEDIA
Click image to the left or use the URL below.
URL: http://www.ck12.org/flx/render/embeddedobject/187155

Examples

Example 1

Earlier, you were asked a question about identifying transformations.


The function y = − 15 x2 is the result of transforming y = x2 by reflecting it over the x axis, because of the negative
co-efficient on the x, and vertically compressing it (making it wider), because the co-efficient on the x is a fraction
between 0 and 1.

Example 2

Identify the graph of the function y = (3x)2 .


We have multiplied x by 3. This should affect the graph horizontally. However, if we simplify the equation, we get y
= 9x2 . Therefore the graph if this parabola will be taller/thinner than y = x2 . Multiplying x by a number greater than
1 creates a horizontal compression, which looks like a vertical stretch.

35
1.7. Stretching and Reflecting Transformations www.ck12.org

Example 3

Identify the transformation described by y = ((1/2)x)2 .


If we simplify this equation, we get y = (1/4) x2 . Therefore multiplying x by a number between 0 and 1 creates a
horizontal stretch, which looks like a vertical compression. That is, the parabola will be shorter/wider.

Example 4

Sketch a graph of y = x3 and y = -x3 on the same axes.

At first the two functions might look like two parabolas. If you graph by hand, or if you set your calculator to
sequential mode (and not simultaneous), you can see that the graph of y = -x3 is in fact a reflection of y = x3 over the
x-axis.
However, if you look at the graph, you can see that it is a reflection over the y-axis as well. This is the case because
in order to obtain a reflection over the y-axis, we negate x. In other words, h(x) = f (-x) is a reflection of f (x) over
the y-axis. For the function y = x3 , h(x) = (-x)3 = (-x) (-x) (-x) = -x3 . This is the same function as the one we have
already graphed.
It is important to note that this is a special case. The graph of y = x2 is also a special case. If we want to reflect y =
x2 over the y-axis, we will just get the same graph! This can be explained algebraically: y = (-x)2 = (-x) (-x) = x2 .

Example 5
√ √
Graph the functions y = x and y = −x.

The equation y = −x might look confusing because of the - x under the square root. It is important to keep in
mind that - x means the opposite of x. Therefore the domain of this function is restricted to values ≤ 0. For example,

36
www.ck12.org Chapter 1. Introduction to Functions
p √ √
if x = - 4, y = −(−4) = 4 = 2. It is this domain, which includes all real numbers not in the domain of y = x
plus zero, that gives us a graph that is a reflection over the y-axis.
In sum, a graph represents a reflection over the x-axis if the function has been negated (i.e. the y has been negated if
we think of y = f (x)). The graph represents a reflection over the y-axis if the variable x has been negated.

Example 6


Identify the function and sketch the graph of y = x reflected over both axes.

√ the graph of y = x over both axes, the function must be negated both outside and inside the root:
To reflect
y = − −x. The negation (negative) outside of the root has the effect of reflecting the graph vertically, and the
negation inside of the root reflects the graph horizontally. The image below shows three versions:

BLUE: y = x

GREEN: y = − x

RED: y = −x

37
1.7. Stretching and Reflecting Transformations www.ck12.org

MEDIA
Click image to the left or use the URL below.
URL: http://www.ck12.org/flx/render/embeddedobject/187158

38
www.ck12.org Chapter 1. Introduction to Functions

1.8 Combining Transformations

Learning Objectives

Here you will learn about combining different types of transformations applied to a single graph.
How do the different forms of transformations result in the differences between the basic parent functions we
have explored and some of the more complex graphs you may have seen? It has likely occurred to you that these
individual transformations are not enough to result in such significant differences. How do we then apply the
individual transformations so that the more complex graphs may be understood?

Combining Transformations

By combining shifts, reflections, and vertical and horizontal stretches and compressions, a simple parent function
graph can represent a much more advanced function.
Consider the equation y = 2(x - 3)2 + 1. We can compare the graph of this function to the graph of the parent y = x2 :
the graph represents a vertical stretch by a factor of 2, a horizontal shift 3 units to the right, and a vertical shift of 1
unit.
We can use this relationship to graph the function y = 2(x - 3)2 + 1. You can start by sketching y = x2 or y = 2x2 .
Then you can shift the graph 3 units to the right, and up 1 unit.

MEDIA
Click image to the left or use the URL below.
URL: http://www.ck12.org/flx/render/embeddedobject/187159

39
1.8. Combining Transformations www.ck12.org

Examples

Example 1

Earlier, you were asked how to apply individual transformations to a more complex graph.
Graph the function below using your knowledge of the parent function y = |x| and your knowledge of transforma-
tions.
f (x) = -|x| + 3
The parent graph of this function is the graph of y = |x|, reflected over the x-axis, and shifted up 3 units. The question
is: which transformation do you perform first?
We can answer this question if we consider a few key function values. The table below shows several function values
for f (x) = -|x| + 3:

TABLE 1.6:
x f (x) = −|x|+3
-3 −|−3|+3 = −(3) + 3 = −3 + 3 = 0
-2 −|−2|+3 = −(2) + 3 = −2 + 3 = 1
-1 2
0 3
1 2
3 0

From the function values in the table we can see that the function increases until a vertex at (0, 3), and then it
decreases again. This tells us that we can obtain the graph if we first reflect y = |x| over the x-axis (turn the “v” upside
down), and then shift the graph up 3 units.

We can also justify this ordering of the transformations of we think about the order of operations. To find any

40
www.ck12.org Chapter 1. Introduction to Functions

function value we take an x value, find its absolute value, find the negative of that number, and then add 3. This is
the same as the order of the transformation: reflection comes before shifting up.

Example 2

Describe the relationship between the graphs of f (x) = 3(x + 7)3 + 5 and g(x) = x3 .
The graph of f (x) = 4(x + 8)3 − 3 is the graph of g(x) = x3 , stretched vertically (made narrower) by a factor of 3,
shifted 7 units to the left, and then shifted 5 units up.

Example 3

Graph the function below using your knowledge of the parent function y = |x| and your knowledge of transforma-
tions.
g(x) = |-x + 3|
This function represents a horizontal shift of y = |x|, and a reflection over the x-axis. Before graphing, consider a few
function values:

TABLE 1.7:
x g(x) = |−x + 3|
|−(−3) + 3|= |3 + 3|= |6|= 6
|−(−2) + 3|= |2 + 3|= |5|= 5
4
0 3
1 2

From the values in the table, we can see that the vertex of the graph is at (3, 0). The graph is shown below.

41
1.8. Combining Transformations www.ck12.org

The graph looks the same as the graph of y = |x - 3|. This is the case because y = |-x + 3| = |-(x - 3)|, and because |- a|
= |a| for all values of a, then |-(x - 3)| = |x - 3|. So the original function is equal to |x - 3|.
We can still think of this graph as a reflection: if we reflect y = |x| over the x-axis, the graph remains the same, as
it is symmetric over the x-axis. Then we shift the graph 3 units to the right. What is important to note here is that
in order to “read” the equation as a horizontal shift, the entire expression inside the function (in this case, inside the
absolute value) must be negated.

Example 4

Sketch the graph of y = −1|x + 2|−3.


The key to multiple transformations is to do them in order. Another way to keep track of which operations to do in
which order is to just do them in the order they appear in the equation, left to right.

To sketch y = −1|x + 2|−3 start with the parent equation: f (x) = |x| and complete the transformations left to right:

First, reflect over the x-axis:

Second, shift left by 2:

Finally, shift down by 3:

42
www.ck12.org Chapter 1. Introduction to Functions

Example 5

Sketch the graph of f (x) = −2 x − 1.
The key to multiple transformations is to do them in order. Another way to keep track of which operations to do in
which order is to just do them in the order they appear in the equation, left to right.
√ √
To sketch the graph of f (x) = −2 x − 1 start with the parent y = x

First, reflect over the x-axis:

Second, stretch by 2:

43
1.8. Combining Transformations www.ck12.org

Third, shift right by 1:

MEDIA
Click image to the left or use the URL below.
URL: http://www.ck12.org/flx/render/embeddedobject/187160

44
www.ck12.org Chapter 1. Introduction to Functions

1.9 References

1. . . CC BY-NC-SA
2. CK-12 Foundation. Desmos Graphing Calculator .
3. CK-12 Foundation. Desmos Graphing Calculator .
4. CK-12 Foundation. Desmos Graphing Calculator .
5. CK-12 Foundation. https://www.flickr.com/photos/oceanyamaha/4579317995/in/photolist-7YEcGx-VpLS3-
VosTt-r842zw-vmJczY-55pFUb-5EtzBt-aiq7Pb-eZHA5e-7xLNk4-6y4noo-b8GLcX-7YHqZQ-7cH5UJ-rsUq4n
-561KZR-dDPyUD-2rEXy-2rEXw-e1rZr-7cuFp-6U9BCi-2sbYk-2rEXv-2sbYj-oWbwC1-2sbYh-48Ytsy-dDPz
aB-2rEXx-8Y88F4-6HMSTc-5FpUBB-oSciLy-7xM9B8-8LSV1p-8LSUL8-7y5eCX-dg9tw8-2sd89-2sbYm-VpLJ
u-2rEXu-cHECdq-cPcUsm-5UcCCx-67azBF-cPcVe3-cPcUBh-7M5Nqr .
6. CK-12 Foundation. https://www.flickr.com/photos/oceanyamaha/4579317995/in/photolist-7YEcGx-VpLS3-
VosTt-r842zw-vmJczY-55pFUb-5EtzBt-aiq7Pb-eZHA5e-7xLNk4-6y4noo-b8GLcX-7YHqZQ-7cH5UJ-rsUq4n
-561KZR-dDPyUD-2rEXy-2rEXw-e1rZr-7cuFp-6U9BCi-2sbYk-2rEXv-2sbYj-oWbwC1-2sbYh-48Ytsy-dDPz
aB-2rEXx-8Y88F4-6HMSTc-5FpUBB-oSciLy-7xM9B8-8LSV1p-8LSUL8-7y5eCX-dg9tw8-2sd89-2sbYm-VpLJ
u-2rEXu-cHECdq-cPcUsm-5UcCCx-67azBF-cPcVe3-cPcUBh-7M5Nqr .
7. CK-12 Foundation. https://www.flickr.com/photos/oceanyamaha/4579317995/in/photolist-7YEcGx-VpLS3-
VosTt-r842zw-vmJczY-55pFUb-5EtzBt-aiq7Pb-eZHA5e-7xLNk4-6y4noo-b8GLcX-7YHqZQ-7cH5UJ-rsUq4n
-561KZR-dDPyUD-2rEXy-2rEXw-e1rZr-7cuFp-6U9BCi-2sbYk-2rEXv-2sbYj-oWbwC1-2sbYh-48Ytsy-dDPz
aB-2rEXx-8Y88F4-6HMSTc-5FpUBB-oSciLy-7xM9B8-8LSV1p-8LSUL8-7y5eCX-dg9tw8-2sd89-2sbYm-VpLJ
u-2rEXu-cHECdq-cPcUsm-5UcCCx-67azBF-cPcVe3-cPcUBh-7M5Nqr .
8. CK-12 Foundation. https://www.flickr.com/photos/oceanyamaha/4579317995/in/photolist-7YEcGx-VpLS3-
VosTt-r842zw-vmJczY-55pFUb-5EtzBt-aiq7Pb-eZHA5e-7xLNk4-6y4noo-b8GLcX-7YHqZQ-7cH5UJ-rsUq4n
-561KZR-dDPyUD-2rEXy-2rEXw-e1rZr-7cuFp-6U9BCi-2sbYk-2rEXv-2sbYj-oWbwC1-2sbYh-48Ytsy-dDPz
aB-2rEXx-8Y88F4-6HMSTc-5FpUBB-oSciLy-7xM9B8-8LSV1p-8LSUL8-7y5eCX-dg9tw8-2sd89-2sbYm-VpLJ
u-2rEXu-cHECdq-cPcUsm-5UcCCx-67azBF-cPcVe3-cPcUBh-7M5Nqr .
9. dev Moore;CK-12 Foundation. https://www.flickr.com/photos/devinmoore/187215281/in/photolist-hxwzF-
9Y2dT2-8R5iar-9Y2cLM-ePfP1W-eP4pca-epUxUk-ePfP4W-ceh8k3-3dMceZ-aehC8w-gmZd-e3dfr7-ac7c3c-
dMr5kM-6bfCWV-6vpzyo-eKFqm1-iEv4o-aYNoV6-aWuFqv-9Uomhm-aWuFb4-eLWJx6-9Y2d4v-aWuFg4-
633FZc-7L1mSC-aWuFn6-eKFqkC-7nAuor-9yD6vQ-7jAj2m-aWuFja-bXcHpR-7q9MQ4-eqQMBS-4V9irC-
4V9isQ-ePfPis-9mMCH4-7qeaJq-ePfNzy-b3SGm-eP4ok2-eP4oZD-7q9Y2k-ePfP9o-eKFxDm-7qdvyU . CC
BY-NC-SA
10. dev Moore;CK-12 Foundation. https://www.flickr.com/photos/devinmoore/187215281/in/photolist-hxwzF-
9Y2dT2-8R5iar-9Y2cLM-ePfP1W-eP4pca-epUxUk-ePfP4W-ceh8k3-3dMceZ-aehC8w-gmZd-e3dfr7-ac7c3c-
dMr5kM-6bfCWV-6vpzyo-eKFqm1-iEv4o-aYNoV6-aWuFqv-9Uomhm-aWuFb4-eLWJx6-9Y2d4v-aWuFg4-
633FZc-7L1mSC-aWuFn6-eKFqkC-7nAuor-9yD6vQ-7jAj2m-aWuFja-bXcHpR-7q9MQ4-eqQMBS-4V9irC-
4V9isQ-ePfPis-9mMCH4-7qeaJq-ePfNzy-b3SGm-eP4ok2-eP4oZD-7q9Y2k-ePfP9o-eKFxDm-7qdvyU . CC
BY-NC-SA
11. dev Moore;CK-12 Foundation. https://www.flickr.com/photos/devinmoore/187215281/in/photolist-hxwzF-
9Y2dT2-8R5iar-9Y2cLM-ePfP1W-eP4pca-epUxUk-ePfP4W-ceh8k3-3dMceZ-aehC8w-gmZd-e3dfr7-ac7c3c-
dMr5kM-6bfCWV-6vpzyo-eKFqm1-iEv4o-aYNoV6-aWuFqv-9Uomhm-aWuFb4-eLWJx6-9Y2d4v-aWuFg4-
633FZc-7L1mSC-aWuFn6-eKFqkC-7nAuor-9yD6vQ-7jAj2m-aWuFja-bXcHpR-7q9MQ4-eqQMBS-4V9irC-
4V9isQ-ePfPis-9mMCH4-7qeaJq-ePfNzy-b3SGm-eP4ok2-eP4oZD-7q9Y2k-ePfP9o-eKFxDm-7qdvyU . CC
BY-NC-SA
12. dev Moore;CK-12 Foundation. https://www.flickr.com/photos/devinmoore/187215281/in/photolist-hxwzF-
9Y2dT2-8R5iar-9Y2cLM-ePfP1W-eP4pca-epUxUk-ePfP4W-ceh8k3-3dMceZ-aehC8w-gmZd-e3dfr7-ac7c3c-
dMr5kM-6bfCWV-6vpzyo-eKFqm1-iEv4o-aYNoV6-aWuFqv-9Uomhm-aWuFb4-eLWJx6-9Y2d4v-aWuFg4-

45
1.9. References www.ck12.org

633FZc-7L1mSC-aWuFn6-eKFqkC-7nAuor-9yD6vQ-7jAj2m-aWuFja-bXcHpR-7q9MQ4-eqQMBS-4V9irC-
4V9isQ-ePfPis-9mMCH4-7qeaJq-ePfNzy-b3SGm-eP4ok2-eP4oZD-7q9Y2k-ePfP9o-eKFxDm-7qdvyU . CC
BY-NC-SA

46
www.ck12.org Chapter 2. Algebraic Expressions

C HAPTER
2 Algebraic Expressions
Chapter Outline
2.1 A DDITION AND S UBTRACTION OF P OLYNOMIALS
2.2 M ULTIPLYING P OLYNOMIALS
2.3 FACTORING P OLYNOMIALS
2.4 S IMPLIFYING R ATIONAL E XPRESSIONS
2.5 M ULTIPLYING AND D IVIDING R ATIONAL E XPRESSIONS
2.6 A DDING AND S UBTRACTING R ATIONAL E XPRESSIONS
2.7 R EFERENCES

47
2.1. Addition and Subtraction of Polynomials www.ck12.org

2.1 Addition and Subtraction of Polynomials

Learning Objectives

Here you’ll learn how to add and subtract polynomials and simplify your answers. You’ll also solve real-world
problems using addition and subtraction of polynomials.

Addition and Subtraction of Polynomials

To add two or more polynomials, write their sum and then simplify by combining like terms.

MEDIA
Click image to the left or use the URL below.
URL: http://www.ck12.org/flx/render/embeddedobject/133009

Adding Polynomials

Add and simplify the resulting polynomials.


a) Add 3x2 − 4x + 7 and 2x3 − 4x2 − 6x + 5

(3x2 − 4x + 7) + (2x3 − 4x2 − 6x + 5)


Group like terms: = 2x3 + (3x2 − 4x2 ) + (−4x − 6x) + (7 + 5)
Simplify: = 2x3 − x2 − 10x + 12

b) Add x2 − 2xy + y2 and 2y2 − 3x2 and 10xy + y3

(x2 − 2xy + y2 ) + (2y2 − 3x2 ) + (10xy + y3 )


Group like terms: = (x2 − 3x2 ) + (y2 + 2y2 ) + (−2xy + 10xy) + y3
Simplify: = −2x2 + 3y2 + 8xy + y3

To subtract one polynomial from another, add the opposite of each term of the polynomial you are subtracting.

48
www.ck12.org Chapter 2. Algebraic Expressions

Subtracting Polynomials

a) Subtract x3 − 3x2 + 8x + 12 from 4x2 + 5x − 9

(4x2 + 5x − 9) − (x3 − 3x2 + 8x + 12) = (4x2 + 5x − 9) + (−x3 + 3x2 − 8x − 12)


Group like terms: = −x3 + (4x2 + 3x2 ) + (5x − 8x) + (−9 − 12)
Simplify: = −x3 + 7x2 − 3x − 21

b) Subtract 5b2 − 2a2 from 4a2 − 8ab − 9b2

(4a2 − 8ab − 9b2 ) − (5b2 − 2a2 ) = (4a2 − 8ab − 9b2 ) + (−5b2 + 2a2 )
Group like terms: = (4a2 + 2a2 ) + (−9b2 − 5b2 ) − 8ab
Simplify: = 6a2 − 14b2 − 8ab

Note: An easy way to check your work after adding or subtracting polynomials is to substitute a convenient value
in for the variable, and check that your answer and the problem both give the same value. For example, in part (b)
above, if we let a = 2 and b = 3, then we can check as follows:

Given Solution
2 2 2 2
(4a − 8ab − 9b ) − (5b − 2a ) 6a2 − 14b2 − 8ab
(4(2)2 − 8(2)(3) − 9(3)2 ) − (5(3)2 − 2(2)2 ) 6(2)2 − 14(3)2 − 8(2)(3)
(4(4) − 8(2)(3) − 9(9)) − (5(9) − 2(4)) 6(4) − 14(9) − 8(2)(3)
(−113) − 37 24 − 126 − 48
− 150 − 150

Since both expressions evaluate to the same number when we substitute in arbitrary values for the variables, we can
be reasonably sure that our answer is correct.
Note: When you use this method, do not choose 0 or 1 for checking since these can lead to common problems.

Problem Solving Using Addition or Subtraction of Polynomials

One way we can use polynomials is to find the area of a geometric figure.

Writing a Polynomial

Write a polynomial that represents the area of each figure shown.


a)

49
2.1. Addition and Subtraction of Polynomials www.ck12.org

This shape is formed by two squares and two rectangles.

The blue square has area y × y = y2 .


The yellow square has area x × x = x2 .
The pink rectangles each have area x × y = xy.

To find the total area of the figure we add all the separate areas:

Total area = y2 + x2 + xy + xy
= y2 + x2 + 2xy

b)

This shape is formed by two squares and one rectangle.

The yellow squares each have area a × a = a2 .


The orange rectangle has area 2a × b = 2ab.

To find the total area of the figure we add all the separate areas:

Total area = a2 + a2 + 2ab


= 2a2 + 2ab

c)

To find the area of the green region we find the area of the big square and subtract the area of the little square.

50
www.ck12.org Chapter 2. Algebraic Expressions

The big square has area : y × y = y2 .


The little square has area : x × x = x2 .
Area o f the green region = y2 − x2

d)

To find the area of the figure we can find the area of the big rectangle and add the areas of the pink squares.

The pink squares each have area a × a = a2 .


The blue rectangle has area 3a × a = 3a2 .

To find the total area of the figure we add all the separate areas:

Total area = a2 + a2 + a2 + 3a2 = 6a2

Another way to find this area is to find the area of the big square and subtract the areas of the three yellow squares:

The big square has area 3a × 3a = 9a2 .


The yellow squares each have area a × a = a2 .

To find the total area of the figure we subtract:

Area = 9a2 − (a2 + a2 + a2 )


= 9a2 − 3a2
= 6a2

51
2.1. Addition and Subtraction of Polynomials www.ck12.org

MEDIA
Click image to the left or use the URL below.
URL: http://www.ck12.org/flx/render/embeddedobject/133010

Example

Example 1

Subtract 4t 2 + 7t 3 − 3t − 5 from 6t + 3 − 5t 3 + 9t 2 .
When subtracting polynomials, we have to remember to subtract each term. If the term is already negative, subtract-
ing a negative term is the same thing as adding:

The final answer is in standard form.

52
www.ck12.org Chapter 2. Algebraic Expressions

2.2 Multiplying Polynomials

Learning Objectives

Here you’ll multiply together several different types of polynomials.


The length of a rectangular garden plot is x3 + 5x2 − 1. The width of the plot is x2 + 3. What is the area of the garden
plot?

Multiplying Polynomials

Multiplying together polynomials is very similar to multiplying together factors. You can FOIL or we will also
present an alternative method. When multiplying together polynomials, you will need to use the properties of
exponents, primarily the Product Property (am · an = am+n ) and combine like terms.

MEDIA
Click image to the left or use the URL below.
URL: http://www.ck12.org/flx/render/embeddedobject/176186

Let’s find the following products.

1. (x2 − 5)(x3 + 2x − 9)

Using the FOIL method, you need be careful. First, take the x2 in the first polynomial and multiply it by every term
in the second polynomial.

Now, multiply the -5 and multiply it by every term in the second polynomial.

Lastly, combine any like terms. In this example, only the x3 terms can be combined.

53
2.2. Multiplying Polynomials www.ck12.org

2. (x2 + 4x − 7)(x3 − 8x2 + 6x − 11)

In this problem, we will use the “box” method. Align the two polynomials along the top and left side of a rectangle
and make a row or column for each term. Write the polynomial with more terms along the top of the rectangle.

Multiply each term together and fill in the corresponding spot.

Finally, combine like terms. The final answer is x5 − 4x4 − 33x3 + 69x2 − 86x + 77. This method presents an
alternative way to organize the terms. Use whichever method you are more comfortable with. Keep in mind, no
matter which method you use, you will multiply every term in the first polynomial by every term in the second.

3. (x − 5)(2x + 3)(x2 + 4)

In this problem we have three binomials. When multiplying three polynomials, start by multiplying the first two
binomials together.

(x − 5)(2x + 3) = 2x2 + 3x − 10x − 15


= 2x2 − 7x − 15

Now, multiply the answer by the last binomial.

(2x2 − 7x − 15)(x2 + 4) = 2x4 + 8x2 − 7x3 − 28x − 15x2 − 60


= 2x4 − 7x3 − 7x2 − 28x − 60

54
www.ck12.org Chapter 2. Algebraic Expressions

Examples

Example 1

Earlier, you were asked to find the area of the garden plot.
Recall that the area of a rectangle is A = lw, where l is the length and w is the width. Therefore, we need to multiply.

A = (x3 + 5x2 − 1)(x2 + 3)


= x5 + 3x3 + 5x4 + 15x2 − x2 − 3

Now combine like terms and simplify. Be sure to write your answer in standard form

x5 + 3x3 + 5x4 + (15x2 − x2 ) − 3


= x5 + 3x3 + 5x4 + 14x2 − 3
= x5 + 5x4 + 3x3 + 14x2 − 3

Therefore, the area of the garden plot is x5 + 5x4 + 3x3 + 14x2 − 3.

Example 2

Find the product: −2x2 (3x3 − 4x2 + 12x − 9).


Use the distributive property to multiply −2x2 by the polynomial.

−2x2 (3x3 − 4x2 + 12x − 9) = −6x5 + 8x4 − 24x3 + 18x2

Example 3

Find the product: (4x2 − 6x + 11)(−3x3 + x2 + 8x − 10).


Multiply each term in the first polynomial by each one in the second polynomial.

(4x2 − 6x + 11)(−3x3 + x2 + 8x − 10) = −12x5 + 4x4 + 32x3 − 40x2


+ 18x4 − 6x3 − 48x2 + 60x
− 33x3 + 11x2 + 88x − 110
= −12x5 + 22x4 − 7x3 − 77x2 + 148x − 110

Example 4

Find the product: (x2 − 1)(3x − 4)(3x + 4).


Multiply the first two binomials together.

(x2 − 1)(3x − 4) = 3x3 − 4x2 − 3x + 4

55
2.2. Multiplying Polynomials www.ck12.org

Multiply this product by the last binomial.

(3x3 − 4x2 − 3x + 4)(3x + 4) = 9x4 + 12x3 − 12x3 − 16x2 − 9x2 − 12x + 12x − 16
= 9x4 − 25x2 − 16

Example 5

Find the product: (2x − 7)2 .


The square indicates that there are two binomials. Expand this and multiply.

(2x − 7)2 = (2x − 7)(2x − 7)


= 4x2 − 14x − 14x + 49
= 4x2 − 28x + 49

56
www.ck12.org Chapter 2. Algebraic Expressions

2.3 Factoring Polynomials

Introduction
In this lesson you will learn to factor polynomials by removing the greatest common factor. You will also learn to
factor polynomials of the form ax2 + bx + c where a = 1. You will learn a method known as the ‘box’ method to do
this factoring. You will use the same method to factor polynomials of the form ax2 + bx + c where a 6= 1. The last
polynomials that you will earn to factor are those known as the sum and difference of two squares.
Objectives
The lesson objectives for Factoring Polynomials are:

• Factoring a common factor


• Factoring ax2 + bx + c where a = 1
• Factoring ax2 + bx + c where a 6= 1
• Factoring the sum and difference of two squares

Factoring a Common Factor

Introduction
In this concept you will learn how to factor polynomials by looking for a common factor. You have used common
factors before in earlier courses. Common factors are numbers (numerical coefficients) or letters (literal coefficients)
that are a factor in all parts of the polynomials. In earlier courses of mathematics you would have studied common
factors of two numbers. Say you had the numbers 25 and 35. Two of the factors of 25 are 5 × 5. Two of the factors
of 35 are 5 × 7. Therefore a common factor of 25 and 35 would be 5.
In this concept of Lesson Factoring Polynomials, you will be using the notion of common factors to factor
polynomials. Sometimes simple polynomials can be factored by looking for a common factor among the terms
in the polynomial. Often, for polynomials, this is referred to as the Greatest Common Factor. The Greatest Common
Factor (or GCF) is the largest monomial that is a factor of (or divides into evenly) each of the terms of the polynomial.
This concept will be the first explored in Lesson Factoring Polynomials.
Watch This
Khan Academy Factoring and the Distributive Property

MEDIA
Click image to the left or use the URL below.
URL: http://www.ck12.org/flx/render/embeddedobject/59347

Guidance
Factor the following polynomial: 12x4 + 6x3 + 3x2 .
Step 1: Identify the GCF of the polynomial.

57
2.3. Factoring Polynomials www.ck12.org

If you look at just the factors of the numbers you can see the following:

12x4 + 6x3 + 3x2


. ↓ &
1 × 12 1×6 1×3
2×6 2×3
3×4

Looking at the factors for each of the numbers, you can see that 12, 6, and 3 can all be divided by 3.
Also notice that each of the terms has an x2 in common.

12x4 = 12 · x · x · x · x
6x3 = 6 · x · x · x
3x2 = 3 · x · x

So the GCF for this polynomial is 3x2


Step 2: Divide out the GCF from each term of the polynomial.

12x4 + 6x3 + 3x2 = 3x2 (4x2 + 2x + 1)


. ↓ &
Remember the Remember the Remember
factors of 12 factors of 6 the factors of
included 3 × 4 included 2 × 3 3 are 1 × 3
and x · x = x and x2 · x = x3 and x2 · 1 = x2
2 2 4

Example A
Factor the following binomial: 5a + 15
Step 1: Identify the GCF.

5a + 15
. &
1×5 1 × 15
3×5

Looking at the factors for each of the numbers, you can see that 5 and 15 can both be divided by 5.
So the GCF for this binomial is 5.
Step 2: Divide out the GCF from each term of the binomial.
5a + 15 = 5(a + 3)
Example B

58
www.ck12.org Chapter 2. Algebraic Expressions

Factor the following polynomial: 4x2 + 8x − 2


Step 1: Identify the GCF.

4x2 + 8x − 2
. ↓ &
1×4 1×8 1×2
2×2 2×4

Looking at the factors for each of the numbers, you can see that 4, 8, and 2 can all be divided by 2.
So the GCF for this polynomial is 2.
Step 2: Divide out the GCF from each term of the polynomial.
4x2 + 8x − 2 = 2(2x2 + 4x − 1)
Example C
Factor the following polynomial: 3x5 − 9x3 − 6x2
Step 1: Identify the GCF.

3x5 − 9x3 − 6x2


. ↓ &
1×3 1×9 1×6
3×3 2×3

Looking at the factors for each of the numbers, you can see that 3, 9, and 6 can all be divided by 3.
Also notice that each of the terms has an x2 in common.

3x5 = 3 · x · x · x · x · x
−9x3 = −9 · x · x · x
−6x2 = −6 · x · x

So the GCF for this polynomial is 3x2 .


Step 2: Divide out the GCF from each term of the polynomial.
3x5 − 9x3 − 6x2 = 3x2 (x3 − 3x − 2)
Vocabulary

Common Factor
Common factors are numbers (numerical coefficients) or letters (literal coefficients) that are factors in all
terms of the polynomials.

Greatest Common Factor


The Greatest Common Factor (or GCF) is the largest monomial that is a factor of (or divides into evenly)
each of the terms of the polynomial.

59
2.3. Factoring Polynomials www.ck12.org

Guided Practice

1. Find the common factors of the following: a2 (b + 7) − 6(b + 7)


2. Factor the following polynomial: 5k6 + 15k4 + 10k3 + 25k2
3. Factor the following polynomial: 27x3 y + 18x2 y2 + 9xy3

Answers
1. a2 (b + 7) − 6(b + 7)
Step 1: Identify the GCF.

a2 (b + 7) − 6(b + 7)

This problem is a little different in that if you look at the expression you notice that (b + 7) is common in both terms.
Therefore (b + 7) is the common factor.
So the GCF for this expression is (b + 7).
Step 2: Divide out the GCF from each term of the expression.

a2 (b + 7) − 6(b + 7) = (a2 − 6)(b + 7)

2. 5k6 + 15k4 + 10k3 + 25k2


Step 1: Identify the GCF.

5k6 + 15k4 + 10k3 + 25k2


. ↓ ↓ &
1×5 1 × 15 1 × 10 1 × 25
3×5 2×5 5×5

Looking at the factors for each of the numbers, you can see that 5, 15, 10, and 25 can all be divided by 5.
Also notice that each of the terms has an k2 in common.

5k6 = 5 · k · k · k · k · k · k
15k4 = 15 · k · k · k · k
10k3 = 10 · k · k · k
25k2 = 25 · k · k

So the GCF for this polynomial is 5k2 .


Step 2: Divide out the GCF out of each term of the polynomial.

5k6 + 15k4 + 10k3 + 25k2 = 5k2 (k4 + 3k2 + 2k + 5)

3. 27x3 y + 18x2 y2 + 9xy3

60
www.ck12.org Chapter 2. Algebraic Expressions

Step 1: Identify the GCF.

27x3 y + 18x2 y2 + 9xy3


. ↓ &
1 × 27 1 × 18 1×9
3×9 2×9 3×3
3×6

Looking at the factors for each of the numbers, you can see that 27, 18, and 9 can all be divided by 9.
Also notice that each of the terms has an xy in common.

27x3 y = 27 · x · x · x · y
18x2 y2 = 18 · x · x · y · y
9xy3 = 9 · x · y · y · y

So the GCF for this polynomial is 9xy.


Step 2: Divide out the GCF out of each term of the polynomial
27x3 y + 18x2 y2 + 9xy3 = 9xy(3x2 + 2xy + y2 )
Summary
Remember that when you factor expressions such as polynomials by taking out a common factor, the first step is to
find the greatest common factor for all of the terms in the polynomial. Once you have found this common factor,
you must divide each of the terms of the polynomial by this common factor.
The common factor can be a number, a variable, or a combination of both. This means that you need to look at both
the numbers in the terms of the polynomial and the variables of the polynomial.

Factoring ax2 + bx + c where a = 1

Introduction
When there are no common factors, you have to have an alternate method to factor polynomials. In this concept you
will begin to factor polynomials that are known as quadratic expressions. A quadratic expression is one in which
one variable will have an exponent of two and all other variables will have an exponent of one. The general form
of a quadratic expression is ax2 + bx + c where ‘a’ and ‘b’ are the coefficients of x2 and x, respectively, and ‘c’ is
a constant. The term ax2 is the one that is necessary to have a quadratic expression. These expressions are often
the result of multiplying polynomials. In this concept, you will be working with quadratic expressions where the
coefficient for ‘a’ is equal to 1.
Quadratic expressions of the form ax2 + bx + c can be factored using algebra tiles. To do this, the tiles modeling the
trinomial are laid out. Next, with the x2 tile in the upper left position the remaining tiles are arranged around it to
form a rectangle. By forming a rectangle, you can easily see the factors of the quadratic expression. Remember you
have used algebra tiles before in previous lessons and chapters. The large square tile represents an x2 , the rectangular
tile represents the x tile, and the small squares represent the 1’s. These are shown for you below.

61
2.3. Factoring Polynomials www.ck12.org

Remember as well that the reverse of these colored tiles are white and this represents the negatives.

Watch This
Khan Academy Factoring trinomials with a leading 1 coefficient

MEDIA
Click image to the left or use the URL below.
URL: http://www.ck12.org/flx/render/embeddedobject/59348

Guidance
An electronic sign is used to advertise specials outside a local store. It is shaped like a rectangle. If the dimensions
of the sign are known to be represented by the trinomial x2 − 5x + 6, find the binomials that represent the length and
width of the electronic sign.
The first step to solving this problem is to lay out the tiles you need to represent the trinomial (or the quadratic
expression).

62
www.ck12.org Chapter 2. Algebraic Expressions

The second step would be to arrange these tiles to form a rectangle.

The width of the rectangle is (x − 2).


The length of the rectangle is (x − 3).
Therefore the factors of x2 − 5x + 6 are (x − 3)(x − 2) or (x − 2)(x − 3).
Example A
Factor x2 + 5x + 6
Step 1: Place the tiles that represent the trinomial in front of you.
1 : x2 tile
5 : x tiles
6 : 1 tiles

Step 2: Now, form the rectangular using the tiles.

63
2.3. Factoring Polynomials www.ck12.org

The width of the rectangle is (x + 2).


The length of the rectangle is (x + 3).
Therefore the factors of x2 + 5x + 6 are (x + 3)(x + 2) or (x + 2)(x + 3).
Example B
Factor x2 − 3x + 2
Step 1: Place the tiles that represent the trinomial in front of you.
1 : x2 tile
3 : −x tiles
2 : 1 tiles

Step 2: Now, form the rectangular using the tiles.

64
www.ck12.org Chapter 2. Algebraic Expressions

The width if the rectangle is (x − 1).


The length of the rectangle is (x − 2).
Therefore the factors of x2 − 3x + 2 are (x − 2)(x − 1) or (x − 1)(x − 2).
Example C
Factor x2 − 7x + 12
Step 1: Place the tiles that represent the trinomial in front of you.
1 : x2 tile
7 : −x tiles
12 : 1 tiles

Step 2: Now, form the rectangular using the tiles.

65
2.3. Factoring Polynomials www.ck12.org

The width if the rectangle is (x − 3).


The length of the rectangle is (x − 4).
Therefore the factors of x2 − 7x + 12 are (x − 3)(x − 4) or (x − 4)(x − 3).
Vocabulary

Quadratic Expression
A quadratic expression is a trinomial in which one variable will have an exponent of two and all other
variables will have an exponent of one. The general form of a quadratic expression is ax2 + bx + c where ‘a’
and ‘b’ are the coefficients of x2 and x, respectively, and ‘c’ is a constant.

Guided Practice

1. Factor a2 + 6a + 8.
2. Factor x2 − 2x − 8.
3. Find the binomial dimensions of a rectangle that is represented by the trinomial h2 + 10h + 16.

Answers
1. Factor a2 + 6a + 8.
Step 1: Place the tiles that represent the trinomial in front of you.
1 : x2 tile
6 : +x tiles
8 : 1 tiles

Step 2: Now, form the rectangular using the tiles.

66
www.ck12.org Chapter 2. Algebraic Expressions

The width if the rectangle is (a + 2).


The length of the rectangle is (a + 4).
Therefore the factors of a2 + 6a + 8. are (a + 2)(a + 4) or (a + 4)(a + 2).
2. Factor x2 − 2x − 8.
Step 1: Place the tiles that represent the trinomial in front of you. In this polynomial you need to find the factors of
8. You can’t build a rectangle with just 2 "−x" tiles and 8 "-1" tiles.
The factors of 8 are 1 × 8, and 2 × 4. Notice that if you use 4 "−x" tiles and 2 +x tiles you will end up with 2 "−x"
tiles left over. Try to build a rectangle with these.
1 : x2 tile
4 : −x tiles
2 : +x tiles
8 : −1 tiles

Step 2: Now, form the rectangular using the tiles.

The width if the rectangle is (x + 2)


The length of the rectangle is (x − 4)
Therefore the factors of x2 − 2x − 8 are (x + 2)(x − 4) or (x − 4)(x + 2)
3. Find the binomial dimensions of a rectangle that is represented by the trinomial h2 + 10h + 16.
The first step to solving this problem is to lay out the tiles you need to represent the trinomial.

67
2.3. Factoring Polynomials www.ck12.org

The second step would be to arrange these tiles to form a rectangle.

The width of the rectangle is (h + 2).


The length of the rectangle is (h + 8).
Therefore the factors of h2 + 10h + 16 are (h + 8)(h + 2) or (h + 2)(h + 8).
Summary
In this concept you again used algebra tiles but you used them to factor trinomials known as quadratic expressions.
These special trinomials were in the form of ax2 + bx + c where a = 1. Algebra tiles are useful for factoring quadratic
expressions because when you lay out the tiles you need and then form your rectangle, you have the binomial factors
of the quadratic. Sometimes this approach is not useful, like when the constant (c) is very large. In these cases,
algebra tiles become very cumbersome to use. In the next few concepts, you will begin to explore other methods for
factoring quadratics.

Factoring ax2 + bx + c where a 6= 1

Introduction
In the last concept you learned how to factor quadratics or trinomials (ax2 + bx + c) where a = 1. You learned that
algebra tiles were very useful as a tool for solving these problems and that by building rectangles, finding the factors

68
www.ck12.org Chapter 2. Algebraic Expressions

of quadratics was a fast way to find the solution. In this lesson, you will work with quadratics, or trinomials, where
the value of a does not equal 1. In these problems, algebra tiles may still be useful but may, at some point, become
cumbersome due to the large numbers of tiles necessary to build the rectangles. You will use algebra to solve these
problems. There are a few other methods that are useful for solving quadratics where a 6= 1. One method you learned
earlier is the FOIL pattern. Remember that with FOIL, you multiply the First two terms, the Outside terms, the
Inside terms and the Last terms. So for example, with the trinomial:

(2x − 5)(x + 7) = 2x2 + 9x − 35

When you factor trinomials, you are really just using FOIL in reverse. Let’s look at the example below. With the
trinomial above (2x2 + 9x − 35), you had to factor both the 2 and the -35 (both the first number and the last number).
You can say then, in general terms, that with the trinomial ax2 + bx + c, you have to factor both “a” and “c”.

ax2 + bx + c = (dx + e)( f x + g)


where a = d × f and c = e × g

The middle term (b) is the sum of the outside two and the inside two terms. Therefore

b = dg + e f

In this lesson you will have the opportunity to work through a number of examples to develop mastery at factoring
trinomials of the form ax2 + bx + c where a 6= 1 using this reverse FOIL method. Now let’s begin.
Guidance
Jack wants to construct a border around his garden. The garden measures 5 yards by 18 yards. He has enough stone
to build a border with a total area of 39 square yards. The border will be twice as wide on the shorter end. What are
the dimensions of the border?

Area of Garden = 18 × 5 = 90 yd 2
Area of border = 39 yd 2
Area of Garden + border = (12 + 2x)(5 + x)
Area of border = (Area of garden + border) − Area of garden
39 = (18 + x)(5 + x) − 90
39 = 90 + 18x + 10x + 2x2 − 90
39 = 23x + 2x2
0 = 2x2 + 23x − 39

69
2.3. Factoring Polynomials www.ck12.org

Remember: ax2 + bx + c = (dx + e)( f x + g)


To factor this trinomial try this method:
In this trinomial, the ‘a’ value is 2 and the ‘c’ value is -39. These values are placed in a box
As shown in the illustration below

The product of 2 and -39 is -78. Find the factors for -78 and look for the factor pair that will combine (add or
subtract) to give the ‘b’ value of +23.

1 − 78 −1 74
2 − 39 −2 39
3 − 26 −3 26
6 − 13 −6 13

From the list, -3 and 26 will work. Put the factors in the box as shown.

Remember the Greatest common factor from the first part of this Lesson. Going across the horizontal rows in the
box, find the GCF of 2 and 26 it will be 2. Find the GCF of -3 and -39 it will be -3. Put these numbers in the box as
shown.

THEN - going down the vertical rows...


Find the GCF of 2 and -3 it will be 1. Find the GCF of 26 and -39 it will be 13.
Put these numbers in the box as shown.

70
www.ck12.org Chapter 2. Algebraic Expressions

These new numbers represent your factors. (1x + 13)(2x − 3)


So 2x2 + 23x − 39 = (1x + 13)(2x − 3). To find the dimensions of the border:

(x + 13)(2x − 3) = 0
. &
x + 13 = 0 2x − 3 = 0
3
x = −13 x=
2

Since x cannot be negative, x must equal 23 .


Back to the question!

3

Width of Border: 2x = 2 2 = 3 yd
3
Length of Border: x = 2 yd
Example A
Factor: 2x2 + 11x + 15
In this trinomial, the ‘a’ value is 2 and the ‘c’ value is 15. Place these values in a box.

71
2.3. Factoring Polynomials www.ck12.org

The product of 2 and 15 is 30. Next find the factors for 30 then look for the factor pair that will combine (add or
subtract) to give the ‘b’ value of +11.

1 30
2 15
3 10
5 6

From the list, 5 and 6 will work. Put the factors in the box as shown.

Going across the horizontal rows in the box, find the GCF of 2 and 6. It will be 2. Find the GCF of 5 and 15. It will
be 5. Put these numbers in the box as shown.

THEN - going down the vertical rows...


Find the GCF of 2 and 5. It will be 1. Find the GCF of 6 and 15. It will be 3.
Put these numbers in the box as shown.

These new numbers represent your factors. (1x + 3)(2x + 5)

72
www.ck12.org Chapter 2. Algebraic Expressions

So 2x2 + 11x + 15 = (x + 3)(2x + 5)


Example B
Factor: 3x2 − 8x − 3
In this trinomial, the ‘a’ value is 3 and the ‘c’ value is -3. Place the values in a box.

The product of 3 and -3 is -9. Find the factors for -9 and look for the factor pair that will combine (add or subtract)
to give the ‘b’ value of -8.

1 −9 −1 9
3 −3

From the list, 1 and -9 will work. Put the factors in the box as shown.

Going across the horizontal rows in the box, find the GCF of 3 and 1. It will be 1. Find the GCF of -9 and -3. It will
be -3. Put these numbers in the box as shown.

THEN - going down the vertical rows...


Find the GCF of 3 and -9. It will be 3. Find the GCF of 1 and -3. It will be 1.
Put these numbers in the box as shown.

73
2.3. Factoring Polynomials www.ck12.org

These new numbers represent your factors. (3x + 1)(1x − 3)


So 3x2 − 8x − 3 = (3x + 1)(x − 3)
Example C
Factor: 5w2 − 21w + 18
In this trinomial, the ‘a’ value is 5 and the ‘c’ value is 18. Place the values in a box.

The product of 5 and 18 is 90. Find the factors for 90 and look for the factor pair that will combine (add or subtract)
to give the ‘b’ value of -21.

1 90 −1 − 90
2 45 −2 − 45
3 30 −3 − 30
5 18 −5 − 18
6 15 −6 − 15
9 10 −9 − 10

From the list, -6 and -15 will work. Put the factors in the box as shown.

Going across the horizontal rows in the box, find the GCF of 5 and -6. It will be 1. Find the GCF of -15 and 18. It
will be 3. Put these numbers in the box as shown.

74
www.ck12.org Chapter 2. Algebraic Expressions

THEN - going down the vertical rows...


Find the GCF of 5 and -15. It will be 5. Find the GCF of -6 and 18. It will be 6.
Put these numbers in the box as shown.

These new numbers represent your factors. (5x − 6)(x − 3)


So 5w2 − 21w + 18 = (5x − 6)(x − 3)
Vocabulary

Greatest Common Factor


The Greatest Common Factor (or GCF) is the largest monomial that is a factor of (or divides into evenly)
each of the terms of the polynomial.

Quadratic Expression
A quadratic expression is a trinomial in which one variable will have an exponent of two and all other
variables will have an exponent of one. The general form of a quadratic expression is ax2 + bx + c where ‘a’
and ‘b’ are the coefficients of x2 and x, respectively, and ‘c’ is a constant.

Guided Practice

1. Factor the following trinomial: 8c2 − 2c − 3


2. Factor the following trinomial: 3m2 + 3m − 60
3. Factor the following trinomial: 5e3 + 30e2 + 40e

75
2.3. Factoring Polynomials www.ck12.org

Answers
Factor...
1. 8c2 − 2c − 3
In this trinomial, the ‘a’ value is 8 and the ‘c’ value is -3. Place the values in a box.

The product of 8 and -3 is -24. Next, find the factors for -24 and look for the factor pair that will combine (add or
subtract) to give the ‘b’ value of -2.

1 − 24 −1 24
2 − 12 −2 12
3 −8 −3 8
4 −6 −4 6

From the list, 4 and -6 will work. Put the factors in the box as shown.

Going across the horizontal rows in the box, find the GCF of 8 and 4. It will be 4. Find the GCF of -6 and -3. It will
be -3. Put these numbers in the box as shown.

THEN - going down the vertical rows...


Find the GCF of 8 and -6. It will be 2. Find the GCF of 4 and -3. It will be 1.
Put these numbers in the box as shown.

76
www.ck12.org Chapter 2. Algebraic Expressions

These new numbers represent your factors. (2c + 1)(4c − 3)


So 8c2 − 2c − 3 = (2c + 1)(4c − 3)
2. 3m2 + 3m − 60
First you can factor out the 3 from the polynomial

3m2 + 3m − 60 = 3(m2 + m − 20)

In this simpler trinomial, the ‘a’ value is 1 and the ‘c’ value is -20. Place the values in a box.

The product of 1 and -20 is -20. Next, find the factors for -20 and look for the factor pair that will combine (add or
subtract) to give the ‘b’ value of 1.

1 − 20 −1 20
2 − 10 −2 10
4 −5 −4 5

From the list, -4 and 5 will work. Put the factors in the box as shown.

Going across the horizontal rows in the box, find the GCF of 1 and -4. It will be 1. Find the GCF of 5 and -20. It
will be 5. Put these numbers in the box as shown.

77
2.3. Factoring Polynomials www.ck12.org

THEN - going down the vertical rows...


Find the GCF of 1 and 5. It will be 1. Find the GCF of -4 and -20. It will be -4.
Put these numbers in the box as shown.

These new numbers represent your factors. (1m − 4)(1m + 5)


So 3m2 + 3m − 60 = 3(m − 4)(m + 5)
3. 5e3 + 30e2 + 40e
First you can factor out the 5 and one e from the polynomial

5e3 + 30e2 + 40e = 5e(e2 + 6e + 8)

First, use the box for the simpler trinomial to factor it:

The product of 1 and 8 is 8.


Need: factors for 8 and a pair that will combine to give the ‘b’ value of 6.

78
www.ck12.org Chapter 2. Algebraic Expressions

Next: GCFs

Last: Put it together


These new numbers represent your factors. (1e + 2)(1e + 4)
So 5e3 + 30e2 + 40e = 5e(e + 2)(e + 4)
Summary
In the first lesson of this section you learned how to take a common factor out of a polynomial. In the next section
you learned to factor trinomials (or quadratics) of the form ax2 + bx + c where a = 1 using algebra tiles. In this
lesson, you continued working with trinomials (or quadratics) but in these quadratics a 6= 1. As well in this lesson,
you were introduced to a neat method known as the box method. With this method, any quadratic can be solved by
finding the factors that multiply together to give “ac” and combine to give “b”. Using the box method and by finding
GCFs, the factors of the quadratic can be found.

79
2.3. Factoring Polynomials www.ck12.org

Factoring the Sum and Difference of Two Squares

Introduction
In lesson Addition and Subtraction of Polynomials you learned about three special cases where binomials can be
expanded using the distributive property to make polynomials. These special cases were

1. (x + y)2 = x2 + 2xy + y2
2. (x − y)2 = x2 − 2xy + y2
3. (x + y)(x − y) = x2 − y2

In this lesson, you will again be working with these special cases. The first two special cases are the sum of two
squares. These are often called perfect square trinomials. The third special case is called the difference of two
squares. Rather than expanding the binomials to make polynomials as you did in the previous lesson, here you will
be factoring the special case polynomials. Factoring is really the reverse of multiplication!
You can use algebra tiles or the box method. Recall that algebra tiles are very visual but can be cumbersome when
you need to use a lot of tiles to find the factors.
Watch This
Khan Academy Factoring the Sum and Difference of Squares

MEDIA
Click image to the left or use the URL below.
URL: http://www.ck12.org/flx/render/embeddedobject/59349

Guidance
A box is to be designed for packaging with a side length represented by the quadratic 9b2 − 64. If this is the most
economical box, what are the dimensions?
First: factor the quadratic to find the value for b.
9b2 − 64
First, the box:

The product of 3 and -64 is -576.


Need: factors for -576 and a pair that will combine to give the ‘b’ value of 0. Start with taking the square root of
576 since you need to have two numbers that are the same. This way they will add to give you 0 and multiply to give
576.

80
www.ck12.org Chapter 2. Algebraic Expressions

Next: GCFs

Last: Put it together


These new numbers represent your factors. (3b + 8)(3b − 8)
So

9b2 − 64 = (3b + 8)(3b − 8)


. &
3b + 8 = 0 3b − 8 = 0
3b = −8 3b = 8
−8 8
b= b=
3 3

The most economical box is a cube. Therefore the dimensions are 83 × 83 × 83 .


Example A
Factor

81
2.3. Factoring Polynomials www.ck12.org

2x2 + 28x + 98
First, the box:

The product of 2 and 98 is 196.


Need: factors for 196 and a pair that will combine to give the ‘b’ value of 28. Start with taking the square root of
196 since if this is a perfect square it will require two numbers that are the same. This way the middle terms would
be the same and would still multiply to give 196. You could also find the factors of 196. This would give you two
numbers that add up to 28 and multiply to give you 196.

Next: GCFs

82
www.ck12.org Chapter 2. Algebraic Expressions

Last: Put it together


These new numbers represent your factors. (2x + 14)(2x + 14)
So 2x2 + 28x + 98 = (2x + 14)(2x + 14) Or
2x2 + 28x + 98 = (2x + 14)2 SPECIAL CASE 1
Example B
Factor:
8a2 − 24a + 18
First, let’s factor out a 2 from this expression (the GCF!)
8a2 − 24a + 18 = 2(4a2 − 12a + 9)
Next the box:

The product of 4 and 9 is 36.


Need: factors for 36 and a pair that will combine to give the ‘b’ value of -12. Start with taking the square root of
36 since if this is a perfect square it will require two numbers that are the same. This way the middle terms would
be the same and would still multiply to give 36. You could also find the factors of 36. This would give you two
numbers that add up to -12 and multiply to give you 36.

83
2.3. Factoring Polynomials www.ck12.org

Next: GCFs

Last: Put it together


These new numbers represent your factors.

So 8a2 − 24a + 18 = 2(2a − 3)(2a − 3)

84
www.ck12.org Chapter 2. Algebraic Expressions

Or
8a2 − 24a + 18 = 2(2a − 3)2 SPECIAL CASE 2
Example C
Factor:
x2 − 16
First the box:

The product of 1 and 16 is 16.


Need: factors for 16 and a pair that will combine to give the ‘b’ value of 0. Start with taking the square root of 16
since if this is a difference of two squares it will require two numbers that are the same. This way the middle terms
would be equal and cancel each other out; plus the numbers would still multiply to give 16.

Next: GCFs

85
2.3. Factoring Polynomials www.ck12.org

Last: Put it together


These new numbers represent your factors.

So x2 − 16 = (x − 4)(x + 4) SPECIAL CASE 3


Vocabulary

Difference of Two Squares


The difference of two squares is a pattern found in polynomial expressions. It is a special case where there is
no middle term found in the quadratic expression. The general equation for the difference of two squares is:

x2 + y2 = (x + y)(x − y)

Sum of Two Squares


The sum of two squares involves two special patterns found in polynomial expressions. These are special
cases where the middle term is twice the product of the first and last term. For the quadratic expression
ax2 + bx + c, the sum of two squares has a middle term (b) equal to 2ac. These trinomials are often called
perfect square trinomials. The general equations for the sum of two square patterns are:

1. (x + y)2 = x2 + 2xy + y2

86
www.ck12.org Chapter 2. Algebraic Expressions

2. (x − y)2 = x2 − 2xy + y2

Guided Practice

1. Factor completely s2 − 18s + 81


2. Factor completely 50 − 98x2
3. Factor completely 4x2 + 36x + 144

Answers
1. s2 − 18s + 81
First the box:

The product of 1 and 81 is 81.


Need: factors for 81 and a pair that will combine to give the ‘b’ value of -18. Start with taking the square root of 81
since if this is a perfect square it will require two numbers that are the same. This way the middle terms would be
the same and would still multiply to give 81. You could also find the factors of 81.

Next: GCFs

87
2.3. Factoring Polynomials www.ck12.org

Last: Put it together


These new numbers represent your factors.

So s2 − 18s + 81 = (s − 9)(s − 9)
Or
s2 − 18s + 81 = (s − 9)2 SPECIAL CASE 2
2. 50 − 98x2
First: let’s factor out a 2 from this expression (the GCF!)
50 − 98x2 = 2(25 − 49x2 )
Next the box:

88
www.ck12.org Chapter 2. Algebraic Expressions

The product of 25 and 49 is 1225. WOW!


Need: factors for 1225 and a pair that will combine to give the ‘b’ value of 0. Start with taking the square root of
1225 since if this is a difference of two squares it will require two numbers that are the same. This way the middle
terms would equal each other and cancel each other out; plus the numbers would still multiply to give 16.

Next: GCFs

Last: Put it together


These new numbers represent your factors.

89
2.3. Factoring Polynomials www.ck12.org

So 50 − 98x2 = 2(5 − 7x)(5 + 7x) SPECIAL CASE 3


3. 4x2 + 24x + 44
First, let’s factor out a 4 from this expression (the GCF!)
4x2 + 48x + 144 = 4(x2 + 12x + 36)
Next the box:

The product of 1 and 36 is 36.


Need: factors for 36 and a pair that will combine to give the ‘b’ value of 12. Start with taking the square root of 36
since if this is a perfect square it will require two numbers that are the same. This way the middle terms would be
the same and would still multiply to give 36.

Next: GCFs

90
www.ck12.org Chapter 2. Algebraic Expressions

Last: Put it together


These new numbers represent your factors. (1x + 6)(1x + 6)
So 4x2 + 48x + 144 = 4(x + 6)(x + 6)
Or
4x2 + 48x + 144 = 4(x + 6)2 SPECIAL CASE 1
Summary
In this final lesson you have had the opportunity to apply your skills of factoring to the special cases that you learned
about in the previous lesson. Remember the three special cases for polynomials:
A: The Sum of Two Squares
1. (x + y)2 = x2 + 2xy + y2
2. (x − y)2 = x2 − 2xy + y2
B: The Difference of Two Squares
3. x2 + y2 = (x + y)(x − y)
It is easier if you are able to recognize the special cases in that you do not need to find all of the factors for a and b.
Rather, you simply need to find the square root of the “ac” term. In special cases 1 and 2, the middle term is always
2ac. For special case 3, there is no middle term. Both algebra tiles and the box method provide you with useful
methods for factoring these special cases as they did with other polynomials.

Summary

In this lesson, you have worked with adding and subtracting polynomials. You also worked with multiplying
polynomials. Remember that when multiplying polynomials, the distributive property is the tool to use. You began
your look at the special cases of polynomials and learned that the more you recognize these special cases, the quicker
it is to work with them when you are factoring.
In this current lesson, you continued your learning of polynomials by working with the greatest common factor.
Make the question simpler by first removing the number or letter that is common in all terms of your polynomial.
You also began factoring quadratics (or trinomials) where the coefficient for a in ax2 + bx + c was equal to 1 and
then for cases where a was not equal to 1. Quadratics where a was not equal to one seemed a bit more complex but
remember the strategy for solving these remains the same.
Lastly, in this current lesson, you worked with factoring polynomials for three special cases. You may have noticed
that these special cases were those you worked with in lesson Addition and Subtraction of Polynomials, only here
you were factoring them. Again, strategies for factoring the special case polynomials remain the same. As well, if
you are able to recognize a quadratic (trinomial) as a special case, factoring becomes somewhat less cumbersome.

91
2.4. Simplifying Rational Expressions www.ck12.org

2.4 Simplifying Rational Expressions

Learning Objectives

Here you’ll learn how to simplify rational expressions involving factorable polynomials.
The area of a rectangle is 2x4 − 2. The width of the rectangle is x2 + 1. What is the length of the rectangle?

Rational Expressions

p(x)
Recall that a rational function is a function, f (x), such that f (x) = q(x) , where p(x) and q(x) are both polynomials.
p(x)
A rational expression, is just q(x) .
Like any fraction, a rational expression can be simplified. To simplify a rational
expression, you will need to factor the polynomials, determine if any factors are the same, and then cancel out any
like factors.
9
Fraction: 15 = 3·3 = 35
3·5
2 +6x+9 (x
+3)
(x+3)
Rational Expression: xx2 +8x+15 = x+3
+ 3)(x+5) = x+5
(x 


With both fractions, we broke apart the numerator and denominator into the prime factorization. Then, we canceled
the common factors.
3x 3
Important Note: x+3 x+3
x+5 is completely factored. Do not cancel out the x’s! 5x reduces to 5 , but x+5 does not because
of the addition sign. To prove this, we will plug in a number for x to and show that the fraction does not reduce to 35 .
2+3
If x = 2, then 2+5 = 57 6= 35 .
Let’s simplify the following expressions.

2x3
1. 4x2 −6x

The numerator factors to be 2x3 = 2 · x · x · x and the denominator is 4x2 − 6x = 2x(2x − 3).
2x3
4x2 −6x
= 2·x·x·x = x2
2x−3
2·x·(2x−3)

6x2 −7x−3
2. 2x3 −3x2

Factor the numerator and find the GCF of the denominator and cancel out the like terms.
6x2 −7x−3 −
(2x 3)
(3x+1)
3
2x −3x 2 = 
x2 −
(2x  = 3x+1
3) x2

x2 −6x+27
3. 2x2 −19x+9

Factor both the top and bottom and see if there are any common factors.
x2 −6x+27 (x−9)
(x+3)
x+3
2
2x −19x+9
= (x − 9)(2x−1) = 2x−1

 


Special Note: Not every polynomial in a rational function will be factorable. Sometimes there are no common
factors. When this happens, write “not factorable.”

92
www.ck12.org Chapter 2. Algebraic Expressions

Examples

Example 1

Earlier, you were asked to find the length of the rectangle.


Recall that the the area of a rectangle is the length times the width. To find the length, we can therefore divide the
4 −2
area by the width. So we’re looking for 2x x2 +1
.
If we factor the numerator and the denominator, we get:

2x4 − 2
x2 + 1
2(x4 − 1)
x2 + 2
2(x + 1)(x2 − 1)
2

x2 + 1
2(x − 1) = 2x2 − 2
2

Therefore, the length of the rectangle is 2(x2 − 1) = 2x2 − 2.


If possible, simplify the following rational functions.

Example 2

3x2 −x
3x2
3x2 −x
3x2
= x(3x−1)
x·x =
3·
3x−1
3x

Example 3

x2 +6x+8
x2 +6x+9
x2 +6x+8 (x+4)(x+2)
x2 +6x+9
= (x+3)(x+3) There are no common factors, so this is reduced.

Example 4

2x2 +x−10
6x2 +17x+5

2x2 +x−10 (2x


+5)
(x−2)
x−2
= (2x

5)(3x+1) =
6x2 +17x+5 + 3x+1


Example 5

x3 −4x
x5 +4x3 −32x
In this problem, the denominator will factor like a quadratic once an x is pulled out of each term.
((
x3 −4x x(x2 −4) x(x−2)(x+2) x(x(−(2)(x
((+ 2)
x5 +4x3 −32x
= x(x +4x2 −32)
4 = x(x2 −4)(x2 +8)
= ( ( = x21+8
x(x
( ( −( ( (+
2)(x 2)
( 2
(x +8)

93
2.5. Multiplying and Dividing Rational Expressions www.ck12.org

2.5 Multiplying and Dividing Rational


Expressions
Introduction
In this lesson you will revisit multiplying and dividing fractions. After this brief review, you will then apply these
concepts to multiplying and dividing rational expressions. To express the answer in simplest form, the rational
expressions will be factored and then the common factors will be cancelled. The restrictions on the denominator will
also be stated.
Objectives
The lesson objectives for Multiplying and Dividing Rational Expressions are:

• A review of multiplying and dividing fractions


• Applying this same method to multiplying and dividing rational expressions
• Simplifying the answer
• Stating the restrictions

Multiplying and Dividing Rational Expressions

Introduction
Following the same procedure as with rational numbers, you can multiply and divide rational expressions. The
process of dividing fractions is simply multiplying by the reciprocal of the divisor. The following example will
demonstrate two methods for multiplying rational numbers.
2
21 × 35
11
A common denominator is not necessary for multiplying fractions. Multiply the numerators and multiply the
denominators.

2 × 35
21 × 11
70
231

Reduce the fraction to lowest terms by dividing both the numerator and the denominator by 7.

70 ÷ 7
231 ÷ 7
10
=
33

OR
2
21 × 35
11
Factor the numerator and the denominator of each fraction.

94
www.ck12.org Chapter 2. Algebraic Expressions

2
3×7 × 5×7
11
Express the factored fractions as one common fraction.
2×5×7
3×7×11
Cancel, one for one, all factors that are common to both the numerator and the denominator.
2×5×7

3×7
×11
Multiply the remaining factors in both the numerator and the denominator.
10
=
33
Watch This
Khan Academy Multiplying and Dividing Rational Expressions

MEDIA
Click image to the left or use the URL below.
URL: http://www.ck12.org/flx/render/embeddedobject/63393

Guidance
Multiply each of the following and state the restrictions.
10y+20 y−3
5y−15 × y2 +10y+16
Begin by factoring the numerator and denominator of each expression.

10y + 20 y−3
× 2
5y − 15 y + 10y + 16
10y + 20 GCF = 10 → 10(y + 2)
5y − 15 GCF = 5 → 5(y − 3)
y2 + 10y + 16 TEST → (8)(2) = 16 and (8) + (2) = 10
= (y + 8)(y + 2)

Rewrite the rational expressions with the factors.


10(y+2) y−3
5(y−3) × (y+8)(y+2)
Express the factored rational expressions as a single rational expression.
10(y+2)(y−3)
5(y−3)(y+8)(y+2)
Cancel the common factors that exist in the numerator and the denominator.
2
(y
+  −
10
 (y
2) 3)


5 −
(y (y
3)(y+8)
 +2)


The result of cancelling the common factors is the answer.


2
=
y+8

95
2.5. Multiplying and Dividing Rational Expressions www.ck12.org

The restrictions on the denominator apply to all the factors of the denominator.
Identify the restrictions.

The solution is:


2
; y 6= 3; y 6= −8; y 6= −2
y+8
Example A
Multiply the following rational expressions and state the restrictions.
4x−8 2 −3x−10
x2 −7x+10
×x x2 −4
Begin by factoring the numerator and denominator of each expression.
4x−8 2 −3x−10
x2 −7x+10
×x x2 −4

4x − 8 GCF = 4 → 4(x − 2)
2
x − 7x + 10 TEST → (−5)(−2) = 10 and (−5) + (−2) = −7
= (x − 5)(x − 2)
x2 − 3x − 10 TEST → (−5)(2) = −10 and (−5) + (2) = −3
= (x − 5)(x + 2)
√ √
x2 − 4 Difference of squares x2 = x and 4 = 2
= (x + 2)(x − 2)

Rewrite the rational expressions with the factors.


4(x−2)
(x−5)(x−2) × (x−5)(x+2)
(x+2)(x−2)
Express the factored rational expressions as a single rational expression.
4(x−2)(x−5)(x+2)
(x−5)(x−2)(x+2)(x−2)
Cancel the common factors that exist in the numerator and the denominator.
4 −
(x 2) −
(x (x
5) + 2)

(x
 − 5)

 (x− 2)
 (x+ 2)(x−2)


96
www.ck12.org Chapter 2. Algebraic Expressions

The result of cancelling the common factors is the answer.


4
(x−2)
The restrictions on the denominator apply to all the factors of the denominator.
Identify the restrictions.

The solution is:


4
= ; x 6= 5; x 6= 2; x 6= −2
(x − 2)
Example B
Divide the following rational expressions and state the restrictions.
m2 −4 2−6m
m2 +9m+14
÷ 3m
m2 −49
To divide rational expressions, multiply by the reciprocal of the divisor. Then, follow the process for multiplying
rational expressions.
m2 −4 2
× m −49
m2 +9m+14 3m2 −6m
Begin by factoring the numerator and denominator of each expression.
m2 −4 2
m −49
m2 +9m+14
× 3m 2 −6m

√ √
m2 − 4 Difference of squares m2 = m and 4 = 2
(m + 2)(m − 2)
m2 + 9m + 14 TEST → (7)(2) = 14 and (7) + (2) = 9
= (m + 7)(m + 2)
√ √
m2 − 49 Difference of squares m2 = m and 49 = 7
= (m + 7)(m − 7)
3m2 − 6m GCF = 3m → 3m(m − 2)

Rewrite the rational expressions with the factors.

97
2.5. Multiplying and Dividing Rational Expressions www.ck12.org

(m+2)(m−2)
(m+7)(m+2) × (m+7)(m−7)
3m(m−2)
Express the factored rational expressions as a single rational expression.
(m+2)(m−2)(m+7)(m−7)
3m(m−2)(m+7)(m+2)
Cancel the common factors that exist in the numerator and the denominator.
(m+2)−
(m (m
2)+ 7)
(m−7)

(m− 2)
3m  (m+ 7)
 (m+2)


The result of cancelling the common factors is the answer.


(m−7)
3m
The restrictions on the denominator apply to all the factors of the denominator.
Identify the restrictions.

m+7 = 0
m+7−7 = 0−7
m = −7 − 7 would make the factor (m + 7) = 0. Therefore,
m 6= −7

m−2 = 0
m−2+2 = 0+2
m=2 2 would make the factor (m − 2) = 0. Therefore,
m 6= 2

m+2 = 0
m+2−2 = 0−2
m = −2 − 2 would make the factor (m + 2) = 0. Therefore,
m 6= −2

3m = 0
3m 0
=
3 3
m = 0 0 would make the factor (3m) = 0. Therefore,
m 6= 0

The solution is:


m−7
= ; m 6= −7; m 6= 2; x 6= −2; m 6= 0
3m
Example C
Simplify the following rational expression and state the restrictions.
12x2 +13x−35 2+16x+21
5x2 −21x+18
÷ 3x
5x2 +9x−18

98
www.ck12.org Chapter 2. Algebraic Expressions

To divide rational expressions, multiply by the reciprocal of the divisor. Then, follow the process for multiplying
rational expressions.
12x2 +13x−35 5x2 +9x−18
5x2 −21x+18
× 3x 2 +16x+21

Begin by factoring the numerator and denominator of each expression.


12x2 +13x−35 2
5x +9x−18
5x2 −21x+18
× 3x 2 +16x+21

TEST → (28)(−15) = −420

,
which when divided by the a value of 12 equals 35 and

(28) + (−15) = 13
12x2 + 13x − 35
(12x2 + 28x) + (−15x − 35)
4x(3x + 7) + −5(3x + 7)
= (4x − 5)(3x + 7)

TEST → (−15)(−6) = 90

,
which when divided by the a value of 5 equals 18 and

(−15) + (−6) = −21


5x2 − 21x + 18
(5x2 − 15x) + (−6x + 18)
5x(x − 3) + −6(x − 3)
= (5x − 6)(x − 3)

TEST → (15)(−6) = −90

, which when divided by the a value of 5 equals 18 and

and (15) + (−6) = 9


5x2 + 9x − 18
(5x2 + 15x) + (−6x − 18)
5x(x + 3) + −6(x + 3)
= (5x − 6)(x + 3)

99
2.5. Multiplying and Dividing Rational Expressions www.ck12.org

TEST → (9)(7) = 63
,
which when divided by the a value of 3 equals 21 and

(9) + (7) = 16
3x2 + 16x + 21
(3x2 + 9x) + (7x + 21)
3x(x + 3) + 7(x + 3)
= (3x + 7)(x + 3)

Rewrite the rational expressions with the factors.


(4x−5)(3x+7)
(5x−6)(x−3) × (5x−6)(x+3)
(3x+7)(x+3)
Express the factored rational expressions as a single rational expression.
(4x−5)(3x+7)(5x−6)(x+3)
(5x−6)(x−3)(3x+7)(x+3)
Cancel the common factors that exist in the numerator and the denominator.
(3x
+ −
(5x (x
7) 6) + 3)
(4x−5)

 − 6)(x−3)
(5x   (3x
+ 7)
 (x + 3)
 

The result of cancelling the common factors is the answer.


(4x−5)
(x−3)
The restrictions on the denominator apply to all the factors of the denominator.
Identify the restrictions.

5x − 6 = 0
5x − 6 + 6 = 0 + 6
5x 6 6
= would make the factor (5x − 6) = 0. Therefore,
5 5 5
6
x 6=
5

x−3 = 0
x−3+3 = 0+3
x=3 3 would make the factor (x − 3) = 0. Therefore,
x 6= 3

3x + 7 = 0
3x + 7 − 7 = 0 − 7
3x −7 −7
= would make the factor (3x + 7) = 0. Therefore,
3 3 3
−7
x 6=
3

100
www.ck12.org Chapter 2. Algebraic Expressions

x+3 = 0
x+3−3 = 0−3
x = −3 − 3 would make the factor (x + 3) = 0. Therefore,
x 6= −3

The solution is:


4x − 5 6 7
= ; x 6= ; m 6= 3; x 6= − ; m 6= −3
x−3 5 3
Vocabulary

Rational Expression
a(x)
A rational expression is an algebraic expression that can be written in the form b(x) where b 6= 0.

Restriction
Any value of the variable in a rational expression that would result in a zero denominator is called a restriction
on the denominator.

Guided Practice
1. Simplify the following and state the restrictions.
x+7 2 −2x−63 2 −15x+54
x2 −5x−36
÷x x+4 ·x x2 −36
2. Simplify the following and state the restrictions.
y2 −25 2 2

y2 −6y
· yy2−12y+36
+2y−15
÷ yy2−11y+30
+4y−21
3. Simplify the following and state the restrictions.
2x2 +7x−4 2 +7x−2
6x2 +x−2
· 15x
5x2 +19x−4
Answers
x+7 2 −2x−63 2 −15x+54
1. x2 −5x−36
÷x x+4 ·x x2 −36
Write the term after the division sign as a reciprocal and multiply.
x+7 2 −15x+54
× x+4
x2 −5x−36 x2 −2x−63
·x x2 −36
Factor the numerator and denominator of each expression.
x+7 x+4 2 −15x+54
x2 −5x−36
× x2 −2x−63 ·x x2 −36

x2 − 5x − 36 TEST → (−9)(4) = −36 and (−9) + (4) = −5


= (x − 9)(x + 4)
x2 − 2x − 63 TEST → (−9)(7) = −63 and (−9) + (7) = −2
= (x − 9)(x + 7)
x2 − 15x + 54 TEST → (−6)(−9) = 54 and (−6) + (−9) = −15
= (x − 6)(x − 9)
√ √
x2 − 36 Difference of squares x2 = x and 36 = 6
= (x + 6)(x − 6)

101
2.5. Multiplying and Dividing Rational Expressions www.ck12.org

Rewrite the rational expressions with the factors.


x+7 x+4 (x−9)(x−6)
(x−9)(x+4) × (x−9)(x+7) · (x+6)(x−6)
Express the factored rational expressions as a single rational expression.
(x+7)(x+4)(x−9)(x−6)
(x−9)(x+4)(x−9)(x+7)(x+6)(x−6)
Cancel the common factors that exist in the numerator and the denominator.
1
z }| {
(x+  −  −
(x
 +
7) (x
4) (x
9) 6)


 −
(x (x
9) +
(x−9)(x
4)  +  −
(x+6)(x
7) 6)


The result of cancelling the common factors is the answer.


1
(x−9)(x+6)
The restrictions on the denominator apply to all the factors of the denominator.
Identify the restrictions.

x−9 = 0
x−9+9 = 0+9
x=9 9 would make the factor (x − 9) = 0. Therefore,
x 6= 9

x+4 = 0
x+4−4 = 0−4
x = −4 − 4 would make the factor (x + 4) = 0. Therefore,
x 6= −4

x+7 = 0
x+7−7 = 0−7
x = −7 − 7 would make the factor (x + 7) = 0. Therefore,
x 6= −7

x+6 = 0
x+6−6 = 0−6
x = −6 − 6 would make the factor (x + 6) = 0. Therefore,
x 6= −6

x−6 = 0
x−6+6 = 0+6
x=6 6 would make the factor (x − 6) = 0. Therefore,
x 6= 6

102
www.ck12.org Chapter 2. Algebraic Expressions

The solution is:


1
= ; x 6= 9; x 6= −4; x 6= −7; x 6= −6; x 6= 6
(x − 9)(x − 6)
y2 −25 2 2
2. y2 −6y
· yy2−12y+36
+2y−15
÷ yy2−11y+30
+4y−21
Write the term after the division sign as a reciprocal and multiply.
y2 −25 2 2

y2 −6y
· yy2−12y+36
+2y−15
+4y−21
× yy2 −11y+30
Factor the numerator and denominator of each expression.
y2 −25 2 2

y2 −6y
· yy2−12y+36
+2y−15
+4y−21
× yy2 −11y+30

p √
y2 − 25 Difference of squares y2 = y and 25 = 5
= (y + 5)(y − 5)
y2 − 6y → GCF = y → y(y − 6)
y2 − 12y + 36 TEST → (−6)(−6) = 36 and (−6) + (−6) = −12
= (y − 6)(y − 6)
y2 + 2y − 15 TEST → (5)(−3) = −15 and (5) + (−3) = 2
= (y + 5)(y − 3)
y2 + 4y − 21 TEST → (7)(−3) = −21 and (7) + (−3) = 4
= (y + 7)(y − 3)
y2 − 11y + 30 TEST → (−6)(−5) = 30 and (−6) + (−5) = −11
= (y − 6)(y − 5)

Rewrite the rational expressions with the factors.


(y+5)(y−5)
y(y−6) · (y−6)(y−6) (y+7)(y−3)
(y+5)(y−3) × (y−6)(y−5)
Express the factored rational expressions as a single rational expression.
(y+5)(y−5)(y−6)(y−6)(y+7)(y−3)
y(y−6)(y+5)(y−3)(y−6)(y−5)
Cancel the common factors that exist in the numerator and the denominator.
(y+  − 5) −  − 6)(y+7) −
(y
5) (y
 (y
6) 
 (y 3)


y − 6)
(y 
 (y
+ 5)

 − 3)
(y 
 − 6)
(y 
 (y− 5)


The result of cancelling the common factors is the answer.


(y+7)
y
The restrictions on the denominator apply to all the factors of the denominator.
Identify the restrictions.

y=0 0 would make the factor (y) = 0. Therefore,


y 6= 0

103
2.5. Multiplying and Dividing Rational Expressions www.ck12.org

y−6 = 0
y−6+6 = 0+6
y=6 6 would make the factor (y − 6) = 0. Therefore,
y 6= 6

y+5 = 0
y+5−5 = 0−5
y = −5 − 5 would make the factor (y + 5) = 0. Therefore,
y 6= −5

y−3 = 0
y−3+3 = 0+3
y=3 3 would make the factor (y − 3) = 0. Therefore,
y 6= 3

y−5 = 0
y−5+5 = 0+5
y=5 5 would make the factor (y − 5) = 0. Therefore,
y 6= 5

The solution is:


y+7
= ; y 6= 0; y 6= 6; y 6= −5; y 6= 3; y 6= 5
y
2x2 +7x−4 2 +7x−2
3. 6x2 +x−2
· 15x
5x2 +19x−4
Factor the numerator and denominator of each expression.
2x2 +7x−4 2+7x−2
6x2 +x−2
· 15x
5x2 +19x−4

TEST → (8)(−1) = −8

, which when divided by the a value of 2 equals -4 and

(8) + (−1) = 7
2x2 + 7x − 4
(2x2 + 8x) + (−x − 4)
2x(x + 4) + −1(x + 4)
= (2x − 1)((x + 4))

104
www.ck12.org Chapter 2. Algebraic Expressions

TEST → (4)(−3) = −12

, which when divided by the a value of 6 equals -2 and

(4) + (−3) = 1
6x2 + x − 2
(6x2 + 4x) + (−3x − 2)
2x(3x + 2) + −1(3x + 2)
= (2x − 1)(3x + 2)

TEST → (10)(−3) = −30

,
which when divided by the a value of 15 equals -2 and

(10) + (−3) = 7
15x2 + 7x − 2
(15x2 + 10x) + (−3x − 2)
5x(3x + 2) + −1(3x + 2)
= (5x − 1)(3x + 2)

TEST → (20)(−1) = −20

, which when divided by the a value of 5 equals -4 and

(20) + (−1) = 19
5x2 + 19x − 4
(5x2 + 20x) + (−x − 4)
5x(x + 4) + −1(x + 4)
= (5x − 1)(x + 4)

Rewrite the rational expressions with the factors.


(2x−1)(x+4)
(2x−1)(3x+2) · (5x−1)(3x+2)
(5x−1)(x+4)
Express the factored rational expressions as a single rational expression.
(2x−1)(x+4)(5x−1)(3x+2)
(2x−1)(3x+2)(5x−1)(x+4)
Cancel the common factors that exist in the numerator and the denominator.

105
2.5. Multiplying and Dividing Rational Expressions www.ck12.org

1
z }| {
 −
(2x (x
1) + 4)
(5x − (3x
1) +
2)

 −
(2x (3x
1)
+ 2)
(5x − (x
1) +4)


The result of cancelling the common factors is the answer.


1
The restrictions on the denominator apply to all the factors of the denominator.
Identify the restrictions.

2x − 1 = 0
2x − 1 + 1 = 0 + 1
2x 1 1
= would make the factor (2x − 1) = 0. Therefore,
2 2 2
1
x 6=
2

3x + 2 = 0
3x + 2 − 2 = 0 − 2
3x −2 2
= − would make the factor (3x + 2) = 0. Therefore,
3 3 3
2
x 6= −
3

5x − 1 = 0
5x − 1 + 1 = 0 + 1
5x 1 1
= would make the factor (5x − 1) = 0. Therefore,
5 5 5
1
x 6=
5

x+4 = 0
x+4−4 = 0−4
x = −4 − 4 would make the factor (x + 4) = 0. Therefore,
x 6= −4

The solution is:


1 2 1
= 1; x 6= ; x 6= − ; x 6= ; x 6= −4
2 3 5
Summary
In this lesson you have learned to multiply and to divide rational expressions. The method applied was the same
as that used for multiplying and dividing fractions. The numerators and the denominators were all factored and
expressed as a single rational expression. The factors that were common to both the numerator and the denominator
were cancelled. The remaining factor(s) were the solution to the problem. The restrictions on the denominator must
be stated for all factors–even the factors that cancelled.

106
www.ck12.org Chapter 2. Algebraic Expressions

2.6 Adding and Subtracting Rational


Expressions
Introduction
In this lesson you will revisit adding and subtracting fractions. After this brief review, you will then apply these
concepts to adding and subtracting rational expressions. You will learn to determine a common denominator for the
expressions in order to add and subtract. You will then apply what you’ve learned in the previous lesson to ensure
that the final answer is expressed in simplest (lowest) terms.
Objectives
The lesson objectives for Adding and Subtracting Rational Expressions are:

• A review of adding and subtracting fractions


• Understanding a common denominator for rational expressions
• Adding rational expressions
• Subtracting rational expressions
• Simplifying the sum or difference

Adding and Subtracting Rational Expressions

Introduction
Following the same procedure as with rational numbers, you can add and subtract rational expressions by first
determining the lowest common denominator (LCD) and then adding or subtracting the numerators.
As an example, when adding or subtracting rational numbers:
3
5 + 72 → Identify the LCD → 5 × 7 = 35
7 3 5 2
 
7 5 + 5 7 → Multiply each fraction to express both with the LCD.
21 10
35 + 35 → Simplify. Put the numerators over the LCD.
21+10
35 → Write as one common fraction.
31
35 → Simplify. Add the numerators.
The sum of the fractions cannot be further simplified.
Watch This
Khan Academy Adding and Subtracting Rational Expressions

MEDIA
Click image to the left or use the URL below.
URL: http://www.ck12.org/flx/render/embeddedobject/63390

Khan Academy Adding and Subtracting Rational Expressions

107
2.6. Adding and Subtracting Rational Expressions www.ck12.org

MEDIA
Click image to the left or use the URL below.
URL: http://www.ck12.org/flx/render/embeddedobject/63391

Khan Academy Adding and Subtracting Rational Expressions

MEDIA
Click image to the left or use the URL below.
URL: http://www.ck12.org/flx/render/embeddedobject/63392

Guidance
Add the following rational expressions and state the restrictions.
3x 2x
x2 +6x−16
+ x−2
Factor the denominator of the first fraction.

x2 + 6x − 16 TEST → (8)(−2) = −16 and (8) + (−2) = 6


= (x + 8)(x − 2)

Replace the denominator with the factors.


3x 2x
(x+8)(x−2) + x−2
The LCD will consist of one of each factor from the denominators. If any factor is repeated in a single denominator,
then the LCD must have as many of that factor as repeats. In this example there are no repeated factors in a single
denominator so the LCD will be (x + 8)(x − 2).

What is different? On the left side, there is nothing different in the two denominators.
What is different? On the right side, there is a difference between the two denominators. The bottom denominator
2x
has (x + 8) but the top denominator does not have this binomial. This indicates that the rational expression x−2 must
be multiplied by (x + 8) to get the common denominator of (x + 8)(x − 2).
3x 2x x+8

(x+8)(x−2) + x−2 x+8
Apply the distributive property to the numerator of the right side. Keep the LCD as factors.
3x 2
2x +16x
(x+8)(x−2) + (x−2)(x+8)

108
www.ck12.org Chapter 2. Algebraic Expressions

Write the two expressions a one common rational expression.


3x+2x2 +16x
(x+8)(x−2)
Simplify the numerator by combining like terms.
2x2 +19x
(x+8)(x−2)
Identify the restrictions.

x+8 = 0
x+8−8 = 0−8
x = −8 − 8 would make the factor (x + 8) = 0. Therefore,
x 6= −8

x−2 = 0
x−2+2 = 0+2
x = 2 2 would make the factor (x − 2) = 0. Therefore,
x 6= 2

2x2 + 19x
; x 6= −8; x 6= 2
(x + 8)(x − 2)

Example A
Identify the lowest common denominator (LCD) in factored form.
2x−3 x−5
i) x2 −7x+10
− x2 −2x−15
2x+1
ii) x2 +6x+9
+ x23x−2
+x−6
To determine the LCD, begin by factoring the denominators.
2x−3 x−5
i) x2 −7x+10
− x2 −2x−15

x2 − 7x + 10 TEST → (−5)(−2) = 10 and (−5) + (−2) = −7


= (x − 5)(x − 2)
x2 − 2x − 15 TEST → (−5)(3) = −15 and (−5) + (3) = −2
= (x − 5)(x + 3)

Rewrite the expressions with the factored denominators.


2x−3 x−5
(x−5)(x−2) − (x−5)(x+3)
The LCD must consist of one of each factor - no factor is repeated in a single denominator.
The LCD is (x − 5)(x − 2)(x + 3)
ii) To determine the LCD, begin by factoring the denominators.
2x+1
x2 +6x+9
+ x23x−2
+x−6

109
2.6. Adding and Subtracting Rational Expressions www.ck12.org

x2 + 6x + 9 TEST → (3)(3) = 9 and (3) + (3) = 6


= (x + 3)(x + 3)
x2 + x − 6 TEST → (3)(−2) = −6 and (3) + −2 = 1
= (x + 3)(x − 2)

Rewrite the expressions with the factored denominators.


2x+1 3x−2
(x+3)(x+3) + (x+3)(x−2)
The LCD must consist of the two factors in the first denominator since the factor (x + 3) is repeated in this
denominator and the factor (x − 2) from the second denominator.
The LCD is (x + 3)(x + 3)(x − 2)
Example B
Perform the indicated operations and state the restrictions.
3x+1
i) x2 +8x+16
+ x22x−3
+x−12
x−3 x−4
ii) x2 −5x+6
− x2 −x−12
Begin by determining the LCD. Factor the denominators of each expression.
3x+1
i) x2 +8x+16
+ x22x−3
+x−12

x2 + 8x + 16 TEST → (4)(4) = 16 and (4) + (4) = 8


= (x + 4)(x + 4)
x2 + x − 12 TEST → (4)(−3) = −12 and (4) + (−3) = 1
= (x + 4)(x − 3)

Rewrite the expressions with the factored denominators.


3x+1 2x−3
(x+4)(x+4) + (x+4)(x−3)
The LCD must consist of the two factors in the first denominator since the factor (x + 4) is repeated in this
denominator and the factor (x − 3) from the second denominator.
The LCD is (x + 4)(x + 4)(x + 3)
Rewrite the two expressions with the LCD to compare them in order to determine the term(s) by which one
or both of the fractions must be multiplied.

What is different? On the left side, the term (x − 3) is different. This indicates that the numerator and the
3x+1
denominator of (x+4)(x+4) must be multiplied by (x − 3) to get the LCD of (x + 4)(x + 4)(x − 3) .

110
www.ck12.org Chapter 2. Algebraic Expressions

What is different? On the right side, the term (x + 4) is different. This indicates that the numerator and the
2x−3
denominator of (x+4)(x−3) must be multiplied by (x + 4) to get the LCD of (x + 4)(x + 4)(x + 3) .
3x+1 x−3 2x−3 x+4
 
(x+4)(x+4) x−3 + (x+4)(x−3) x+4
Apply the distributive property (FOIL) to the numerators. Keep the LCD as factors.
3x2 −8x−3 2
2x +5x−12
(x+4)(x+4)(x−3) + (x+4)(x+4)(x−3)
Write the two expressions as one common rational expression.
3x2 −8x−3+2x2 +5x−12
(x+4)(x+4)(x−3)
Simplify the numerator by combining like terms.
5x2 −3x−15
(x+4)(x+4)(x−3)
Identify the restrictions.

x+4 = 0
x+4−4 = 0−4
x = −4 − 4 would make the factor (X + 4) = 0. Therefore,
x 6= −4

x−3 = 0
x−3+3 = 0+3
x = 3 3 would make the factor (x − 3) = 0. Therefore,
x 6= 3

The numerator cannot be factored - it is prime. The answer in lowest terms is:
5x2 − 3x − 15
; x 6= −4; x 6= 3
(x + 4)(x + 4)(x − 3)
Determine the LCD.
x−3 x−4
ii) x2 −5x+6
− x2 −x−12

x2 − 5x + 6 TEST → (−3)(−2) = 6 and (−3) + (−2) = −5


= (x − 3)(x − 2)

x2 − x − 12 TEST → (−4)(3) = −12 and (−4) + (3) = −1


= (x − 4)(x + 3)

Rewrite each expression with its factored denominator.


x−3 x−4
(x−3)(x−2) − (x−4)(x+3)
There is a common factor (x − 3) in both the numerator and the denominator of the first expression. In the
second expression, there is a common factor of (x − 4) in both the numerator and the denominator. These
factors can be cancelled.

111
2.6. Adding and Subtracting Rational Expressions www.ck12.org

1 1
x−
 3 x−
 4

x − 3)(x − 2) 
(  x − 4(x + 3)

1 1

(x − 2) (x + 3)

The LCD is (x − 2)(x + 3)


Write the expressions with the LCD and determine the multiplier for one or both of the expressions.

What is different? On the left side, the term (x + 3) is different. This indicates that the numerator and the
1
denominator of (x−2) must be multiplied by (x + 3) to get the LCD of (x + 3)(x − 2) .
What is different? On the right side, the term (x − 2) is different. This indicates that the numerator and the
1
denominator of (x+3) must be multiplied by (x − 2) to get the LCD of (x + 3)(x − 2) .
1 x+3 1 x−2
 
(x−2) x+3 − (x+3) x−2
Apply the distributive property to the numerators. Keep the LCD as factors.
x+3 x−2
(x−2)(x+3) − (x−2)(x+3)
Write the two expressions as one common rational expression.
x+3−(x−2)
(x−2)(x+3)

Multiply (x − 2) by -1.
x+3−x+2
(x−2)(x+3)
Simplify the numerator by combining like terms.
5
(x−2)(x+3)
Identify the restrictions.

x−2 = 0
x−2+2 = 0+2
x = 2 2 would make the factor (x − 2) = 0. Therefore,
x 6= 2

x+3 = 0
x+3−3 = 0−3
x = −3 − 3 would make the factor (x + 3) = 0. Therefore,
x 6= −3

112
www.ck12.org Chapter 2. Algebraic Expressions

But don’t forget about the denominator terms you cancelled earlier.

x−3 = 0
x−3+3 = 0+3
x = 3 3 would make the factor (x − 3) = 0. Therefore,
x 6= 3

x−4 = 0
x−4+4 = 0+4
x = 4 4 would make the factor (x − 4) = 0. Therefore,
x 6= 4

The solution is:


5
; x 6= 2; x 6= −3; x 6= 3; x 6= 4
(x − 2)(x + 3)
Example C
Simplify the following rational expressions and state the restrictions.
x x−2
x2 −9x+18
− x2 −10x+24
Determine the LCD.

x2 − 9x + 18 TEST → (−3)(−6) = 18 and (−3) + (−6) = −9


= (x − 3)(x − 6)
x2 − 10x + 24 TEST → (−4)(−6) = 24 and (−4) + (−6) = −10
= (x − 4)(x − 6)

Write the expressions with the factored denominators.


x x−2
(x−6)(x−3) − (x−6)(x−4)

The LCD must consist of one of each factor. The LCD is (x − 6)(x − 3)(x − 4)

(x − 6)(x − 3)(x − 4) (x − 6)(x − 3)(x − 4)


(x − 6)(x − 3) (x − 6)(x − 4)
| {z } | {z }
multiply by (x−4) multiply by (x−3)

Multiply each expression by the indicated term to get the LCD.


x x−4 x−2 x−3
 
(x−6)(x−3) x−4 − (x−6)(x−4) x−3
Apply the distributive property to the numerators.
x2 −4x 2
x −5x+6
(x−6)(x−3)(x−4) − (x−6)(x−3)(x−4)
Write the expressions as one common rational expression.
x2 −4x−(x2 −5x+6)
(x−6)(x−3)(x−4)

113
2.6. Adding and Subtracting Rational Expressions www.ck12.org

Multiply (x2 − 5x + 6) by -1.


x2 −4x−x2 +5x−6
(x−6)(x−3)(x−4)
Simplify the numerator by combining like terms.
x−6
(x−6)(x−3)(x−4)

The term (x − 6) is common to both the numerator and the denominator. This term can be cancelled.

1
x−
 
6
x−
( 6)(x − 3)(x − 4)
1
(x − 3)(x − 4)

Identify the restrictions.

x−3 = 0
x−3+3 = 0+3
x = 3 3 would make the factor (x − 3) = 0. Therefore,
x 6= 3

x−4 = 0
x−4+4 = 0+4
x = 4 4 would make the factor (x − 4) = 0. Therefore,
x 6= 4

But don’t forget about the denominator term you cancelled.

x−6 = 0
x−6+6 = 0+6
x = 6 6 would make the factor (x − 6) = 0. Therefore,
x 6= 6

The solution is:


1
; x 6= 3; x 6= 4; x 6= 6
(x − 3)(x − 4)
Vocabulary

Rational Expression
a(x)
A rational expression is an algebraic expression that can be written in the form b(x) where b 6= 0.

Restriction
Any value of the variable in a rational expression that would result in a zero denominator is called a restriction
on the denominator.

114
www.ck12.org Chapter 2. Algebraic Expressions

Guided Practice
2x 1
1. Simplify the following and state the restrictions. x2 −4
− x−2
−2 3
2. Simplify the following and state the restrictions. 3y2 +5y+2
+ y2 −7y−8
3m−1
3. Simplify the following and state the restrictions. 9m3 −36m2
+ 2m22m+1
−5m−12
Answers
2x 1
1. x2 −4
− x−2
Determine the LCD.

√ √
x2 − 4 Difference of squares x2 = x and 4 = 2
= (x + 2)(x − 2)

Write the expressions with the factored denominators.


2x 1
(x+2)(x−2) − x−2

The LCD must consist of one of each factor. The LCD is (x + 2)(x − 2)

(x + 2)(x − 2) (x + 2)(x − 2)
(x + 2)(x − 2) (x − 2)
| {z } | {z }
no need to multiply multiply by (x+2)

Multiply the numerator and denominator of each expression by the indicated term to get the LCD.
2x 1 x+2

(x+2)(x−2) − (x−2) x+2
Apply the distributive property to the numerator on the right side.
2x x+2
(x+2)(x−2) − (x+2)(x−2)
Write the expressions as one common rational expression.
2x−(x+2)
(x+2)(x−2)

Multiply (x + 2) by -1.
2x−x−2
(x+2)(x−2)
Simplify the numerator by combining like terms.
x−2
(x+2)(x−2)

The term (x − 2) is common to both the numerator and the denominator. This term can be cancelled.

1
x−
 2

x−
(x + 2)( 
2)
1
(x + 2)

Identify the restrictions.

115
2.6. Adding and Subtracting Rational Expressions www.ck12.org

x+2 = 0
x+2−2 = 0−2
x = −2 − 2 would make the factor (x + 2) = 0. Therefore,
x 6= −2

Don’t forget the denominator term you cancelled.

x−2 = 0
x−2+2 = 0+2
x = 2 2 would make the factor (x − 2) = 0. Therefore,
x 6= 2

The solution is:


1
; x 6= −2; x 6= 2
(x + 2)
−2 3
2. 3y2 +5y+2
+ y2 −7y−8
Determine the LCD.

3y2 + 5y + 2 TEST → (3)(2) = 6 and (3) + (2) = 5


2
(3y + 3y) + (2y + 2)
3y(y + 1) + 2(y + 1)
= (3y + 2)(y + 1)

y2 − 7y − 8 TEST → (−8)(1) = −8 and (−8) + (1) = −7


= (y − 8)(y + 1)

Write the expressions with the factored denominators.


−2 3
(3y+2)(y+1) + (y−8)(y−1)

The LCD must consist of one of each factor. The LCD is (3y + 2)(y − 8)(y + 1)

(3y + 2)(y − 8)(y + 1) (3y + 2)(y − 8)(y + 1)


(3y + 2)(y + 1) (y − 8)(y + 1)
| {z } | {z }
multiply by (y−8) multiply by (3y+2)

Multiply the numerator and denominator of each expression by the indicated term to get the LCD.
   
−2 y−8 3 3y+2
(3y+2)(y+1) y−8 + (y−8)(y+1) 3y+2

Apply the distributive property to the numerators.


−2y+16 9y+6
(3y+2)(y−8)(y+1) + (3y+2)(y−8)(y+1)
Write the expressions as one common rational expression.

116
www.ck12.org Chapter 2. Algebraic Expressions

−2y+16+9y+6
(3y+2)(y−8)(y+1)
Simplify the numerator by combining like terms.
7y+22
(3y+2)(y−8)(y+1)
The expression cannot be simplified. There are no common factors.
7y+22
(3y+2)(y−8)(y+1)
Identify the restrictions.

3y + 2 = 0
3y + 2 − 2 = 0 − 2
3y = −2
3y −2 2
= − would make the factor (3y + 2) = 0. Therefore,
3 3 3
2
y 6= −
3

y−8 = 0
y−8+8 = 0+8
y=8 8 would make the factor (y − 8) = 0. Therefore,
y 6= 8

y+1 = 0
y+1−1 = 0−1
y = −1 − 1 would make the factor (y + 1) = 0. Therefore,
y 6= −1

The solution is:


7y + 22 2
; y 6= − ; y 6= 8; y 6= −1
(3y + 2)(y − 8)(y + 1) 3
3m−1
3. 9m3 −36m2
+ 2m22m+1
−5m−12
Determine the LCD.

9m3 − 36m2
9m3 − 36m2 → GCF = (9m2 ) = 9m2 (m − 4)

2m2 − 5m − 12 TEST → (−8)(3) = −24

which when divided by the a value of 2 equals -12 and...

117
2.6. Adding and Subtracting Rational Expressions www.ck12.org

(−8) + (3) = −5
(2m2 − 8m) + (3m − 12)
2m(m − 4) + 3(m − 4)
= (2m + 3)(m − 4)

Write the expressions with the factored denominators.


3m−1 2m+1
9m2 (m−4)
+ (2m+3)(m−4)

The LCD must consist of one of each factor. The LCD is 9m2 (m − 4)(2m + 3)

9m2 (m − 4)(2m + 3) 9m2 (m − 4)(2m + 3)


9m2 (m − 4) (2m + 3)(m − 4)
| {z } | {z }
multiply by (2m+3) multiply by (9m2 )

Multiply the numerator and denominator of each expression by the indicated term to get the LCD.
 2
3m−1 2m+3 2m+1 9m

2
9m (m−4) 2m+3
+ (2m+3)(m−4) 9m2

Apply the distributive property to the numerators.


6m2 +9m−2m−3 3
+9m 2
9m2 (m−4)(2m+3)
+ 9m218m
(2m+3)(m−4)

Write the expressions as one common rational expression.


6m2 +9m−2m−3+18m3 +9m2
9m2 (m−4)(2m+3)

Simplify the numerator by combining like terms.


18m3 +15m2 +7m−3
9m2 (m−4)(2m+3)

The expression cannot be simplified. There are no common factors.


18m3 +15m2 +7m−3
9m2 (m−4)(2m+3)

Identify the restrictions.

118
www.ck12.org Chapter 2. Algebraic Expressions

2m + 3 = 0
2m + 3 − 3 = 0 − 3
2m = −3
2m −3 3
= − would make the factor (2m + 3) = 0. Therefore,
2 2 2
3
m 6= −
2

m−4 = 0
m−4+4 = 0+4
m=4 4 would make the factor (m − 4) = 0. Therefore,
m 6= 4

9m2 = 0
9m2 0
=
9 9
m2 = 0 0 would make the factor (9m2 ) = 0. Therefore,
√ √
m2 = 0
m=0
m 6= 0

The solution is:


18m3 + 15m2 + 7m − 3 3
2
; m 6= − ; m 6= 4; m 6= 0
9m (m − 4)(2m + 3) 2
Summary
In this lesson you have learned to add and to subtract rational expressions. The method applied was the same as
that used for adding and subtracting fractions. The LCD of the rational expressions consisted of one of each of the
factors of each denominator unless a factor was repeated in a single denominator. If there were repeats in a single
denominator, then the repeats were included in the LCD.
Reducing the rational expressions was done once the denominators of the given expressions were factored and/or at
the end when the numerators were simplified and written as a one common rational expression. The restrictions on
the denominator were also stated.

119
2.7. References www.ck12.org

2.7 References

1. CK-12 Foundation. https://www.flickr.com/photos/potteryandeverythingelse/3205009474/in/photolist-pMpUi


A-dRbzB1-59Se3G-dwXZX4-6gYNvp-6LiDZh-6h3Yx9-krjLGS-rLvNs-eGzLzE-qKPaQB-714gpK-ccFM8A
-q8hShX-9H1H4m-qkBjoe-rfrqhQ-93Bd3X-cuKa4w-8en5DK-ewFdrK-5TmoDH-5TmoER-4zqSoK-rhKCqi-
5TdvCh-boZa4m-7UtS3Y-rAcU1K-xAS9Y3-dFMSq5-7YoJx7-7ukUjt-5TdvE3-boZa4h-7UqBq4-s5CZbu-5T9
azv-5TdvEL-Das2Uf-f2dM5M-6UcC5M-5T9ayM-CQimpt-rDSGV4-ymH7Bu-xxTef4-xtNX64-xiKqeE-rkQmYE
.
2. CK-12 Foundation. https://www.flickr.com/photos/potteryandeverythingelse/3205009474/in/photolist-pMpUi
A-dRbzB1-59Se3G-dwXZX4-6gYNvp-6LiDZh-6h3Yx9-krjLGS-rLvNs-eGzLzE-qKPaQB-714gpK-ccFM8A
-q8hShX-9H1H4m-qkBjoe-rfrqhQ-93Bd3X-cuKa4w-8en5DK-ewFdrK-5TmoDH-5TmoER-4zqSoK-rhKCqi-
5TdvCh-boZa4m-7UtS3Y-rAcU1K-xAS9Y3-dFMSq5-7YoJx7-7ukUjt-5TdvE3-boZa4h-7UqBq4-s5CZbu-5T9
azv-5TdvEL-Das2Uf-f2dM5M-6UcC5M-5T9ayM-CQimpt-rDSGV4-ymH7Bu-xxTef4-xtNX64-xiKqeE-rkQmYE
.
3. CK-12 Foundation. https://www.flickr.com/photos/potteryandeverythingelse/3205009474/in/photolist-pMpUi
A-dRbzB1-59Se3G-dwXZX4-6gYNvp-6LiDZh-6h3Yx9-krjLGS-rLvNs-eGzLzE-qKPaQB-714gpK-ccFM8A
-q8hShX-9H1H4m-qkBjoe-rfrqhQ-93Bd3X-cuKa4w-8en5DK-ewFdrK-5TmoDH-5TmoER-4zqSoK-rhKCqi-
5TdvCh-boZa4m-7UtS3Y-rAcU1K-xAS9Y3-dFMSq5-7YoJx7-7ukUjt-5TdvE3-boZa4h-7UqBq4-s5CZbu-5T9
azv-5TdvEL-Das2Uf-f2dM5M-6UcC5M-5T9ayM-CQimpt-rDSGV4-ymH7Bu-xxTef4-xtNX64-xiKqeE-rkQmYE
.
4. CK-12 Foundation. .
5. CK-12 Foundation. .
6. . . CC BY-NC
7. . . CC BY-NC
8. . . CC BY-NC
9. . . CC BY-NC
10. . . CC BY-NC
11. . . CC BY-NC
12. . . CC BY-NC
13. . . CC BY-NC
14. . . CC BY-NC
15. . . CC BY-NC
16. . . CC BY-NC
17. . . CC BY-NC
18. . . CC BY-NC
19. . . CC BY-NC
20. . . CC BY-NC
21. . . CC BY-NC
22. . . CC BY-NC
23. . . CC BY-NC
24. . . CC BY-NC
25. . . CC BY-NC
26. . . CC BY-NC
27. . . CC BY-NC
28. . . CC BY-NC
29. . . CC BY-NC
30. . . CC BY-NC

120
www.ck12.org Chapter 2. Algebraic Expressions

31. . . CC BY-NC
32. . . CC BY-NC
33. . . CC BY-NC
34. . . CC BY-NC
35. . . CC BY-NC
36. . . CC BY-NC
37. . . CC BY-NC
38. . . CC BY-NC
39. . . CC BY-NC
40. . . CC BY-NC
41. . . CC BY-NC
42. . . CC BY-NC
43. . . CC BY-NC
44. . . CC BY-NC
45. . . CC BY-NC
46. . . CC BY-NC
47. . . CC BY-NC
48. . . CC BY-NC
49. . . CC BY-NC
50. . . CC BY-NC
51. . . CC BY-NC
52. . . CC BY-NC
53. . . CC BY-NC
54. . . CC BY-NC
55. . . CC BY-NC
56. . . CC BY-NC
57. . . CC BY-NC
58. . . CC BY-NC
59. . . CC BY-NC
60. . . CC BY-NC
61. . . CC BY-NC
62. . . CC BY-NC
63. . . CC BY-NC
64. . . CC BY-NC
65. . . CC BY-NC
66. . . CC BY-NC
67. . . CC BY-NC
68. . . CC BY-NC
69. . . CC BY-NC
70. . . CC BY-NC
71. . . CC BY-NC
72. . . CC BY-NC
73. . . CC BY-NC
74. . . CC BY-NC
75. . . CC BY-NC
76. . . CC BY-NC
77. . . CC BY-NC
78. . . CC BY-NC

121
www.ck12.org

C HAPTER
3 Quadratic Functions
Chapter Outline
3.1 P ROPERTIES OF Q UADRATIC F UNCTIONS
3.2 V ERTEX F ORM OF A Q UADRATIC E QUATION
3.3 I NVERSE OF Q UADRATIC F UNCTIONS
3.4 S IMPLIFYING S QUARE R OOTS
3.5 M ETHODS FOR S OLVING Q UADRATIC F UNCTIONS
3.6 FAMILIES OF Q UADRATIC F UNCTIONS
3.7 T HE D ISCRIMINANT
3.8 L INEAR -Q UADRATIC S YSTEMS
3.9 R EFERENCES

122
www.ck12.org Chapter 3. Quadratic Functions

3.1 Properties of Quadratic Functions

Learning Objectives

Here you will explore different methods of graphing quadratic functions. You have likely been exposed to one or
all of these methods in the past, so pay particular attention to any that remain confusing or difficult. Being skilled
at graphing quadratic functions can save a lot of time on more complex problems, particularly when only a good
approximate answer is needed.
In prior lessons, we have discussed the standard form of a quadratic function: f (x) = ax2 + bx + c. You may have
seen other forms, such as the vertex form, or the factored form. Why are there so many common ways to write the
same equations? Why should we learn all these different forms if the standard form is the most common anyway?

Graphing Quadratic Functions

Quadratic Functions

A function f defined by f (x) = ax2 + bx + c, where a, b, and c are real numbers and a 6= 0, is called a quadratic
function.
The defining characteristic of a quadratic function is that it is a polynomial whose highest exponent is 2.
There are several ways to write quadratic functions:

• standard form, the form of the quadratic function above: f (x) = ax2 + bx + c
• vertex form, commonly used for quick sketching: f (x) = a(x − h)2 + k
• factored form, excellent for finding x-intercepts: f (x) = a(x − r1 )(x − r2 )

The y−intercept of a quadratic function in standard form is (0, c) and it is found by substituting 0 for x in f (x) =
ax2 + bx + c.

Summary of Vertex Form

Given a quadratic function in vertex form: f (x) = a(x − h)2 + k:

• The vertex is at (h, k)


• The parabola opens up if a > 0
• The parabola opens down if a < 0
• The parabola is narrower than y = x2 if |a|> 1
• The parabola opens wider than y = x2 if |a|< 1

Vertex of a Parabola

In the standard form of a quadratic function, the x−coordinate of the vertex of the parabola is given by the equation:

b
x=−
2a

123
3.1. Properties of Quadratic Functions www.ck12.org

The y−coordinate of the vertex is found with:

 
b
y= f −
2a

Axis of Symmetry of a Parabola

A parabola has reflective symmetry about a vertical line through the vertex.
b
The vertical line x = − 2a is also the parabola’s axis of symmetry.

MEDIA
Click image to the left or use the URL below.
URL: http://www.ck12.org/flx/render/embeddedobject/187257

Examples

Example 1

Graph g(x) = x2 + 6x + 7 using transformations.


b 2

First, complete the square to write this function in vertex form. Add and subtract 2 to the right hand side of the
equation:

g(x) = x2 + 6x + 7
= x2 + 6x + 9 + 7 − 9

Now, factor the right hand side:

g(x) = (x + 3)2 − 2

Thus, a = 1 and the vertex of this parabola is (-3, -2). We know that the parabola opens up with the same width as
y = x2 and it has a minimum value at the vertex. The graph of the parabola is below.

124
www.ck12.org Chapter 3. Quadratic Functions

Example 2

Find the vertex and graph the quadratic function g(x) = x2 − 8x + 12.
The x−coordinate of the vertex is x = − −8
2 = 4.
The y−coordinate of the vertex is g(4) = (4)2 − 8(4) + 12 = 16 − 32 + 12 = −4
Thus the vertex is at (4, -4).
To graph the parabola, we will make a table of points staring with the x−coordinate of 4:

TABLE 3.1:
x y = g(x)
4 -4
5 -3
6 0
7 5

125
3.1. Properties of Quadratic Functions www.ck12.org

Now we can use the symmetry of g(x) to fill in this table for g(3). Note that g(3) = g(5) = −3. Likewise, g(2) =
g(6) = 0. The final graph is below.

Example 3

Sketch the graph of the function y = f (x) = x2 + 2x − 3.


Let’s first find the intercepts. For the y−intercept, if x = 0, then f (0) = −3, or y = −3, so the y−intercept point is
(0, -3).
Now, for the x−intercepts, if y = f (x) = 0, then x2 + 2x − 3 = 0, or

126
www.ck12.org Chapter 3. Quadratic Functions

x2 + 2x − 3 = (x + 3)(x − 1) = 0

so that x = −3 and x = 1 are the x−intercepts, that is (-3, 0) and (1, 0).
The vertex (extreme point) is at

−b −2
x= = = −1
2a 2(1)

Since

f (−1) = (−1)2 + 2(−1) − 3


= −4

The vertex is (-1, -4).


Since the coefficient of x2 is positive, a > 0, the extreme point is a minimum and the parabola opens up. From this
information, we can make a rough sketch of the parabola containing the points determined above. Notice that the
range of the function is y ≥ −4.

Example 4

Sketch the graph of the quadratic function f (x) = −x2 + 4x.


To find the y−intercept, set x = 0, and f (0) = −(0)2 + 4(0) = 0
Thus the parabola intercepts the y−axis at the origin.

127
3.1. Properties of Quadratic Functions www.ck12.org

The x−intercept is obtained by setting y = 0. Thus, −x2 + 4x = 0.


Factoring,

−x2 + 4x = −x(x − 4) = 0

so that x = 0 and x = 4 are the x−intercepts.


We have a = −1 and b = 4, so that the extreme point occurs when

−b −4
x= = =2
2a 2(−1)

Since f (2) = −(2)2 +4(2) = −4+8 = 4, then (2, 4) is the extreme point. It is a maximum point since a = −1 < 0 and
the parabola opens down. Finally, the graph can be obtained by sketching a parabola through the points determined
above. From graph, the range of the function is y ≤ 4.

Example 5

Sketch the graph of y = −3(x − 2)2 + 1.


The equation y = −3(x − 2)2 + 1 is already in vertex form, so graphing is relatively easy:
Recall the when the equation is written in vertex form like this, the vertex is the point (h,k):
vertex form: y = a(x − h)2 + k
Our equation: y = −3(x − 2)2 + 1
Examining our equation, we can see the vertex of the parabola is at (2, 1).
To find another point on the parabola solve for an x value.
Since the vertex is at x = 2, let’s try one unit to the right: x = 3.
−3(3 − 2)2 + 1 = −2
∴ There is a point on the parabola at (3, -2)

128
www.ck12.org Chapter 3. Quadratic Functions

Since a parabola has an axis of symmetry that passes through its vertex, we can reflect the point (3, -2) across the
axis of symmetry to get another point, (1, -2) also on the parabola.
The graph of y = −3(x − 2)2 + 1 is shown below, using vertex (2, 1) and points (3, -2) and (1, -2).

MEDIA
Click image to the left or use the URL below.
URL: http://www.ck12.org/flx/render/embeddedobject/187259

129
3.2. Vertex Form of a Quadratic Equation www.ck12.org

3.2 Vertex Form of a Quadratic Equation

Learning Objectives

Here you’ll learn how to use the vertex form of a quadratic equation in order to find the vertex by completing the
square.
Suppose a diver jumped into the ocean, and his path could be traced by the parabola y = x2 − 4x + 2, with the value
of y representing the diver’s distance above or below the surface of the water in feet. How far below the surface of
the water would the diver descend?

Vertex Form of a Quadratic Equation

There are several ways to write an equation for a parabola:

• Standard form: y = ax2 + bx + c


• Factored form: y = (x + m)(x + n)
• Vertex form: y = a(x − h)2 + k

So far, you have used both standard form and factored from. Now, we will use vertex form. The vertex form of a
quadratic equation is y = a(x − h)2 + k, where (h, k) = vertex of the parabola and a = leading coefficient

Let’s determine the vertex of y = − 12 (x − 4)2 − 7 and state if it is a minimum or maximum:

Using the definition of vertex form, h = 4 and k = −7.

• The vertex is (4, -7).


• Because a is negative, the parabola opens downward.
• Therefore, the vertex (4, -7) is a maximum point of the parabola.

Once you know the vertex, you can use symmetry to graph the parabola.

TABLE 3.2:
x y
2 -9
3 -7.5
4 -7
5 -7.5
6 -9

Now, let’s write the equation for a parabola with a = 3 and vertex (-4, 5) in vertex form:

Using the definition of vertex form, y = a(x − h)2 + k, h = −4 and k = 5.

130
www.ck12.org Chapter 3. Quadratic Functions

y = 3(x − (−4))2 + 5
y = 3(x + 4)2 + 5

Finding the Vertex by Completing the Square

Consider the quadratic equation y = x2 + 4x − 2. What is its vertex? You could graph this using your calculator and
determine the vertex or you could complete the square.
Start by completing the square. Since 21 b = 21 (4) = 2, 22 = 4.

y = x2 + 4x − 2
Add 2 to each side y + 2 = x2 + 4x
Add 4 to each side y + 2 + 4 = x2 + 4x + 4
Factor the perfect square trinomial y + 6 = (x + 2)2
Subtract 6 from each side to get into vertex form y = (x + 2)2 − 6

The vertex is (−2, −6).

MEDIA
Click image to the left or use the URL below.
URL: http://www.ck12.org/flx/render/embeddedobject/180236

MEDIA
Click image to the left or use the URL below.
URL: http://www.ck12.org/flx/render/embeddedobject/180238

Examples

Example 1

Earlier, you were told that a diver who jumps into the ocean follows the path of the parabola y = x2 − 4x + 2, with
the value of y representing the diver’s distance above or below the surface of the water in feet. How far below the

131
3.2. Vertex Form of a Quadratic Equation www.ck12.org

surface of the water would the diver descend?


To find how far below the surface the diver can go, we need to find the minimum for the parabola.
Start by completing the square. Since 12 b = 21 (−4) = −2, (−2)2 = 4.

y = x2 − 4x + 2
Subtract 2 from each side y − 2 = x2 − 4x
Add 4 to each side y − 2 + 4 = x2 − 4x + 4
Factor the perfect square trinomial y + 2 = (x − 2)2
Subtract 2 from each side to get into vertex form y = (x − 2)2 − 2

The vertex is (2, −2).


The minimum value that the parabola reaches is the y-value of the vertex, -2. Since we want to know how far below
the surface the diver goes, we use the absolute value of the minimum value. The diver can go 2 feet below the surface
of the water.

Example 2

An arrow is shot straight up from a height of 2 meters with a velocity of 50 m/s. What is the maximum height that
the arrow will reach and at what time will that happen?
The maximum height is the vertex of the parabola, when the parabola faces down. Therefore, we need to rewrite the
equation in vertex form.

We rewrite the equation in vertex form. y = −4.9t 2 + 50t + 2


y − 2 = −4.9t 2 + 50t
y − 2 = −4.9(t 2 − 10.2t)
Complete the square inside the parentheses. y − 2 − 4.9(5.1)2 = −4.9(t 2 − 10.2t + (5.1)2 )
Add 129.45 to get into vertex form. y − 129.45 = −4.9(t − 5.1)2
y = −4.9(t − 5.1)2 + 129.45

Since the y-value of the vertex is 129.45, then the maximum height is 129.45 meters.

132
www.ck12.org Chapter 3. Quadratic Functions

3.3 Inverse of Quadratic Functions

133
3.3. Inverse of Quadratic Functions www.ck12.org

134
www.ck12.org Chapter 3. Quadratic Functions

135
3.3. Inverse of Quadratic Functions www.ck12.org

136
www.ck12.org Chapter 3. Quadratic Functions

137
3.4. Simplifying Square Roots www.ck12.org

3.4 Simplifying Square Roots

Learning Objectives

Here you’ll simplify, add, subtract and multiply square roots.


√ √
The length of the two legs of a right triangle are 2 5 and 3 4. What is the length of the triangle’s hypotenuse?

Simplifying Square Roots

Before we can solve a quadratic equation using square roots, we need to review how to simplify, add, subtract, and
multiply them. Recall that the square root is a number that, when multiplied by itself, produces another number. 4
is the square root of 16, for example. -4 is also the square root of 16 because (−4)2 = 16. The symbol for square

root is the radical sign, or . The number under the radical is called the radicand. If the square root of an integer
is not another integer, it is an irrational number.

Let’s find 50.

a. using a calculator.

To plug the square root into your graphing calculator, typically there is a or SQRT button. Depending on your

model, you may have to enter 50 before or after the square root button. Either way, your answer should be 50 =
7.071067811865 . . . In general, we will round to the hundredths place, so 7.07 is sufficient.

b. by simplifying the square root.

To simplify the square root, the square numbers must be “pulled out.” Look for factors of 50 that are square numbers:
4, 9, 16, 25... 25 is a factor of 50, so break the factors apart.
√ √ √ √ √
50 = 25 · 2 = 25 · 2 = 5 2. This is the most accurate answer.
Radical Rules
√ √ √ √
1. ab = a · b = a · b Any two radicals can be multiplied together.
√ √ √
2. x a ± y a = x ± y a The radicands must be the same in order to add or subtract.
√ 2 √ 2
3. a = a = a The square and square root cancel each other out.
Now, let’s simplify the following expressions.
√ √ √
1. 45 + 80 − 2 5

At first glance, it does not look like we can simplify this. But, we can simplify each radical by pulling out the perfect
squares.

√ √ √
45 = 9 · 5 = 3 5
√ √ √
80 = 16 · 5 = 4 5
√ √ √
Rewriting our expression, we
√ have: 3 5 + 4 5 − 2 5 and all the radicands are the same. Using the Order of
Operations, our answer is 5 5.

138
www.ck12.org Chapter 3. Quadratic Functions
√ √
2. 2 35 · 4 7

Multiply across.

√ √ √ √
2 35 · 4 7 = 2 · 4 35 · 7 = 8 245

√ √ √ √
Now, simplify the radical. 8 245 = 8 49 · 5 = 8 · 7 5 = 56 5

Examples

Example 1

Earlier, you were asked to find the length of the triangle’s hypotenuse.
We must use the Pythagorean Theorem, which states that the square of one leg of a right triangle plus the square of
the other leg equals the square of the hypotenuse.
√ √
So we are looking for c such that (2 5)2 + (3 4)2 = c2 .
Simplifying, we get 4 · 5 + 9 · 4 = c2 , or 20 + 36 = c2 .
Therefore, c2 = 56, so to find c, we must take the square root of 56.
√ √ √
56 = 4 · 14 = 2 14.

Therefore, c = 2 14.
Simplify the following radicals.

Example 2

150
Pull out all the square numbers.

√ √ √
150 = 25 · 6 = 5 6

Alternate Method: Write out the prime factorization of 150.

√ √
150 = 2 · 3 · 5 · 5

Now, pull out any number that has a pair. Write it once in front of the radical and multiply together what is left over
under the radical.

√ √ √
150 = 2 · 3 · 5 · 5 = 5 6

Example 3
√ √ √
2 3 − 6 + 96

139
3.4. Simplifying Square Roots www.ck12.org

Simplify 96 to see if anything can be combined. We will use the alternate method above.

√ √ √ √
96 = 2 · 2 · 2 · 2 · 2 · 3 = 2 · 2 6 = 4 6

√ √ √ √ √ √ √
Rewrite the expression: 2 3− 6+4 6 = 2 3+3 6. This is fully simplified. 3 and 6 cannot be combined
because they do not have the same value under the radical.

Example 4
√ √
8 · 20
This problem can be done two different ways.
First Method: Multiply radicals, then simplify the answer.

√ √ √ √ √
8 · 20 = 160 = 16 · 10 = 4 10

Second Method: Simplify radicals, then multiply.

√ √ √  √  √ √ √ √
8 · 20 = 4·2 · 4 · 5 = 2 2 · 2 5 = 2 · 2 2 · 5 = 4 10

Depending on the complexity of the problem, either method will work. Pick whichever method you prefer.

140
www.ck12.org Chapter 3. Quadratic Functions

3.5 Methods for Solving Quadratic Functions

Learning Objectives

Here you will review multiple methods of solving quadratic functions for the x intercepts. This is a far-reaching
lesson with concepts such as: applying the quadratic formula, completing the square, and solving by factoring, that
you will be returning to time and again as you progress further into your mathematics studies.
Can you measure height with a stopwatch? How could that be possible?
This lesson is all about quadratic functions, like the one used in Physics and described later in the lesson, which
allows a good approximation of the height of a tall object to be calculated with time!

Methods for Solving Quadratic Functions

A quadratic function can be described as:


Formally: A function f defined by f (x) = ax2 + bx + c, where a, b, and c are real numbers and a 6= 0.
Informally: The defining characteristic of a quadratic function is that it is a polynomial whose highest exponent is 2.
There are several ways to write quadratic functions:

• standard form, the form of the quadratic function above: f (x) = ax2 + bx + c
• vertex form, commonly used for quick sketching: f (x) = a(x − h)2 + k
• factored form, excellent for finding x-intercepts: f (x) = a(x − r1 )(x − r2 )

You can convert between forms of quadratic functions using algebra and you will see that there are uses for each of
these forms when working with quadratic functions.
When graphing, the y−intercept of a quadratic function in standard form is (0, c) and it is found by substituting 0
for x in f (x) = ax2 + bx + c.

MEDIA
Click image to the left or use the URL below.
URL: http://www.ck12.org/flx/render/embeddedobject/187255

MEDIA
Click image to the left or use the URL below.
URL: http://www.ck12.org/flx/render/embeddedobject/187256

141
3.5. Methods for Solving Quadratic Functions www.ck12.org

Examples

Example 1

Earlier, you were asked if you could measure height with a stopwatch.
What formula could be used to calculate the height of an object based on the time required for an object to fall from
the top?

One formula is h = 12 gt 2

This is not completely accurate, as it ignores the effects of air resistance of an object, but offers a very close
approximate for most common objects.
The value g is gravity. In Metric units this is 9.81m/s/s
The value t is the time taken for the object to fall in seconds
The result h is the distance or the height in meters.

Example 2

Solve x2 − 5x + 6 = 0 using the "Factor to Solve" method.


The factor method is based on writing the quadratic equation in factored form. That is, as a product of two linear
expressions. So our equation maybe solved by the following way:

x2 − 5x + 6 = 0
(x − 3)(x − 2) = 0

Recall, that

a · b = 0 if and only if a = 0 and b = 0

This tells us that

x−3 = 0

or

x−2 = 0

which give the roots (or zeros)

x=3

and

142
www.ck12.org Chapter 3. Quadratic Functions

x=2

In other words, the solution set is {2, 3}.

Example 3

Solve x2 − 5x + 6 = 0 by the "Completing the Square" method.


To solve the above equation by completing the square, first move the “c” term to the other side of the equation,

x2 − 5x = −6

Next, make the left-hand side a “perfect square” by adding the appropriate number. To do so, take one-half of the
coefficient of x (the b coefficient) and square it and then add the result to both sides of the equation:

b = −5
b −5
=
2 2
 2
b 25
=
2 4

Adding to both sides of the equation,

25 25
x2 − 5x + = −6 +
4 4
2 25 1
x − 5x + =
4 4
 2
5 1
x− =
2 4

this last equation can be easily solved by taking the square root of both sides,

  r
5 1 1
x− = =±
2 4 2

Hence

1 5
x=± +
2 2

and the solutions are

x=3

143
3.5. Methods for Solving Quadratic Functions www.ck12.org

and

x=2

which are identical to answers of the factor method.

Example 4

Solve x2 − 5x + 6 = 0 using the Quadratic Formula.


The quadratic formula works for finding the x-intercepts in all quadratic equations, therefore it is highly encouraged
that you memorize the formula. The other methods can be much faster though, so it is well worth understanding
each of the different methods.
Recall, if ax2 + bx + c = 0 where a, b, and c are real numbers and a 6= 0, then the roots of the equation can be
determined by the quadratic formula.


−b ± b2 − 4ac
x= .
2a
Here the coefficients are a = 1, b = −5, and c = 6.
Substituting into the quadratic formula, we get:

p
−(−5) ± (−5)2 − 4(1)(6)
x=
2(1)

5 ± 25 − 24
=
2
5±1
=
2
= 3 or 2

which is, again, identical to our two solutions above.

Example 5

Solve the following equation by factoring: 4x2 + 7x = 2.


4x2 + 7x − 2 = 0
0 = (4x − 1)(x + 2) ∴ x = 41 , −2

Example 6

Solve the following equation by completing the square: 23 x2 − x + 13 = 0.


x2 − 32 x + 21 = 0
x2 − 32 x + 16
9
= 9
16 − 21
(x − 34 )(x − 43 ) = 1
16
x = 21 , 1

144
www.ck12.org Chapter 3. Quadratic Functions

Example 7

Solve the following equation by using the quadratic formula: (z + 6)2 + 2z = 0.



2 −14± 142 − 4 · 1 · 36
z + 14z + 36 = 0 ∴ x = 2·1

x = −7 ± 13

145
3.6. Families of Quadratic Functions www.ck12.org

3.6 Families of Quadratic Functions

146
www.ck12.org Chapter 3. Quadratic Functions

147
3.6. Families of Quadratic Functions www.ck12.org

148
www.ck12.org Chapter 3. Quadratic Functions

149
3.6. Families of Quadratic Functions www.ck12.org

150
www.ck12.org Chapter 3. Quadratic Functions

3.7 The Discriminant

Learning Objectives

Here you’ll learn what the discriminant is and how to use it to help you to describe the roots and graph of a quadratic
function.
Suppose you want to know whether the function y = x2 − 5x + 12 has real roots. What part of the quadratic formula
would you need to test in order to determine if the roots of the function are real or complex?

The Discriminant

Consider the potential roots of a quadratic function. There are three options:

1. The function has 2 distinct real roots and 2 x-intercepts.

2. The function has 1 real root (of multiplicity 2) and 1 x-intercept.

3. The function has 0 real roots, 2 complex roots, and 0 x-intercepts.

151
3.7. The Discriminant www.ck12.org

The quadratic formula states that the two roots of a quadratic function y = ax2 + bx + c are:
√ √
−b+ b2 − 4ac −b− b2 − 4ac
r1 = 2a and r2 = 2a
Under the radical sign is the expression b2 − 4ac. This expression is known as the discriminant of the quadratic
function and is important especially because when it is negative the roots of the function are complex. There are
only three possible outcomes for the value of the discriminant. These outcomes are:

1. The value of the discriminant is positive (b2 − 4ac > 0). This means the function has 2 distinct real roots and
2 x-intercepts.
2. The value of the discriminant is zero (b2 − 4ac = 0). This means the function has 1 real root (of multiplicity
2) and 1 x-intercept.
3. The value of the discriminant is negative (b2 − 4ac < 0). This means the function has 0 real roots, 2 complex
roots, and 0 x-intercepts.

This discriminant is helpful when you are only looking to describe the graph of a quadratic function or its roots,
but don’t need to know its exact roots. If you need to know its exact roots, you will still have to use the complete
quadratic formula.

For the following problems, if the discriminant of a quadratic function has the value indicated, let’s
determine if the function will have two distinct real roots, 1 real root of multiplicity 2, or two distinct
complex roots:
1. 7

b2 − 4ac > 0 so the quadratic function will have two distinct real roots.

2. -3

b2 − 4ac < 0 so the quadratic function will have two distinct complex roots.

1
3. 2

b2 − 4ac > 0 so the quadratic function will have two distinct real roots.

4. 0

152
www.ck12.org Chapter 3. Quadratic Functions

b2 − 4ac = 0 so the quadratic function will have 1 real root of multiplicity 2.


Now, for the following problems, if the discriminant of a quadratic function has the value indicated, let’s determine
how many times will the graph intersect the x-axis:

1. 5

b2 − 4ac > 0 so the parabola will cross the x-axis twice.

2. -6

b2 − 4ac < 0 so the parabola will not cross the x-axis.

3. 0

b2 − 4ac = 0 so the parabola will cross the x-axis once.

4. 0.2

b2 − 4ac > 0 so the parabola will cross the x-axis twice.

Finally, for each of the following quadratic equations, let’s determine the value of the discriminant
and use that value to describe the nature of the roots:

Let D represent the discriminant.

1. 4x2 − 4x + 1 = 0

The quadratic equation will have 1 real solution of multiplicity 2.

D = b2 − 4ac
D = (−4)2 − 4(4)(1)
D = 16 − 16
D=0
b2 − 4ac = 0

2. x2 + 3x + 9 = 0

The quadratic equation will have two distinct complex solutions and no real solutions.

D = b2 − 4ac
D = (3)2 − 4(1)(9)
D = 9 − 36
D = −27
b2 − 4ac < 0

153
3.7. The Discriminant www.ck12.org

3. 2x2 + 3x − 4 = 0

The quadratic equation will have two distinct real solutions.

D = b2 − 4ac
D = (3)2 − 4(2)(−4)
D = 9 + 32
D = 41
b2 − 4ac > 0

4. 9x2 + 12x + 4 = 0

The quadratic equation will have 1 real root of multiplicity 2.

D = b2 − 4ac
D = (12)2 − 4(9)(4)
D = 144 − 144
D=0
b2 − 4ac = 0

Examples

Example 1

Earlier, you were asked what part of the quadratic formula would you need to test in order to determine if the roots
of the function are real or complex.
In order to determine if the roots of y = x2 − 5x + 12 are real or complex, you must find the discriminant, b2 − 4ac.
For this function:

b2 − 4ac = (−5)2 − 4(1)(12)


= 25 − 48
= −23

Because the discriminant is negative, this function has two distinct complex roots.

Example 2

Given the following quadratic equation, find the value of ‘m’ such that the equation will have 1 real solution of
multiplicity 2.
mx2 + (m + 8)x + 9 = 0
Begin by determining the value of the discriminant.

154
www.ck12.org Chapter 3. Quadratic Functions

b2 − 4ac = (m + 8)2 − 4(m)(9)

Expand and Simplify

b2 − 4ac = (m + 8)(m + 8) − 4(m)(9)


b2 − 4ac = m2 + 8m + 8m + 64 − 36m
b2 − 4ac = m2 − 20m + 64

If the equation has 1 real solution of multiplicity 2, the value of the discriminant must equal zero.

b2 − 4ac = 0
∴ m2 − 20m + 64 = 0

Factor the quadratic equation and solve for the variable ‘m’.

(m − 16)(m − 4) = 0
m − 16 = 0 or m − 4 = 0
m = 16 or m = 4

The values of ‘m’ that would produce 1 solution of multiplicity 2 for the quadratic equation are m = 16 or m = 4.

Example 3

Given the following quadratic equation, determine the nature of the solutions:
4(y2 − 5y + 5) = −5
Write the quadratic equation in general form.

4(y2 − 5y + 5) = −5

Apply the distributive property.

4y2 − 20y + 20 = −5

Set the equation equal to zero.

4y2 − 20y + 20+5 = −5+5


4y2 − 20y + 25 = 0

Determine the value of the discriminant for this quadratic equation.

155
3.7. The Discriminant www.ck12.org

D = b2 − 4ac
D = (−20)2 − 4(4)(25)
D = 400 − 400
D=0
b2 − 4ac = 0

The quadratic equation will have 1 real solution of multiplicity 2.

Example 4

Given the following quadratic equation, find the value of ‘m’ such that the equation will have two distinct complex
solutions.
(m + 1)e2 − 2e − 3 = 0
Begin by determining the value of the discriminant.

b2 − 4ac = (−2)2 − 4(m + 1)(−3)

Expand and Simplify

b2 − 4ac = 12m + 16

If the equation has two distinct complex solutions the value of the discriminant must be less than zero.

b2 − 4ac < 0
∴ 12m + 16 < 0

Solve the inequality.

12m + 16−16 < 0−16


12m

The value of ‘m’ that would produce two distinct complex solutions for the quadratic equation is m.

156
www.ck12.org Chapter 3. Quadratic Functions

3.8 Linear-Quadratic Systems

157
3.8. Linear-Quadratic Systems www.ck12.org

158
www.ck12.org Chapter 3. Quadratic Functions

159
3.8. Linear-Quadratic Systems www.ck12.org

160
www.ck12.org Chapter 3. Quadratic Functions

3.9 References

1. Nelson. Nelson . CC-BY-SA


2. Nelson. Nelson . CC-BY-SA
3. Nelson. Nelson . CC-BY-SA
4. Nelson. Nelson . CC-BY-SA
5. Nelson. Nelson . CC-BY-SA
6. Nelson. Nelson . CC-BY-SA
7. Nelson. Nelson . CC-BY-SA
8. Nelson. Nelson . CC-BY-SA
9. Nelson. Nelson . CC-BY-SA
10. Nelson. Nelson . CC-BY-SA
11. Nelson. Nelson . CC-BY-SA
12. Nelson. Nelson . CC-BY-SA
13. Nelson. Nelson . CC-BY-SA
14. Nelson. Nelson . CC-BY-SA
15. Nelson. Nelson . CC-BY-SA
16. Nelson. Nelson . CC-BY-SA

161
www.ck12.org

C HAPTER
4 Exponential Functions
Chapter Outline
4.1 P RODUCT R ULES FOR E XPONENTS
4.2 Q UOTIENT R ULES FOR E XPONENTS
4.3 P OWER R ULE FOR E XPONENTS
4.4 Z ERO AND N EGATIVE E XPONENTS
4.5 F RACTIONAL E XPONENTS
4.6 E XPONENTIAL E XPRESSIONS
4.7 G RAPHS OF E XPONENTIAL F UNCTIONS
4.8 G RAPHING T RANSFORMED E XPONENTIAL F UNCTIONS
4.9 A PPLICATIONS OF E XPONENTIAL F UNCTIONS
4.10 R EFERENCES

162
www.ck12.org Chapter 4. Exponential Functions

4.1 Product Rules for Exponents

Learning Objectives

Here you’ll learn how to multiply two terms with the same base and how to find the power of a product.
Suppose you have the expression:
x·x·x·x·x·x·x·x·x·y·y·y·y·y·x·x·x·x
How could you write this expression in a more concise way?

Product Rules for Exponents

In the expression x3 , the x is called the base and the 3 is called the exponent. Exponents are often referred to as
powers. When an exponent is a positive whole number, it tells you how many times to multiply the base by itself.
For example:

• x3 = x · x · x
• 24 = 2 · 2 · 2 · 2 = 16.

There are many rules that have to do with exponents (often called the Laws of Exponents) that are helpful to know
so that you can work with expressions and equations that involve exponents more easily. Here you will learn two
rules that have to do with exponents and products.

1. To multiply two terms with the same base, add the exponents.

am × an = (a × a × . . . × a) (a × a × . . . × a)
←−−−−−−−−−→ ←−−−−−−−−−→
↓ ↓
m factors n factors
m n
a × a = (a × a × a . . . × a)
←−−−−−−−−−−→

m + n factors
m n m+n
a ×a = a

2. To raise a product to a power, raise each of the factors to the power.

163
4.1. Product Rules for Exponents www.ck12.org

(ab)n = (ab) × (ab) × . . . × (ab)


←−−−−−−−−−−−−−−→

n factors
n
(ab) = (a × a × . . . × a) × (b × b × . . . × b)
←−−−−−−−−−→ ←−−−−−−−−−→
↓ ↓
n factors n factors
n n n
(ab) = a b

Let’s simplify the following expressions:


1. 32 × 33

32 × 33 The base is 3.
2+3
3 Keep the base of 3 and add the exponents.
5
3 This answer is in exponential form.

The answer can be taken one step further. The base is numerical so the term can be evaluated.

35 = 3 × 3 × 3 × 3 × 3
35 = 243
32 × 33 = 35 = 243

2. (x3 )(x6 ).

(x3 )(x6 ) The base is x.


x3+6 Keep the base of x and add the exponents.
9
x The answer is in exponential form.
(x3 )(x6 ) = x9

3. y5 · y2 .

y5 · y2 The base is y.
y5+2 Keep the base of y and add the exponents.
7
y The answer is in exponential form.
y5 · y2 = y7

164
www.ck12.org Chapter 4. Exponential Functions

4. 5x2 y3 · 3xy2 .

5x2 y3 · 3xy2 The bases are x and y.


2 3 2
15(x y )(xy ) Multiply the coefficients - 5 × 3 = 15. Keep the base of x and y and add
the exponents of the same base. If a base does not have a written
exponent, it is understood as 1.
15x2+1 y3+2
15x3 y5 The answer is in exponential form.
5x2 y3 · 3xy2 = 15x3 y5

Examples

Example 1

Earlier, you were asked to write the following expression in a more concise way;
x·x·x·x·x·x·x·x·x·y·y·y·y·y·x·x·x·x
The expression can be rewritten as x9 y5 x4 . Then, you can use the rules of exponents to simplify the expression to
x13 y5 . This is certainly much quicker to write!

Example 2

Simplify the following expression:


(−3x)2
9x2 . Here are the steps:

(−3x)2 = (−3)2 · (x)2


= 9x2

Example 3

Simplify the following expression:


(5xy)3
125x3 y3 . Here are the steps:

(5x2 y4 )3 = (5)3 · (x)3 · (y)3


= 125x3 y3

Example 4

Simplify the following expression:

165
4.1. Product Rules for Exponents www.ck12.org

(23 · 32 )2
Here are the steps:

(23 · 32 )2 = (8 · 9)2
= (72)2
= 5184

OR

(23 · 32 )2 = (8 · 9)2
= 82 · 92
= 64 · 81
= 5184

166
www.ck12.org Chapter 4. Exponential Functions

4.2 Quotient Rules for Exponents

Learning Objectives

Here you’ll learn how to divide two terms with the same base and find the power of a quotient.
Suppose you have the expression:
x·x·x·x·x·x·x·x·x·y·y·y·y·y
x·x·x·x·x·x·y·y·y
How could you write this expression in a more concise way?

Quotient Rules for Exponents

Recall that in the expression x3 , the x is called the base and the 3 is called the exponent. Exponents are often referred
to as powers. When an exponent is a positive whole number, it tells you how many times to multiply the base by
itself. For example:

• x3 = x · x · x

There are many rules that have to do with exponents (often called the Laws of Exponents) that are helpful to know
so that you can work with expressions and equations that involve exponents more easily. Here you will learn two
rules that have to do with exponents and quotients.

1. To divide two powers with the same base, subtract the exponents.

m factors

← −−− −−−−−−→
am (a × a × . . . × a)
= m > n; a 6= 0
an (a × a × . . . × a)
←−−−−−−−−−→

n factors
am
= (a × a × . . . × a)
an ←−−−−−−−−−→

m − n factors
am
= am−n
an

2. To raise a quotient to a power, raise both the numerator and the denominator to the power.

167
4.2. Quotient Rules for Exponents www.ck12.org

 a n a a a
= × ×...×
b b b b
←−−−−−−−−−→

n factors
n factors

 a n (a × a × . .−.−
← −−− −−− −→
× a)
=
b (b × b × . . . × b)
←−−−−−−−−−→

n factors
 a n an
= (b 6= 0)
b bn

Let’s simplify the following expressions:


1. 27 ÷ 23

27 ÷ 23 The base is 2.
27−3 Keep the base of 2 and subtract the exponents.
4
2 The answer is in exponential form.

The answer can be taken one step further. The base is numerical so the term can be evaluated.

24 = 2 × 2 × 2 × 2
24 = 16
27 ÷ 23 = 24 = 16

x8
2. x2

x8
The base is x.
x2
x8−2 Keep the base of x and subtract the exponents.
6
x The answer is in exponential form.
x8
= x6
x2

16x5 y5
3. 4x2 y3

168
www.ck12.org Chapter 4. Exponential Functions

16x5 y5
The bases are x and y.
4x2 y3
 5 5
x y
4 2 3 Divide the coefficients - 16 ÷ 4 = 4. Keep the base of x and y and
x y
subtract the exponents of the same base.
5−2 5−3
4x y
4x3 y2

Examples

Example 1

Earlier, you were asked to simplify the following expression:


x·x·x·x·x·x·x·x·x·y·y·y·y·y
x·x·x·x·x·x·y·y·y
x 9 y5
This expression can be rewritten as x 6 y3
and then simplified to x3 y2 .

Example 2

Simplify the following expression:


2 2

3
2 2 22 4

3 = 32
= 9

Example 3

Simplify the following expression:


x 3

6
x 3 x3 x3

6 = 63
= 216

Example 4

Simplify the following expression:


 2
3x
4y
 2
3x 32 x2 9x2
4y = 4 2 y2
= 16y2

169
4.3. Power Rule for Exponents www.ck12.org

4.3 Power Rule for Exponents

Learning Objectives

Here you’ll learn how to find the power of a power.


Can you simplify an expression where an exponent has an exponent? For example, how would you simplify [(23 )2 ]4 ?

Power Rule for Exponents

Recall that in the expression x3 , the x is called the base and the 3 is called the exponent. Exponents are often referred
to as powers. When an exponent is a positive whole number, it tells you how many times to multiply the base by
itself. For example:

• x3 = x · x · x

There are many rules that have to do with exponents (often called the Laws of Exponents) that are helpful to know
so that you can work with expressions and equations that involve exponents more easily. Here you will learn a rule
that has to do with raising a power to another power.

1. To raise a power to a new power, multiply the exponents.

(am )n = (a × a × . . . × a)n
←−−−−−−−−−−→

m factors
m n
(a ) = (a × a × . . . × a) × (a × a × . . . × a) (a × a × . . . × a)
←−−−−−−−−−→ ←−−−−−−−−−→ ←−−−−−−−−−→
↓ ↓ ↓
m factors m factors m factors
←−−−−−−−−−−−−−−−−−−−−−−−−−−−−−−−−−−→
n times
(am )n = a × a × a . . . × a
←−−−−−−−−−→
mn factors
(am )n = amn

Let’s simplify the following expressions:


1. (23 )2

(23 )2 = 26 = 64

2. (x7 )4

170
www.ck12.org Chapter 4. Exponential Functions

(x7 )4 = x28

3. (32 )3

(32 )3 = 36 = 729

4. (x2 y4 )2 · (xy4 )3

(x2 y4 )2 · (xy4 )3 = x4 y8 · x3 y12 = x7 y20

Examples

Example 1

Earlier, you were asked if you can simplify an expression where an exponent has an exponent like [(23 )2 ]4 .
[(23 )2 ]4 = [26 ]4 = 224 . Notice that the power rule applies even when a number has been raised to more than one
power. The overall exponent is 24 which is 3 · 2 · 4.

You know you can rewrite 24 as 2 × 2 × 2 × 2 and then calculate in order to find that

24 = 16

This concept can also be reversed. To write 32 as a power of 2, 32 = 2 × 2 × 2 × 2 × 2. There are 5 twos; therefore,

32 = 25

Use this idea to complete the following problems.

Example 2

Write 81 as a power of 3.
81 = 3 × 3 = 9 × 3 = 27 × 3 = 81

There are 4 threes. Therefore

81 = 34

Example 3

Write (9)3 as a power of 3.


9 = 3×3 = 9

There are 2 threes. Therefore

9 = 32

171
4.3. Power Rule for Exponents www.ck12.org

(32 )3 Apply the law of exponents for power to a power-multiply the exponents.
32×3 = 36
Therefore

(9)3 = 36

Example 4

Write (43 )2 as a power of 2.


4 = 2×2 = 4

There are 2 twos. Therefore

4 = 22
2
(22 )3 Apply the law of exponents for power to a power-multiply the exponents.

22×3 = 26

(26 )2 Apply the law of exponents for power to a power-multiply the exponents.

26×2 = 212

Therefore

(43 )2 = 212

172
www.ck12.org Chapter 4. Exponential Functions

4.4 Zero and Negative Exponents

Learning Objectives

Here you’ll learn how to work with zero and negative exponents.
How can you use the quotient rules for exponents to understand the meaning of a zero or negative exponent?

Watch This

Khan Academy Negative Exponent Intuition

MEDIA
Click image to the left or use the URL below.
URL: http://www.ck12.org/flx/render/embeddedobject/59342

Guidance

Zero Exponent

Recall that

am
= am−n
an

. If m = n, then the following would be true:

am
= am−n = a0
an
33
= 33−3 = 30
33

33
However, any quantity divided by itself is equal to one. Therefore, 33
= 1 which means 30 = 1. This is true in
general:

a0 = 1 if a 6= 0.

Note that if a = 0, 00 is not defined.

173
4.4. Zero and Negative Exponents www.ck12.org

Negative Exponents

42 × 4−2 = 42+(−2) = 40 = 1
Therefore:

42 × 4−2 = 1
42 × 4−2 1
2
= 2 Divide both sides by 42 .
4 4
42 × 4−2 1
= Simplify the equation.
42 42
1
4−2 =
42

This is true in general and creates the following laws for negative exponents:


1
a−m =
am

1
= am
a−m

These laws for negative exponents can be expressed in many ways:

• If a term has a negative exponent, write it as 1 over the term with a positive exponent. For example: a−m = a1m
1
and a−m = am
−2 2
• If a term has a negative exponent, write the reciprocal with a positive exponent. For example: 23 = 32
−m
and a−m = a 1 = a1m
• If the term is a factor in the numerator with a negative exponent, write it in the denominator with a positive
n
exponent. For example: 3x−3 y = 3y x3
and a−m bn = a1m (bn ) = abm
• If the term is a factor in the denominator with a negative exponent, write it in the numerator with a positive
3 n m
exponent. For example: x2x−2 = 2x3 (x2 ) and ab−m = bn aa = bn am

These ways for understanding negative exponents provide shortcuts for arriving at solutions without doing tedious
calculations. The results will be the same.

Example A

Evaluate the following using the laws of exponents.


3 −2

4
Solution:
There are two methods that can be used to evaluate the expression.
Method 1: Apply the negative exponent rule
1
a−m =
am

174
www.ck12.org Chapter 4. Exponential Functions

 −2
3 1 1
= Write the expression with a positive exponent by applying a−m = .
4 3 2 am

4
1 1  a n an
2
= 32 Apply the law of exponents for raising a quotient to a power. =
3 b bn

4 42
1 1
32
= 9
Evaluate the powers.
42 16
1 9
9
= 1÷ Divide
16
16
9 16 16
1÷ = 1× =
16 9 9
 −2
3 16
=
4 9

Method 2: Apply the shortcut and write the reciprocal with a positive exponent.

 −2  2
3 4
= Write the reciprocal with a positive exponent.
4 3
 2
4 42  a n an
= 2 Apply the law of exponents for raising a quotient to a power. =
3 3 b bn
42 16
= Simplify.
32 9
 −2
3 16
=
4 9

Applying the shortcut facilitates the process for obtaining the solution.

Example B

State the following using only positive exponents: (If possible, use shortcuts)
i) y−6
−3
ii) ab
x5
iii) y−4

iv) a2 × a−5
Solutions:
i)

1
y−6 Write the expression with a positive exponent by applying a−m = .
am
1
y−6 =
y6

175
4.4. Zero and Negative Exponents www.ck12.org

ii)

 a −3
Write the reciprocal with a positive exponent.
b
 a −3  b 3  a n an
= Apply the law of exponents for raising a quotient to a power. =
b a b bn
 3
b b3
= 3
a a
 a −3 b3
= 3
b a

iii)

x5 1
Apply the negative exponent rule. = am
y−4 a−m
x5 y4
 
= x5 Simplify.
y−4 1
x5
= x 5 y4
y−4

iv)

a2 × a−5 Apply the product rule for exponents am × an = am+n .


a2 × a−5 = a2+(−5) Simplify.
1
a2+(−5) = a−3 Write the expression with a positive exponent by applying a−m = .
am
1
a−3 =
a3
1
a2 × a−5 =
a3

Example C

7−2 +7−1
Evaluate the following: 7−3 +7−4
Solution:
There are two methods that can be used to evaluate the problem.
Method 1: Work with the terms in the problem in exponential form.
Numerator:

176
www.ck12.org Chapter 4. Exponential Functions

1 1 1
7−2 = 2
and 7−1 = Apply the definition a−m =
7 7 am
1 1
+ A common denominator is needed to add the fractions.
72 7  
1 1 7 1 7
+ Multiply by to obtain the common denominator of 72
72 7 7 7 7
1 7 1+7 8
2
+ 2= 2 = 2 Add the fractions.
7 7 7 7

Denominator:

1 1 1
7−3 = 3
and 7−4 = 4 Apply the definition a−m =
7 7 am
1 1
+ A common denominator is needed to add the fractions.
73  74

7 1 1 1 7
3
+ 4 Multiply 3
by to obtain the common denominator of 74
7 7 7 7 7
7 1 1+7 8
4
+ 4= 4 = 4 Add the fractions.
7 7 7 7

Numerator and Denominator:

8 8
2
÷ 4 Divide the numerator by the denominator.
7 7
8 74
× Multiply by the reciprocal.
72 8
8 74 74
× = 2 = 72 = 49 Simplify.
72 8 7
7−2 + 7−1
= 49
7−3 + 7−4

Method 2: Multiply the numerator and the denominator by 74 . This will change all negative exponents to
positive exponents. Apply the product rule for exponents and work with the terms in exponential form.

7−2 + 7−1
7−3 + 7−4
 4  −2
7 7 + 7−1
Apply the distributive property with the product rule for exponents.
74 7−3 + 7−4
72 + 73
Evaluate the numerator and the denominator.
71 + 70
49 + 343 392
= = 49
7+1 8
7−2 + 7−1
= 49
7−3 + 7−4

Whichever method is used, the result is the same.

177
4.4. Zero and Negative Exponents www.ck12.org

Concept Problem Revisited

m xm
By the quotient rule for exponents, xxm = xm−m = x0 . Since anything divided by itself is equal to 1 (besides 0), xm = 1.
Therefore, x0 = 1 as long as x 6= 0.
x2
Also by the quotient rule for exponents, x5
= x2−5 = x−3 . If you were to expand and reduce the original expression
x2
you would have x5
= x·x
x·x·x·x·x = 1
x3
. Therefore, x−3 = 1
x3
. This generalizes to x−a = 1
xa .

Vocabulary

Base
In an algebraic expression, the base is the variable, number, product or quotient, to which the exponent refers.
Some examples are: In the expression 25 , ‘2’ is the base. In the expression (−3y)4 , ‘−3y’ is the base.

Exponent
In an algebraic expression, the exponent is the number to the upper right of the base that tells how many times
to multiply the base times itself. Some examples are:

In the expression 25 , ‘5’ is the exponent. It means to multiply 2 times itself 5 times as shown here: 25 = 2 × 2 ×
2×2×2
In the expression (−3y)4 , ‘4’ is the exponent. It means to multiply −3y times itself 4 times as shown here:
(−3y)4 = −3y × −3y × −3y × −3y.

Laws of Exponents
The laws of exponents are the algebra rules and formulas that tell us the operation to perform on the exponents
when dealing with exponential expressions.

Guided Practice

1. Use the laws of exponents to simplify the following: (−3x2 )3 (9x4 y)−2
2. Rewrite the following using only positive exponents. (x2 y−1 − 1)2
3. Use the laws of exponents to evaluate the following: [5−4 × (25)3 ]2
Answers:
1.

178
www.ck12.org Chapter 4. Exponential Functions

1
(−3x2 )3 (9x4 y)−2 Apply the laws of exponents (am )n = amn and a−m =
am
 
1
(−3x2 )3 (9x4 y)−2 = (−33 x6 ) Simplify and apply (ab)n = an bn
(9x4 y)2
   
3 6 1 6 1
(−3 x ) = −27x Simplify.
(9x4 y)2 (92 x8 y2 )
−27x6 am
 
6 1
− 27x = Simplify and apply the quotient rule for exponents = am−n .
(92 x8 y2 ) 81x8 y2 an
−27x6 1x−2 1
8 2
=− 2 Apply the negative exponent rule a−m =
81x y 3y am
1
(−3x2 )3 (9x4 y)−2 = −
3x2 y2

2.

(x2 y−1 − 1)2 Begin by expanding the binomial and using the FOIL m
to apply the product rule for exponents am × an = am+n
(x2 y−1 − 1)(x2 y−1 − 1) = (x2+2 y−1+(−1) − 1x2 y−1 − 1x2 y−1 + 1) Simplify.
1
(x2+2 y−1+(−1) − 1x2 y−1 − 1x2 y−1 + 1) = (x4 y−2 − 2x2 y−1 + 1) Apply the negative exponent rule a−m = .
am
 4
2x2

4 −2 2 −1 x
(x y − 2x y + 1) = − +1
y2 y
 4
2x2

2 −1 2 x
(x y − 1) = − +1
y2 y

3.

[5−4 × (25)3 ]2 Try to do this one by applying the laws of exponents.


−4 3 2 −4 2 3 2
[5 × (25) ] = [5 × (5 ) ]
[5−4 × (52 )3 ]2 = [5−4 × 56 ]2
[5−4 × 56 ]2 = (52 )2
(52 )2 = 54
54 = 625
[5−4 × (25)3 ]2 = 54 = 625

179
4.5. Fractional Exponents www.ck12.org

4.5 Fractional Exponents

Learning Objectives

Here you’ll learn how to work with exponents that are fractions.
If an exponent usually tells you the number of times to multiply the base by itself, what does it mean if the exponent
3
is a fraction? How can you think about and calculate 4 2 ?

Watch This

Khan Academy Level 3 Exponents

MEDIA
Click image to the left or use the URL below.
URL: http://www.ck12.org/flx/render/embeddedobject/59343

Guidance

A fraction exponent is related to a root. Raising a number to the power of 12 is the same as taking the square root of
m
the number. If you have a n , you can think about this expression in multiple ways:

m 1 m
 1 m
a n = (am ) n or a n = an

m √
n m √
an = am or a n = ( n a)m

All of these ideas can be summarized as the following rule for fractional exponents:

m √
n √
an = am = ( n a)m m, nεN

Example A

Simplify the following:


2
(125)− 3
Solution:

180
www.ck12.org Chapter 4. Exponential Functions

2 1
(125)− 3 Apply the law of exponents for negative exponents a−m = .
am
1 m √
n √ m
2 Apply the law of exponents for rational exponents a n = am = n a m, nεN.
125 3
1
 √ 2
3
125

The cube root of 125 is ‘5’.


1
52
Evaluate the denominator.

1
25
2 1
(125)− 3 =
25
1
a−m =
am

Example B

Simplify the following:


3
(2a2 b4 ) 2
Solution:

3
(2a2 b4 ) 2 Apply the law of exponents for raising a power to a power (am )n = amn .
3 3 3 3
(2a2 b4 ) 2 = 21× 2 (a2 ) 2 (b4 ) 2 Simplify the expression.
1× 23 3 3 3
2× 32 4× 23 m √
n √ m
2 (a2 ) (b4 ) = 2 (a)
2 2 2 (b) Simplify. Apply the rule for rational exponents a n = am = n a m, nεN .
2
32× 4×3 3 √
2 (a) 2 (b) 2 =
2 23 (a)3 (b)6 Simplify
√ √
23 (a)3 (b)6 = 8a3 b6
√ 3 6 √
8a b = 2 2a3 b6
3 √
(2a2 b4 ) 2 = 2 2a3 b6

Example C

The rational exponents represent the exponent and the index of the base. The numerator is the exponent and the
denominator is the index.
a) State the following using radicals:
3
i) 2 8

181
4.5. Fractional Exponents www.ck12.org

1
ii) 7− 5
3
iii) 3 4
b) State the following using exponents:

3
i) 72
1
ii)  √ 3
4
5

5
2
iii) a
Solutions:
a) i)
3
28
3 √
8 √
8
28 = 23 = 8

ii)
1
7− 5
1 1
7− 5 = 1
75
1 1
7− 5 = √
5
7

iii)
3
34
3 √
4 √
4
34 = 33 = 27

b) i)

3
72
√ 3 2
72 = 7 3

ii)
1
 √ 3
4
5
1 1
 √ 3 = 3
4
5 54

1 3
3 = 5− 4
5 4

iii)

5
2
a
√5
2 2
a = a5

182
www.ck12.org Chapter 4. Exponential Functions

Concept Problem Revisited


3
To determine the value of 4 2 , there are several methods that can be applied.
3
i) 4 2

3 √
4 2 = ( 4)3

‘3’ is the exponent and ‘2’ is the index. Remember the index tells what root to find. The ‘2’ is understood and is
never written when the operation is to take the square root of a number or term.


( 4)3 = (2)3 The square root of ‘40 is ‘20 .
(2)3 = 8

Therefore,
3
42 = 8

.
3
ii) 4 2

3 √
4 2 = 43 The first step is to evaluate 43 .
√ √
43 = 64 Take the square root of 64.

64 = 8

Therefore,
3
42 = 8

.
3
iii) 4 2

3
 1 3  1 m
42 = 42 Write the power as a n .
 1 3  √ 3 1
42 = 4 4 2 means to take the square root of 4.
 √ 3
4 = 23 Evaluate 23 .
23 = 8

Therefore,
3
42 = 8

183
4.5. Fractional Exponents www.ck12.org

iv)
3
42 Write 4 as a power of ‘20 .
3 3
4 2 = (22 ) 2 Raise the power to a power by multiplying the exponents.
3
2× 23
(22 ) = 2
2 Multiply the exponents.
2× 3
2  = 23
2 Evaluate 23 .
23 = 8

Therefore,
3
42 = 8

Vocabulary

Base
In an algebraic expression, the base is the variable, number, product or quotient, to which the exponent refers.
Some examples are: In the expression 25 , ‘2’ is the base. In the expression (−3y)4 , ‘−3y’ is the base.

Exponent
In an algebraic expression, the exponent is the number to the upper right of the base that tells how many times
to multiply the base times itself. Some examples are:

In the expression 25 , ‘5’ is the exponent. It means to multiply 2 times itself 5 times as shown here: 25 = 2 × 2 ×
2×2×2
In the expression (−3y)4 , ‘4’ is the exponent. It means to multiply −3y times itself 4 times as shown here:
(−3y)4 = −3y × −3y × −3y × −3y.

Laws of Exponents
The laws of exponents are the algebra rules and formulas that tell us the operation to perform on the exponents
when dealing with exponential expressions.

Guided Practice

3 1
1. Use the laws of exponents to evaluate the following: 9 2 ÷ 36− 2
3
2. Simplify the following using the laws of exponents. (20a2 b3 c−1 ) 2
2
64 3
3. Use the laws of exponents to evaluate the following: 1
216− 3
Answers:
3 1
1. 9 2 ÷ 36− 2

Apply the law of exponents for rational exponents


m √
n √ m 3
an = am = n a m, nεN to9 2

184
www.ck12.org Chapter 4. Exponential Functions

Apply the law of exponents for negative exponents

1 1
a−m = m
to36− 2
a
.

3
 √ 3 1 √ √ m
− 12 m n 1
9 ÷ 36 =
2 9 ÷ 1 Find the square root of 9. Apply a n = am = n a m, nεN 36 2 .
36 2
 √ 3 1 1
9 ÷ 1 = (3)3 ÷ √ Simplify
36 2 36
1 1
(3)3 ÷ √ = 27 ÷ √ Perform the indicated operation of division.
36 6
1 6
27 ÷ = 27 × = 162
6 1
3 1
9 2 ÷ 36− 2 = 162

2.

3
(20a2 b3 c−1 ) 2 Apply the law of exponents (ab)n = an bn .
3 3 3 3 3
(20a2 b3 c−1 ) 2 = 201× 2 (a)2× 2 (b)3× 2 (c)−1× 2 Simplify the exponents.
1× 23 2× 23 3× 32 −1× 23 3 9
− 23
20 (a) (b) (c) = 20 (a)3 (b) (c)
2 2

Apply the law of exponents for rational exponents


m √
n √ m
an = am = n a m, nεN
3 9 3
to 20 2 and b 2 . To c− 2 , apply the law for negative exponents

1
a−m =
am
and then the law
m √
n √ m
an = am = n a m, nεN

for rational exponents.

3 9
 √ 3 √ 1
3 − 323
20 (a) (b) (c) =
2 2 20 (a) b9 3 Simplify
c2
3 9 3
 √ 3  √  1

3 − 3 4
20 2 (a) (b) 2 (c) 2 = 2 5 (a ) b b √ Simplify
c3
 √ 3  √  1  √ √ 1
2 5 (a3 ) b4 b √ = 8 125a3 b4 b √
c3 c c
√ √ 1 √ √ √ −1
8 125a3 b4 b √ = 40 5a3 b4 b c c

Simplify
c c
3 √ √ √ −1
(20a2 b3 c−1 ) 2 = 40 5a3 b4 b c c

185
4.5. Fractional Exponents www.ck12.org

2
64 3
3. 1 .
216− 3

Numerator Denominator
2 1
64 3 216− 3
2
 √ 2 1 1
3
64 3 = 64 216− 3 = 1
216 3
 √ 2 1 1
3
64 = (4)2 216− 3 = √3
216
1 1
(4)2 = 16 √3 =
216 6
Numerator divided by denominator:

1
16 ÷
6
6
16 × = 96
1
2
64 3
1 = 96
216− 3

186
www.ck12.org Chapter 4. Exponential Functions

4.6 Exponential Expressions

Learning Objectives

Here you’ll learn how to use all of the laws of exponents to simplify and evaluate exponential expressions.
Can you simplify the following expression so that it has only positive exponents?

8x3 y−2
(−4a2 b4 )−2

Watch This

MEDIA
Click image to the left or use the URL below.
URL: http://www.ck12.org/flx/render/embeddedobject/59355

James Sousa: Simplify Exponential Expressions

Guidance

The following table summarizes all of the rules for exponents.


Laws of Exponents
If aεR, a ≥ 0 and m, nεQ, then

1. am × an = am+n
am m−n (if m > n, a 6= 0)
2. an = a
3. (a ) = amn
m n

(ab) n n n
4.
a n
 = aan b
5. b = bn (b 6= 0)
6. 0
a = 1 (a 6= 0)
7. a−m = a1m
m √
n √ m
8. a n = am = n a

Example A
1
Evaluate 81− 4 .
Solution:

187
4.6. Exponential Expressions www.ck12.org

First, rewrite with a positive exponent:


1 1
81− 4 = 11 = 81 1 4
.
81 4
Next, evaluate the fractional exponent:
r
1 4
1 4 1
81 = =1
81 3

Example B

Simplify (4x3 y)(3x5 y2 )4 .


Solution:

(4x3 y)(3x5 y2 )4 = (4x3 y)(81x20 y8 )


= 324x23 y9

Example C
 −2
x−2 y
Simplify x 4 y3
.
Solution:

−2 2
x−2 y x 4 y3
 
=
x 4 y3 x−2 y
= (x6 y2 )2
= x12 y4

Concept Problem Revisited

8x3 y−2
= (8x3 y−2 )(−4x2 y4 )2
(−4x2 y4 )−2
= (8x3 y−2 )(16x4 y8 )
= 8 · 16 · x3 · x4 · y−2 · y8
= 128x7 y6

Vocabulary

Base
In an algebraic expression, the base is the variable, number, product or quotient, to which the exponent refers.
Some examples are: In the expression 25 , ‘2’ is the base. In the expression (−3y)4 , ‘−3y’ is the base.

Exponent
In an algebraic expression, the exponent is the number to the upper right of the base that tells how many times
to multiply the base times itself. Some examples are:

188
www.ck12.org Chapter 4. Exponential Functions

In the expression 25 , ‘5’ is the exponent. It means to multiply 2 times itself 5 times as shown here: 25 = 2 × 2 ×
2×2×2
In the expression (−3y)4 , ‘4’ is the exponent. It means to multiply −3y times itself 4 times as shown here:
(−3y)4 = −3y × −3y × −3y × −3y.

Laws of Exponents
The laws of exponents are the algebra rules and formulas that tell us the operation to perform on the exponents
when dealing with exponential expressions.

Guided Practice

Use the laws of exponents to simplify each of the following:


1. (−2x)5 (2x2 )
2. (16x10 ) 43 x5


(x15 )(x24 )(x25 )


3. (x7 )8

Answers:
1. (−2x)5 (2x2 ) = (−32x5 )(2x2 ) = −64x7
2. (16x10 ) 43 x5 = 12x15


(x15 )(x24 )(x25 ) x64


3. (x7 )8
= x56
= x8

189
4.7. Graphs of Exponential Functions www.ck12.org

4.7 Graphs of Exponential Functions

Learning Objectives

Here you’ll learn how to graph exponential functions and how to compare the graphs of exponential functions on the
same coordinate axes.

Graphs of Exponential Functions

A colony of bacteria has a population of three thousand at noon on Monday. During the next week, the colony’s
population doubles every day. What is the population of the bacteria colony just before midnight on Saturday?
At first glance, this seems like a problem you could solve using a geometric sequence. And you could, if the bacteria
population doubled all at once every day; since it doubled every day for five days, the final population would be
3000 times 25 .
But bacteria don’t reproduce all at once; their population grows slowly over the course of an entire day. So how do
we figure out the population after five and a half days?

Exponential Functions

Exponential functions are a lot like geometrical sequences. The main difference between them is that a geometric
sequence is discrete while an exponential function is continuous.
Discrete means that the sequence has values only at distinct points (the 1st term, 2nd term, etc.)
Continuous means that the function has values for all possible values of x. The integers are included, but also all
the numbers in between.
The problem with the bacteria is an example of a continuous function. Here’s an example of a discrete function:
An ant walks past several stacks of Lego blocks. There is one block in the first stack, 3 blocks in the 2nd stack and
9 blocks in the 3rd stack. In fact, in each successive stack there are triple the number of blocks than in the previous
stack.
In this example, each stack has a distinct number of blocks and the next stack is made by adding a certain number of
whole pieces all at once. More importantly, however, there are no values of the sequence between the stacks. You
can’t ask how high the stack is between the 2nd and 3rd stack, as no stack exists at that position!
As a result of this difference, we use a geometric series to describe quantities that have values at discrete points, and
we use exponential functions to describe quantities that have values that change continuously.
When we graph an exponential function, we draw the graph with a solid curve to show that the function has values
at any time during the day. On the other hand, when we graph a geometric sequence, we draw discrete points to
signify that the sequence only has value at those points but not in between.
Here are graphs for the two examples above:

190
www.ck12.org Chapter 4. Exponential Functions

The formula for an exponential function is similar to the formula for finding the terms in a geometric sequence. An
exponential function takes the form

y = A · bx

where A is the starting amount and b is the amount by which the total is multiplied every time. For example, the
bacteria problem above would have the equation y = 3000 · 2x .

MEDIA
Click image to the left or use the URL below.
URL: http://www.ck12.org/flx/render/embeddedobject/133172

Compare Graphs of Exponential Functions

Let’s graph a few exponential functions and see what happens as we change the constants in the formula. The basic
shape of the exponential function should stay the same—but it may become steeper or shallower depending on the
constants we are using.
First, let’s see what happens when we change the value of A.
Compare the graphs of y = 2x and y = 3 · 2x .
Let’s make a table of values for both functions.

TABLE 4.1:
x y = 2x y = 3 · 2x
-3 1
8 y = 3 · 2−3 = 3 · 213 = 3
8
-2 1
4 y = 3 · 2−2 = 3 · 212 = 3
4

191
4.7. Graphs of Exponential Functions www.ck12.org

TABLE 4.1: (continued)


x y = 2x y = 3 · 2x
-1 1
2 y = 3 · 2−1 = 3 · 211 = 23
0 1 y = 3 · 20 = 3
1 2 y = 3 · 21 = 6
2 4 y = 3 · 22 = 3 · 4 = 12
3 8 y = 3 · 23 = 3 · 8 = 24

Now let’s use this table to graph the functions.

We can see that the function y = 3 · 2x is bigger than the function y = 2x . In both functions, the value of y doubles
every time x increases by one. However, y = 3 · 2x “starts” with a value of 3, while y = 2x “starts” with a value of 1,
so it makes sense that y = 3 · 2x would be bigger as its values of y keep getting doubled.
Similarly, if the starting value of A is smaller, the values of the entire function will be smaller.

Comparing Graphs

Compare the graphs of y = 2x and y = 13 · 2x .


Let’s make a table of values for both functions.

TABLE 4.2:
1
x y = 2x y= 3 · 2x
-3 1
8 y= 1
3 · 2−3 = 13 · 213 = 1
24
-2 1
4 y= 1
3 · 2−2 = 13 · 212 = 1
12
-1 1
2 y= 1
3 · 2−1 = 13 · 211 = 1
6
1
0 1 y= 3 · 20 = 31
1
1 2 y= 3 · 21 = 32
1
2 4 y= 3 · 22 = 31 · 4 = 43
1
3 8 y= 3 · 23 = 31 · 8 = 83

Now let’s use this table to graph the functions.

192
www.ck12.org Chapter 4. Exponential Functions

As we expected, the exponential function y = 13 · 2x is smaller than the exponential function y = 2x .


So what happens if the starting value of A is negative? Let’s find out.

Example C

Graph the exponential function y = −5 · 2x .


Solution
Let’s make a table of values:

TABLE 4.3:
x y = −5 · 2x
-2 − 54
-1 − 52
0 -5
1 -10
2 -20
3 -40

Now let’s graph the function:

193
4.7. Graphs of Exponential Functions www.ck12.org

This result shouldn’t be unexpected. Since the starting value is negative and keeps doubling over time, it makes
sense that the value of y gets farther from zero, but in a negative direction. The graph is basically just like the graph
of y = 5 · 2x , only mirror-reversed about the x−axis.
Now, let’s compare exponential functions whose bases (b) are different.

Graphing Multiple Functions

Graph the following exponential functions on the same graph: y = 2x , y = 3x , y = 5x , y = 10x .


First we’ll make a table of values for all four functions.

TABLE 4.4:
x y = 2x y = 3x y = 5x y = 10x
1 1 1 1
-2 4 9 25 100
1 1 1 1
-1 2 3 5 10
0 1 1 1 1
1 2 3 5 10
2 4 9 25 100
3 8 27 125 1000

Now let’s graph the functions.

194
www.ck12.org Chapter 4. Exponential Functions

Notice that for x = 0, all four functions equal 1. They all “start out” at the same point, but the ones with higher
values for b grow faster when x is positive—and also shrink faster when x is negative.
Finally, let’s explore what happens for values of b that are less than 1.

Example E

1 x

Graph the exponential function y = 5 · 2 .
Solution
Let’s start by making a table of values. (Remember that a fraction to a negative power is equivalent to its reciprocal
to the same positive power.)

TABLE 4.5:
1 x

x y = 5· 2
1 −3
-3 y = 5· 2 = 5 · 23 = 40
1 −2
-2 y = 5· 2 = 5 · 22 = 20
1 −1
-1 y = 5· 2 = 5 · 21 = 10
1 0
0 y = 5· 2 = 5·1 = 5
1 1 5
1 y = 5· 2 = 2
1 2
2 y = 5· 2 = 54

Now let’s graph the function.

195
4.7. Graphs of Exponential Functions www.ck12.org

This graph looks very different than the graphs from the previous example! What’s going on here?
When we raise a number greater than 1 to the power of x, it gets bigger as x gets bigger. But when we raise a number
smaller than 1 to the power of x, it gets smaller as x gets bigger—as you can see from the table of values above. This
makes sense because multiplying any number by a quantity less than 1 always makes it smaller.
So, when the base b of an exponential function is between 0 and 1, the graph is like an ordinary exponential graph,
only decreasing instead of increasing. Graphs like this represent exponential decay instead of exponential growth.
Exponential decay functions are used to describe quantities that decrease over a period of time.
When b can be written as a fraction, we can use the Property of Negative Exponents to write the function in a
x
different form. For instance, y = 5 · 12 is equivalent to 5 · 2−x . These two forms are both commonly used, so it’s
important to know that they are equivalent.

MEDIA
Click image to the left or use the URL below.
URL: http://www.ck12.org/flx/render/embeddedobject/133173

Examples

Example 1

Graph the exponential function y = 8 · 3−x


a.) Here is our table of values and the graph of the function.

TABLE 4.6:
x y = 8 · 3−x
-3 y = 8 · 3−(−3) = 8 · 33 = 216

196
www.ck12.org Chapter 4. Exponential Functions

TABLE 4.6: (continued)


x y = 8 · 3−x
-2 y = 8 · 3−(−2) = 8 · 32 = 72
-1 y = 8 · 3−(−1) = 8 · 31 = 24
0 y = 8 · 30 = 8
1 y = 8 · 3−1 = 83
2 y = 8 · 3−2 = 89

Example 2

Graph the functions y = 4x and y = 4−x on the same coordinate axes.


Here is the table of values for the two functions. Looking at the values in the table, we can see that the two functions
are “backwards” of each other, in the sense that the values for the two functions are reciprocals.

TABLE 4.7:
x y = 4x y = 4−x
-3 y = 4−3 = 641
y = 4−(−3) = 64
-2 −2
y = 4 = 16 1
y = 4−(−2) = 16
-1 y = 4−1 = 14 y = 4−(−1) = 4
0 y = 40 = 1 y = 40 = 1
1 y = 41 = 4 y = 4−1 = 14
2 y = 42 = 16 y = 4−2 = 161

3 y = 43 = 64 −3
y = 4 = 64 1

Here is the graph of the two functions. Notice that the two functions are mirror images of each other if the mirror is
placed vertically on the y−axis.

197
4.7. Graphs of Exponential Functions www.ck12.org

In the next lesson, you’ll see how exponential growth and decay functions can be used to represent situations in the
real world.

198
www.ck12.org Chapter 4. Exponential Functions

4.8 Graphing Transformed Exponential


Functions
Objective
To analyze and use exponential growth and decay functions.
Review Queue
Simplify the following expressions. Your final answer should have only positive exponents.
1. 2x2 · 6x4
5xy−1
2. 15x3 y3

3. (3x2 y5 )3
4. Solve for x: 2x = 32

Exponential Growth Function

Objective
To analyze an exponential growth function and its graph.
Guidance
An exponential function has the variable in the exponent of the expression. All exponential functions have the
form: f (x) = a · bx−h + k, where h and k move the function in the x and y directions respectively, much like the other
functions we have seen in this text. b is the base and a changes how quickly or slowly the function grows. Let’s take
a look at the parent graph, y = 2x .
Example A
Graph y = 2x . Find the y-intercept.
Solution: Let’s start by making a table. Include some positive and negative values for x and zero.

TABLE 4.8:
x 2x y
3 23 8
2 22 4
1 21 2
0 20 1
-1 2−1 1
2
-2 2−2 1
4
-3 2−3 1
8

199
4.8. Graphing Transformed Exponential Functions www.ck12.org

This is the typical shape of an exponential growth function. The function grows “exponentially fast”. Meaning, in
this case, the function grows in powers of 2. For an exponential function to be a growth function, a > 0 and b > 1
and h and k are both zero (y = abx ). From the table, we see that the y-intercept is (0, 1).
Notice that the function gets very, very close to the x-axis, but never touches or passes through it. Even if we chose
x = −50, y would be 2−50 = 2150 , which is still not zero, but very close. In fact, the function will never reach zero,
even though it will get smaller and smaller. Therefore, this function approaches the line y = 0, but will never touch
or pass through it. This type of boundary line is called an asymptote. In the case with all exponential functions,
there will be a horizontal asymptote. If k = 0, then the asymptote will be y = 0.
Example B
Graph y = 3x−2 + 1. Find the y-intercept, asymptote, domain and range.
Solution: This is not considered a growth function because h and k are not zero. To graph something like this
(without a calculator), start by graphing y = 3x and then shift it h units in the x-direction and k units in the y-direction.

200
www.ck12.org Chapter 4. Exponential Functions

Notice that the point (0, 1) from y = 3x gets shifted to the right 2 units and up one unit and is (2, 2) in the translated
function, y = 3x−2 + 1. Therefore, the asymptote is y = 1. To find the y-intercept, plug in x = 0.

1
y = 30−2 + 1 = 3−2 + 1 = 1 = 1.1
9

The domain of all exponential functions is all real numbers. The range will be everything greater than the asymptote.
In this example, the range is y > 1.
Example C
Graph the function y = − 21 · 4x . Determine if it is an exponential growth function.
Solution: In this example, we will outline how to use the graphing calculator to graph an exponential function.
First, clear out anything in Y =. Next, input the function into Y 1, − 21 4∧ X and press GRAPH. Adjust your window
accordingly.

201
4.8. Graphing Transformed Exponential Functions www.ck12.org

This is not an exponential growth function, because it does not grow in a positive direction. By looking at the
definition of a growth function, a > 0, and it is not here.
Guided Practice
Graph the following exponential functions. Determine if they are growth functions. Then, find the y-intercept,
asymptote, domain and range. Use an appropriate window.
1. y = 3x−4 − 2
2. f (x) = (−2)x+5
3. f (x) = 5x
4. Abigail is in a singles tennis tournament. She finds out that there are eight rounds until the final match. If the
tournament is single elimination, how many games will be played? How many competitors are in the tournament?
Answers
1 80
1. This is not a growth function because h and k are not zero. The y-intercept is y = 30−4 − 2 = 81 − 2 = −1 81 , the
asymptote is at y = −2, the domain is all real numbers and the range is y > −2.

202
www.ck12.org Chapter 4. Exponential Functions

2. This is not a growth function because h is not zero. The y-intercept is y = (−2)0+5 = (−2)5 = −32, the asymptote
is at y = 0, the domain is all real numbers and the range is y > 0.

3. This is a growth function. The y-intercept is y = 5◦ = 1, the asymptote is at y = 0, the domain is all real numbers
and the range is y > 0.

203
4.8. Graphing Transformed Exponential Functions www.ck12.org

4. If there are eight rounds to single’s games, there are will be 28 = 256 competitors. In the first round, there will
be 128 matches, then 64 matches, followed by 32 matches, then 16 matches, 8, 4, 2, and finally the championship
game. Adding all these all together, there will be 128 + 64 + 32 + 16 + 8 + 4 + 2 + 1 or 255 total matches.
Vocabulary

Exponential Function
A function whose variable is in the exponent. The general form is y = a · bx−h + k.

Exponential Growth Function


A specific type of exponential function where h = k = 0, a > 0, and b > 1. The general form is y = abx .

Asymptote
A boundary line that restricts the domain or range. This line is not part of the graph.

Exponential Decay Function

Objective
To graph and analyze an exponential decay function.
Guidance

x concept, we only addressed functions where |b|> 1. So, what happens when b is less than 1? Let’s analyze
In the last
y = 21 .

204
www.ck12.org Chapter 4. Exponential Functions

Example A
1 x

Graph y = 2 and compare it to y = 2x .
Solution: Let’s make a table of both functions and then graph.

TABLE 4.9:
1 x

x 2 2x
1 3 1
3 2 = 8 23 = 8
1 2 1
2 2 = 4 22 = 4
1 1 1
1 2 = 2 21 = 2
1 0
0 2 = 1 20 = 1
1 −1
-1 2 =2 2−1 = 12
1 −2
-2 2 =4 2−2 = 14
1 3
-3 2 =8 2−3 = 18
x
Notice that y = 12 is a reflection over the y-axis of y = 2x . Therefore, instead of exponential growth, the function
x
y = 12 decreases exponentially, or exponentially decays. Anytime b is a fraction or decimal between zero and one,
the exponential function will decay. And, just like an exponential growth function, and exponential decay function
has the form y = abx and a > 0. However, to be a decay function, 0 < b < 1. The exponential decay function also
has an asymptote at y = 0.
Example B
Determine which of the following functions are exponential decay functions, exponential growth functions, or
neither. Briefly explain your answer.
a) y = 4(1.3)x

205
4.8. Graphing Transformed Exponential Functions www.ck12.org

6 x

b) f (x) = 3 5
3 x

c) y = 10
d) g(x) = −2(0.65)x
Solution: a) and b) are exponential growth functions because b > 1. c) is an exponential decay function because b
is between zero and one. d) is neither growth or decay because a is negative.
Example C
2 x−1

Graph g(x) = −2 3 + 1. Find the y-intercept, asymptote, domain, and range.
∧
Solution: To graph this function, you can either plug it into your calculator (entered Y = −2 23 X − 1 + 1) or
x
graph y = −2 23 and shift it to the right one unit and up one unit. We will use the second method; final answer is
the blue function below.

The y-intercept is:


0−1
y = −2 23 + 1 = −2 · 32 + 1 = −3 + 1 = −2
The horizontal asymptote is y = 1, the domain is all real numbers and the range is y < 1.
Guided Practice
Graph the following exponential functions. Find the y-intercept, asymptote, domain, and range.
x
1. f (x) = 4 31
x+3
2. y = −2 23
x
3. g(x) = 35 − 6
4. Determine if the following functions are exponential growth, exponential decay, or neither.
a) y = 2.3x

206
www.ck12.org Chapter 4. Exponential Functions

4 −x

b) y = 2 3
c) y = 3 · 0.9x
1 4 x

d) y = 2 5
Answers
1.

y-intercept: (4, 0)
asymptote: y = 0
domain: all reals
range: y < 0
2.

207
4.8. Graphing Transformed Exponential Functions www.ck12.org

y-intercept: 0, − 16

27
asymptote: y = 0
domain: all reals
range: y < 0
3.

208
www.ck12.org Chapter 4. Exponential Functions

y-intercept: (−5, 0)
asymptote: y = −6
domain: all reals
range: y > −6
4. a) exponential growth
4 −x

b) exponential decay; recall that a negative exponent flips whatever is in the base. y = 2 3 is the same as
x
y = 2 34 , which looks like our definition of a decay function.
c) exponential decay
d) neither; a < 0
Vocabulary

Exponential Decay Function


An exponential function that has the form y = abx where a > 0 and 0 < b < 1.

209
4.9. Applications of Exponential Functions www.ck12.org

4.9 Applications of Exponential Functions

Learning Objectives

Here you’ll use different exponential functions in real-life situations.


The half-life of an isotope of barium is about 10 years. The half-life of a substance is the amount of time it takes for
half of that substance to decay. If a nuclear scientist starts with 200 grams of barium, how many grams will remain
after 100 years?

Growth and Decay Factors

When a real-life quantity increases by a percentage over a period of time, the final amount can be modeled by the
equation: A = P(1 + r)t , where A is the final amount, P is the initial amount, r is the rate (or percentage), and t is the
time (in years). 1 + r is known as the growth factor.
Conversely, a real-life quantity can decrease by a percentage over a period of time. The final amount can be modeled
by the equation: A = P(1 − r)t , where 1 − r is the decay factor.
Let’s solve the following problems.

1. The population of Coleman, Texas grows at a 2% rate annually. If the population in 2000 was 5981, what was
the population is 2010? Round up to the nearest person.

First, set up an equation using the growth factor. r = 0.02,t = 10, and P = 5981

A = 5981(1 + 0.02)10
= 5981(1.02)10
= 7291 people

2. You deposit $1000 into a savings account that pays 2.5% annual interest. Find the balance after 3 years if the
interest rate is compounded a) annually, b) monthly, and c) daily.

For part a, we will use A = 1000(1.025)3 = 1008.18.


But, to determine the amount if it is compounded
nt in amounts other than yearly, we need to alter the equation. For
compound interest, the equation is A = P 1 + nr , where n is the number of times the interest is compounded within
a year. For part b, n = 12.

0.025 12·3
 
A = 1000 1 +
12
= 1000(1.002)36
= 1077.80

In part c, n = 365.

210
www.ck12.org Chapter 4. Exponential Functions

0.025 365·3
 
A = 1000 1 +
365
= 1000(1.000068)1095
= 1077.88

3. You buy a new car for $35,000. If the value of the car decreases by 12% each year, what will the value of the
car be in 5 years?

This is a decay function because the value decreases.

A = 35000(1 − 0.12)5
= 35000(0.88)5
= 18470.62

The car would be worth $18,470.62 after five years.

Examples

Example 1

Earlier, you were asked to find the number of grams of barium that will remain after 100 years if you start with 200
grams and the half-life of this barium isotope is 10 years.
This is an example of exponential decay, so we can once again use the exponential form f (x) = a · bx−h + k. In this
case, a = 200, the starting amount; b is 1/2, the rate of decay; x-h = 100/10 = 10, and k = 0.

1 10
P = 200 ·
2
1
= 200 · = 0.195
1024

Therefore, 0.195 grams of the barium still remain 100 years later.

Example 2

Tommy bought a truck 7 years ago that is now worth $12,348. If the value of his truck decreased 14% each year,
how much did he buy it for? Round to the nearest dollar.
Tommy needs to use the formula A = P(1 − r)t and solve for P.

12348 = P(1 − 0.14)7


12348 = P(0.86)7 Tommy’s truck was originally $35, 490.
12348
= P ≈ 35490
(0.86)7

211
4.9. Applications of Exponential Functions www.ck12.org

Example 3

The Wetakayomoola credit card company charges an Annual Percentage Rate (APR) of 21.99%, compounded
monthly. If you have a balance of $2000 on the card, what would the balance be after 4 years (assuming you
do not make any payments)? If you pay $200 a month to the card, how long would it take you to pay it off? You
may need to make a table to help you with the second question.
nt
You need to use the formula A = P 1 + nr , where n = 12 because the interest is compounded monthly.

0.2199 12·4
 
A = 2000 1 +
12
= 2000(1018325)48
= 4781.65

To determine how long it will take you to pay off the balance, you need to find how much interest is com-
pounded in one month, subtract $200, and repeat. A table might be helpful. For each month after the first,
12·( 121 )
we will use the equation, B = R 1 + 0.2199 12 = R(1.018325), where B is the current balance and R is the
remaining balance from the previous month. For example, in month 2, the balance (including interest) would be
12·( 121 )
B = 1800 1 + 0.2199
12 = 1800 · 1.08325 = 1832.99.

TABLE 4.10:
Month 1 2 3 4 5 6 7 8 9 10 11
Balance 2000 1832.99 1662.91 1489.72 1313.35 930.09 790.87 640.06 476.69 299.73 108.03
Payment 200 200.00 200.00 200.00 200.00 200.00 200.00 200.00 200.00 200.00 108.03
Remain $1800 1632.99 1462.91 1289.72 913.35 730.09 590.87 440.06 276.69 99.73 0

It is going to take you 11 months to pay off the balance and you are going to pay 108.03 in interest, making your
total payment $2108.03.

Example 4

As the altitude increases, the atmospheric pressure (the pressure of the air around you) decreases. For every 1000
feet up, the atmospheric pressure decreases about 4%. The atmospheric pressure at sea level is 101.3. If you are on
top of Hevenly Mountain at Lake Tahoe (elevation about 10,000 feet) what is the atmospheric pressure?
The equation will be A = 101, 325(1 − 0.04)100 = 1709.39. The decay factor is only raised to the power of 100
because for every 1000 feet the pressure decreased. Therefore, 10, 000 ÷ 1000 = 100. Atmospheric pressure is what
you don’t feel when you are at a higher altitude and can make you feel light-headed. The picture below demonstrates
the atmospheric pressure on a plastic bottle. The bottle was sealed at 14,000 feet elevation (1), and then the resulting
pressure at 9,000 feet (2) and 1,000 feet (3). The lower the elevation, the higher the atmospheric pressure, thus the
bottle was crushed at 1,000 feet.

212
www.ck12.org Chapter 4. Exponential Functions

213
4.10. References www.ck12.org

4.10 References

1. . . CC BY-NC
2. . . CC BY-NC
3. . . CC BY-NC
4. . . CC BY-NC
5. . . CC BY-NC
6. . . CC BY-NC
7. . . CC BY-NC
8. . . CC BY-NC
9. . . CC BY-NC
10. . . CC BY-NC
11. . . CC BY-NC
12. quantockgoblin. https://commons.wikimedia.org/wiki/File:Plastic_bottle_at_14000_feet,_9000_feet_and_1000
_feet,_sealed_at_14000_feet.png .

214
www.ck12.org Chapter 5. Trigonometric Functions and Applications

C HAPTER
5 Trigonometric Functions
and Applications
Chapter Outline
5.1 S INE , C OSINE , TANGENT
5.2 S OLVING R IGHT T RIANGLES
5.3 S PECIAL R IGHT T RIANGLES
5.4 R EFERENCE A NGLES AND A NGLES IN THE U NIT C IRCLE
5.5 S IGNS OF T RIGONOMETRIC R ATIOS ON THE U NIT C IRCLE
5.6 T RIGONOMETRIC R ATIOS ON THE U NIT C IRCLE
5.7 I NTRODUCTION TO T RIG I DENTITIES
5.8 L AWS OF S INES AND C OSINES
5.9 A MBIGUITY AND 3D T RIGONOMETRY
5.10 S INUSOIDAL F UNCTIONS
5.11 T RANSFORMING T RIGONOMETRIC F UNCTIONS
5.12 M ODELLING S INUSOIDAL F UNCTIONS
5.13 T RIGONOMETRIC A PPLICATIONS
5.14 R EFERENCES

215
5.1. Sine, Cosine, Tangent www.ck12.org

5.1 Sine, Cosine, Tangent

Learning Objectives

Here you’ll define and apply the trigonometric ratios sine, cosine and tangent to solve for the lengths of unknown
sides in right triangles.
An isoceles right triangle has leg lengths of 4 units each. What is the sine of each of the triangle’s acute angles?

Sine, Cosine and Tangent

The trigonometric ratios sine, cosine and tangent refer to the known ratios between particular sides in a right triangle
based on an acute angle measure.

In this right triangle, side c is the hypotenuse.


If we consider the angle B, then we can describe each of the legs by its position relative to angle B: side a is adjacent
to B; side b is opposite B
If we consider the angle A, then we can describe each of the legs by its position relative to angle A: side b is adjacent
to A; side a is opposite A
Now we can define the trigonometry ratios as follows:

opposite ad jacent opposite


Sine is Cosine is Tangent is
hypotenuse hypotenuse ad jacent

A shorthand way to remember these ratios is to take the letters in red above and write the phrase:

SOH CAH TOA

Now we can find the trigonometric ratios for each of the acute angles in the triangle above.

216
www.ck12.org Chapter 5. Trigonometric Functions and Applications

a b
sin A = sin B =
c c
b a
cos A = cos B =
c c
a b
tan A = tan B =
b a

It is important to understand that given a particular (acute) angle measure in a right triangle, these ratios are constant
no matter how big or small the triangle. For example, if the measure of the angle is 25◦ , then sin 25◦ ≈ 0.4226 and
ratio of the opposite side to the hypotenuse is always 0.4226 no matter how big or small the triangle.

MEDIA
Click image to the left or use the URL below.
URL: http://www.ck12.org/flx/render/embeddedobject/182859

Let’s find the trig ratios for the acute angles R and P in ∆PQR.

From angle R, O = 8; A = 15; and H = 17. Now the trig ratios are:

8 15 8
sin R = ; cos R = ; tan R =
17 17 15

From angle P, O = 15; A = 8; and H = 17. Now the trig ratios are:

15 8 15
sin P = ; cos P = ; tan P =
17 17 8

Do you notice any patterns or similarities between the trigonometric ratios? The opposite and adjacent sides are
switched and the hypotenuse is the same. Notice how this switch affects the ratios:

217
5.1. Sine, Cosine, Tangent www.ck12.org

1
sin R = cos P cos R = sin P tan R =
tan P

Now, let’s use trigonometric ratios to find x and y.

First identify or label the sides with respect to the given acute angle. So, x is opposite, y is hypotenuse (note that
it is the hypotenuse because it is the side opposite the right angle, it may be adjacent to the given angle but the
hypotenuse cannot be the adjacent side) and 6 is the adjacent side.
To find x, we must use the given length of 6 in our ratio too. So we are using opposite and adjacent. Since tangent is
the ratio of opposite over adjacent we get:

x
tan 35◦ =
6
x = 6 tan 35◦ multiply both sides by 6
x ≈ 4.20 Use the calculator to evaluate-type in 6TAN(35) ENTER

NOTE: make sure that your calculator is in DEGREE mode. To check, press the MODE button and verify that
DEGREE is highlighted (as opposed to RADIAN). If it is not, use the arrow buttons to go to DEGREE and press
ENTER. The default mode is radian, so if your calculator is reset or the memory is cleared it will go back to radian
mode until you change it.
To find y using trig ratios and the given length of 6, we have adjacent and hypotenuse so we’ll use cosine:

6
cos 35◦ =
y
cos 35◦ 6
= set up a proportion to solve for y
1 y
6 = y cos 35◦ cross multiply
6
y= divide by cos 35◦
cos 35◦
y = 7.32 Use the calculator to evaluate-type in 6/TAN(35) ENTER

Alternatively, we could find y using the value we found for x and the Pythagorean theorem:

4.202 + 62 = y2
53.64 = y2
y ≈ 7.32

218
www.ck12.org Chapter 5. Trigonometric Functions and Applications

The downside of this method is that if we miscalculated our x value, we will double down on our mistake and
guarantee an incorrect y value. In general you will help avoid this kind of mistake if you use the given information
whenever possible.
Finally, given ∆ABC, with m6 A = 90◦ , m6 C = 20◦ and c = 9, let’s find a and b.
Visual learners may find it particularly useful to make a sketch of this triangle and label it with the given information:

To find a (the hypotenuse) we can use the opposite side and the sine ratio: sin 20◦ = 9a , solving as we did in the
previous problem, we get a = sin920◦ ≈ 26.31. To find b (the adjacent side) we can use the opposite side and the
tangent ratio: tan 20◦ = 9b , solving for b we get b = tan920◦ ≈ 24.73.

Examples

Example 1

Earlier, you were asked to find the sine of each of the triangle’s acute angles.
opposite
If you draw the triangle described in this problem, you will see that the sine hypotenuse of each of the acute angles in
4
the same. It is hypotenuse . So we need to find the hypotenuse.
Let’s use the Pythagorean Theorem.

42 + 42 = c2
16 + 16 = c2
32 = c2

c=4 2

4
Therefore, the sine of both of the acute angles is √ or 2.
2
4 2

Example 2

Use trig ratios to find x and y:

219
5.1. Sine, Cosine, Tangent www.ck12.org

For x:
5
cos 62◦ =
x
5
x= ≈ 10.65
cos 62◦

For y:
y
tan 62◦ =
5
y = 5 tan 62◦ ≈ 9.40

Example 3

Given ∆ABC with m6 B = 90◦ , m6 A = 43◦ and a = 7, find b and c.


For b:
7
sin 43◦ =
b
7
b= ≈ 10.26
sin 43◦

For c:
7
tan 43◦ =
c
7
c= ≈ 7.51
tan 43◦

Example 4

The base of a playground slide is 6 ft from the base of the platform and the slide makes a 60◦ angle with the ground.
To the nearest tenth of a foot, how high is the platform at the top of the slide?

h
tan 60◦ =
6
h = 6 tan 60◦ ≈ 10.39
So the height of the platform is 10.4 ft.

220
www.ck12.org Chapter 5. Trigonometric Functions and Applications

5.2 Solving Right Triangles

Review Questions

1. Solve the triangle.

2. Two friends are writing practice problems to study for a trigonometry test. Sam writes the following problem
for his friend Anna to solve: In right triangle ABC, the measure of angle C is 90 degrees, and the length of
side c is 8 inches. Solve the triangle. Anna tells Sam that the triangle cannot be solved. Sam says that she is
wrong. Who is right? Explain your thinking.
3. Use the Pythagorean Theorem to verify the sides of the triangle in example 2.
4. Estimate the measure of angle B in the triangle below using the fact that sin B = 35 and sin 30◦ = 12 . Use a
calculator to find sine values. Estimate B to the nearest degree.
5. The angle of elevation from the ground to the top of a flagpole is measured to be 53◦ . If the measurement was
taken from 15 feet away, how tall is the flagpole?
6. From the top of a hill, the angle of depression to a house is measured to be 14◦ . If the hill is 30 feet tall, how
far away is the house?
7. A surveyor is measuring the width of a pond. She chooses a landmark on the opposite side of the pond, and
measures the angle to this landmark from a point 50 feet away from the original point. How wide is the pond?

221
5.2. Solving Right Triangles www.ck12.org

8. Find the length of side x:

9. A deck measuring 10 feet by 16 feet will require laying boards with one board running along the diagonal
and the remaining boards running parallel to that board. The boards meeting the side of the house must be cut
prior to being nailed down. At what angle should the boards be cut?

222
www.ck12.org Chapter 5. Trigonometric Functions and Applications

Review Answers

1.

6 A = 50◦
b ≈ 5.83
a ≈ 9.33

2. Anna is correct. There is not enough information to solve the triangle. That is, there are infinitely many right
triangles with hypotenuse 8. For example:

3. 62 + 5.032 = 36 + 25.3009 = 61.3009 = 7.832 .


4. 6 B ≈37◦
5. About 19.9 feet tall
6. About 120.3 feet
7. About 41.95 feet
8. About 7.44
9.
opposite
tan θ =
ad jacent
tan θ = 0.625
θ = 32◦

223
5.3. Special Right Triangles www.ck12.org

5.3 Special Right Triangles

Learning Objectives

Here you will review properties of 30-60-90 and 45-45-90 right triangles.
The Pythagorean Theorem is great for finding the third side of a right triangle when you already know two other
sides. There are some triangles like 30-60-90 and 45-45-90 triangles that are so common that it is useful to know the
side ratios without doing the Pythagorean Theorem each time. Using these patterns also allows you to totally solve
for the missing sides of these special triangles when you only know one side length.
Given a 45-45-90 right triangle with sides 6 inches, 6 inches and x inches, what is the value of x?

Special Right Triangles

There are three types of special right triangles, 30-60-90 triangles, 45-45-90 triangles, and Pythagorean triple
triangles.

30-60-90 Triangles

A 30-60-90 right triangle has side ratios x, x 3, 2x.

Confirm with Pythagorean Theorem:

 √ 2
x2 + x 3 = (2x)2
x2 + 3x2 = 4x2
4x2 = 4x2

45-45-90 Triangles

A 45-45-90 right triangle has side ratios x, x, x 2.

224
www.ck12.org Chapter 5. Trigonometric Functions and Applications

Confirm with Pythagorean Theorem:

 √ 2
x2 + x2 = x 2
2x2 = 2x2
√ √
Note that the order of the side ratios x, x 3, 2x and x, x, x 2 is important because each side ratio has a corresponding
angle. In all triangles, the smallest sides correspond to smallest angles and largest sides always correspond to the
largest angles.

Pythagorean Triple Triangles

Pythagorean number triples are special right triangles with integer sides. While the angles are not integers, the side
ratios are very useful to know because they show up everywhere. Knowing these number triples also saves a lot of
time from doing the Pythagorean Theorem repeatedly. Here are some examples of Pythagorean number triples:

• 3, 4, 5
• 5, 12, 13
• 7, 24, 25
• 8, 15, 17
• 9, 40, 41

More Pythagorean number triples can be found by scaling any other Pythagorean number triple. For example:

225
5.3. Special Right Triangles www.ck12.org

3, 4, 5 → 6, 8, 10 (scaled by a factor of 2)
Even more Pythagorean number triples can be found by taking any odd integer like 11, squaring it to get 121, halving
the result to get 60.5. The original number 11 and the two numbers that are 0.5 above and below (60 and 61) will
always be a Pythagorean number triple.
112 + 602 = 612

Examples

Example 1

Earlier you were asked about a 45-45-90 right triangle with sides 6 inches, 6 inches and x inches. If you can
recognize√the pattern for 45-45-90
√ right triangles, a right triangle with legs 6 inches and 6 inches has a hypotenuse
that is 6 2 inches. x = 6 2.

Example 2

A 30-60-90 right triangle has hypotenuse of length 10. What are the lengths of the other two sides?
The hypotenuse is the side opposite 90. Sometimes it is helpful to draw a picture or make a table.

TABLE 5.1:
30 60 90

x x 3 2x

10

From the table you can write very small subsequent equations to solve for the missing sides.

10 = 2x
x=5
√ √
x 3=5 3


The other sides are 5 and 5 3.

Example 3

A 30-60-90 right triangle has a side length of 18 inches corresponding to 60 degrees. What are the lengths of the
other two sides?
Make a table with the side ratios and the information given, then write equations and solve for the missing side
lengths.

TABLE 5.2:
30 60 90

226
www.ck12.org Chapter 5. Trigonometric Functions and Applications

TABLE 5.2: (continued)



x x 3 2x

18


18 = x 3
18
√ =x
3
√ √
18 18 3 18 3 √
x= √ = √ · √ = =6 3
3 3 3 3

x=6 3

Note that you need to rationalize denominators.


Now use the calculated x value to solve for 2x.


2x = 2(6 3)

2x = 12 3

√ √
The other sides are 6 3 and 12 3.

Example 4

Using your knowledge of special right triangle ratios, solve for the missing sides of the right triangle.


The other sides are each 5 2.
2

TABLE 5.3:
45 45 90

x x x 2

5
227
5.3. Special Right Triangles www.ck12.org


x 2=5
√ √
5 2 5 2
x= √ · √ =
2 2 2

The other sides are each 5 2.
2

Example 5

Using your knowledge of special right triangle ratios, solve for the missing sides of the right triangle.


The other sides are 9 and 6 3.

TABLE 5.4:
30 60 90

x x 3 2x

3 3


x=3 3

2x = 6 3
√ √ √
x 3 = 3 3· 3 = 9


The other sides are 9 and 6 3.

228
www.ck12.org Chapter 5. Trigonometric Functions and Applications

5.4 Reference Angles and Angles in the Unit


Circle

Learning Objectives

Here you’ll learn the definition of reference angles and how to express angles on the unit circle.
When you walk into math class one day, your teacher has a surprise for the class. You’re going to play series of
games related to the things you’ve been learning about in class. For the first game, your teacher hands each group
of students a spinner with an "x" and "y" axis marked. The game is to see how many angles you identify correctly.
However, in this game, you are supposed to give what is called the "reference angle". You spin your spinner three
times. Each picture below shows one of the spins:

229
5.4. Reference Angles and Angles in the Unit Circle www.ck12.org

Can you correctly identify the reference angles for these pictures?

Reference Angles

Consider the angle 150◦ . If we graph this angle in standard position, we see that the terminal side of this angle is a
reflection of the terminal side of 30◦ , across the y−axis.

230
www.ck12.org Chapter 5. Trigonometric Functions and Applications

Notice that 150◦ makes a 30◦ angle with the negative x−axis. Therefore we say that 30◦ is the reference angle for
150◦ . Formally, the reference angle of an angle in standard position is the angle formed with the closest portion of
the x−axis. Notice that 30◦ is the reference angle for many angles. For example, it is the reference angle for 210◦
and for −30◦ .
In general, identifying the reference angle for an angle will help you determine the values of the trig functions of the
angle.

MEDIA
Click image to the left or use the URL below.
URL: http://www.ck12.org/flx/render/embeddedobject/70413

Identifying Reference Angles

Graph each of the following angles and identify their reference angles.
a. 140◦
140◦ makes a 40◦ angles with the negative x−axis. Therefore the reference angle is 40◦ .
b. 240◦
240◦ makes a 60◦ angle with the negative x−axis. Therefore the reference angle is 60◦

231
5.4. Reference Angles and Angles in the Unit Circle www.ck12.org

c. 380◦
380◦ is a full rotation of 360◦ , plus an additional 20◦ . So this angle is co-terminal with 20◦ , and 20◦ is its reference
angle.

Determining the Value of Trigonometric Functions

1. Find the ordered pair for 240◦ and use it to find the value of sin 240◦ .

sin 240◦ = − 2 3
As we found in part b under the question above, the reference angle for 240◦ is 60◦ . The figure below shows 60◦ and
the three other angles in the unit circle that have 60◦ as a reference angle.

232
www.ck12.org Chapter 5. Trigonometric Functions and Applications

The terminal side of the angle ◦ a reflection of the terminal side of 60◦ over √
 240 √represents
 both axes. So the
coordinates of the point are − 2 , − 2 . The y−coordinate is the sine value, so sin 240◦ = − 2 3 .
1 3

Just as the figure above shows 60◦ and three related angles, we can make similar graphs for 30◦ and 45◦ .

Knowing these ordered pairs will help you find the value of any of the trig functions for these angles.
2. Find the value of cot 300◦
cot 300◦ = − √1
3
 √ 
1 3. Therefore the cotangent value is cot 300◦ =
Using the graph above, you will find that the ordered pair is 2,− 2
1
x
y = 2
√ = 12 × − √2 = − √1
− 3 2
3 3

233
5.4. Reference Angles and Angles in the Unit Circle www.ck12.org

We can also use the concept of a reference angle and the ordered pairs we have identified to determine the values of
the trig functions for other angles.

Examples

Example 1

Earlier, you were asked if you can correctly identify the reference angles in the pictures.
Since you know how to measure reference angles now, upon examination of the spinners, you know that the first
angle is 30◦ , the second angle is 45◦ , and the third angle is 60◦ .

Example 2

Graph 210◦ and identify its reference angle.


The graph of 120◦ looks like this:

and since the angle makes a 60◦ angle with the negative "x" axis, the reference angle is 60◦ .

Example 3

Graph 315◦ and identify its reference angle.


The graph of 315◦ looks like this:

234
www.ck12.org Chapter 5. Trigonometric Functions and Applications

and since the angle makes a 45◦ angle with the positive "x" axis, the reference angle is 45◦ .

Example 4

Find the ordered pair for 150◦ and use it to find the value of cos 150◦ .
 √ 
Since the reference angle is 30◦ , 3 1
we know that the coordinates for the point on the unit circle are − 2 , 2 . This
is the same as the value for 30◦ , except the "x" coordinate is negative instead of positive. Knowing this,

235
5.5. Signs of Trigonometric Ratios on the Unit Circle www.ck12.org

5.5 Signs of Trigonometric Ratios on the Unit


Circle
Learning Objectives

Here you’ll learn the domain and range, as well as the sign in different quadrants, for six trig functions. You are
doodling in art class one day when you draw a circle. Then you draw a few lines extending outward from the center
to the edge of the circle. You draw a triangle with the "x" axis, and realize that you’re thinking about your math class
again.

You notice that the relationship for the sine function involves the length of the side opposite the angle divided by the
length of the hypotenuse. But while the hypotenuse is always a positive number, the sign of the opposite side can be
different, depending on what quadrant the angle is drawn in.
Can you determine what the sign of the sine function will be in each of the four quadrants, based on the knowledge
of the ratio that defines the sine function?

Domain and Range of Trigonometric Functions

While the trigonometric functions may seem quite different from other functions you have worked with, they are in
fact just like any other function. We can think of a trig function in terms of “input” and “output.” The input is always
an angle. The output is a ratio of sides of a triangle. If you think about the trig functions in this way, you can define
the domain and range of each function.

236
www.ck12.org Chapter 5. Trigonometric Functions and Applications

MEDIA
Click image to the left or use the URL below.
URL: http://www.ck12.org/flx/render/embeddedobject/143441

Let’s first consider the sine and cosine functions. The input of each of these functions is always an angle, and as
you learned in the previous sections, these angles can take on any real number value. Therefore the sine and cosine
function have the same domain, the set of all real numbers, R. We can determine the range of the functions if we
think about the fact that the sine of an angle is the y−coordinate of the point where the terminal side of the angle
intersects the unit circle. The cosine is the x−coordinate of that point. Now recall that in the unit circle, we defined
the trig functions in terms of a triangle with hypotenuse 1.

In this right triangle, x and y are the lengths of the legs of the triangle, which must have lengths less than 1, the length
of the hypotenuse. Therefore the ranges of the sine and cosine function do not include values greater than one. The
ranges do, however, contain negative values. Any angle whose terminal side is in the third or fourth quadrant will
have a negative y−coordinate, and any angle whose terminal side is in the second or third quadrant will have a
negative x−coordinate.

237
5.5. Signs of Trigonometric Ratios on the Unit Circle www.ck12.org

In either case, the minimum value is -1. For example, cos 180◦ = −1 and sin 270◦ = −1. Therefore the sine and
cosine function both have range from -1 to 1.
The table below summarizes the domains and ranges of these functions:

TABLE 5.5:
Domain Range
Sine θ=R −1 ≤ y ≤ 1
Cosine θ=R −1 ≤ y ≤ 1

Knowing the domain and range of the cosine and sine function can help us determine the domain and range of
the secant and cosecant function. First consider the sine and cosecant functions, which as we showed above, are
reciprocals. The cosecant function will be defined as long as the sine value is not 0. Therefore the domain of the
cosecant function excludes all angles with sine value 0, which are 0◦ , 180◦ , 360◦ , etc.
In Chapter 2 you will analyze the graphs of these functions, which will help you see why the reciprocal relationship
results in a particular range for the cosecant function. Here we will state this range, and in the review questions you
will explore values of the sine and cosecant function in order to begin to verify this range, as well as the domain and
range of the secant function.

TABLE 5.6:
Domain Range
Cosecant θεR, θ 6= 0, 180, 360 . . . csc θ ≤ −1 or csc θ ≥ 1
Secant θεR, θ 6= 90, 270, 450 . . . sec θ ≤ −1 or sec θ ≥ 1

Now let’s consider the tangent and cotangent functions. The tangent function is defined as tan θ = xy . Therefore the
domain of this function excludes angles for which the ordered pair has an x−coordinate of 0 : 90◦ , 270◦ , etc. The

238
www.ck12.org Chapter 5. Trigonometric Functions and Applications

cotangent function is defined as cot θ = xy , so this function’s domain will exclude angles for which the ordered pair
has a y−coordinate of 0 : 0◦ , 180◦ , 360◦ , etc.

TABLE 5.7:
Function Domain Range
Tangent θεR, θ 6= 90, 270, 450 . . . All reals
Cotangent θεR, θ 6= 0, 180, 360 . . . All reals

Knowing the ranges of these functions tells you the values you should expect when you determine the value of a trig
function of an angle. However, for many problems you will need to identify the sign of the function of an angle: Is
it positive or negative?
In determining the ranges of the sine and cosine functions above, we began to categorize the signs of these functions
in terms of the quadrants in which angles lie. The figure below summarizes the signs for angles in all 4 quadrants.

An easy way to remember this is “All Students Take Calculus”. Quadrant I: All values are positive, Quadrant II:
Sine is positive, Quadrant III: Tangent is positive, and Quadrant IV: Cosine is positive. This simple memory device
will help you remember which trig functions are positive and where.

Stating the Sign

1. State the sign of cos 100◦


The angle 100◦ is in the second quadrant. Therefore the x−coordinate is negative and so cos 100◦ is negative.
2. State the sign of csc 220◦
The angle 220◦ is in the third quadrant. Therefore the y−coordinate is negative. So the sine, and the cosecant are
negative.

239
5.5. Signs of Trigonometric Ratios on the Unit Circle www.ck12.org

3. State the sign of tan 370◦


The angle 370◦ is in the first quadrant. Therefore the tangent value is positive.

Examples

Example 1

Earlier, you were asked to determine what the sign of the sine function will be in each of the four quadrants.
Since the sine function is defined to be the length of the opposite side divided by the length of the hypotenuse, the
sign of the sine function is the sign of the y -coordinate for whatever quadrant is being considered. In quadrants
1 and 2, the "y" coordinate is positive, so the sine function is positive. In quadrants 3 and 4, the y-coordinate is
negative, so the sine function is negative as well.

Example 2

State the sign of cos 70◦


The angle 70◦ is in the first quadrant. Cosine is defined to be the adjacent side divided by the hypotenuse. Since
the hypotenuse of the unit circle is one and the adjacent side is the x-coordinate, the sign of the cosine function is
determined by the sign of the x-coordinate. Since 70◦ is in the first quadrant, the x value is positive. Therefore the
cosine value is positive.

Example 3

State the sign of sin 130◦


The angle 130◦ is in the second quadrant. Sine is defined to be the opposite side divided by the hypotenuse. Since
the hypotenuse of the unit circle is one and the opposite side is the y-coordinate, the sign of the sine function is
determined by the sign of the y-coordinate. Since 130◦ is in the second quadrant, the y value is positive. Therefore
the sine value is positive.

Example 4

State the sign of tan 250◦


The angle 250◦ is in the third quadrant. Tangent is defined to be the opposite side divided by the adjacent side. In
the third quadrant, the x values are negative, and the y values are negative. A negative divided by a negative equals a
positive. Therefore the tangent of 250◦ is positive.

240
www.ck12.org Chapter 5. Trigonometric Functions and Applications

5.6 Trigonometric Ratios on the Unit Circle

Here you’ll learn how to determine exact value of trigonometric ratios for multiples of 0◦ , 30◦ and 45◦ (or 0, π6 , π4
radians).
What are the exact values of the following trigonometric functions?

a. cos 495◦
b. tan 300◦

Trigonometric Ratios on the Unit Circle


Recall special right triangles from Geometry. In a (30◦ − 60◦ − 90◦ ) triangle, the sides are in the ratio 1 : 3 : 2.

In an isosceles triangle (45◦ − 45◦ − 90◦ ), the congruent sides and the hypotenuse are in the ratio 1 : 1 : 2.


In a (30◦ − 60◦ − 90◦ ) triangle, the sides are in the ratio 1 : 3 : 2.

Now let’s make the hypotenuse equal to 1 in each of the triangles so we’ll be able to put them inside the unit circle.
Using the appropriate ratios, the new side lengths are:

241
5.6. Trigonometric Ratios on the Unit Circle www.ck12.org

Using these triangles, we can evaluate sine, cosine and tangent for each of the angle measures.

√ √
◦ 2 ◦3 1
sin 45 = sin 60 = sin 30◦ =
2
√ 2 2√
2 1 3
cos 45◦ = cos 60◦ = cos 30◦ =
2 2

2
3

◦ ◦ 2
√ ◦
1
2 3
tan 45 = 1 tan 60 = 1
= 3 tan 30 = √ =
2
3 2
2

These triangles can now fit inside the unit circle.

Putting together the trigonometric ratios and the coordinates of the points on the circle, which represent the lengths of
(∆x, ∆y), we can see that each point is actually (cos θ, sin θ), where θ is the reference angle.
the legs of the triangles, √
For example, sin 60 = 3 is the y - coordinate of the point on the unit circle in the triangle with reference angle

2
60◦ . By reflecting these triangles across the axes and finding the points on the axes, we can find the trigonometric
ratios of all multiples of 0◦ , 30◦ and 45◦ .

242
www.ck12.org Chapter 5. Trigonometric Functions and Applications

MEDIA
Click image to the left or use the URL below.
URL: http://www.ck12.org/flx/render/embeddedobject/99159

Let’s solve the following problems using the unit circle.

1. Find sin 270◦ .

Find 270◦ on the unit circle and the corresponding point is (0, −1). Since each point on the unit circle is (cos θ, sin θ), sin 270◦ =
−1.

2. Find tan 210◦ .



This time we need to look at the ratio sin θ
cos θ . We can use the unit circle to find sin 210◦ = − 21 and cos 210◦ = − 3.
√ 2
− 21

Now, tan 210 = √ = √1
= 33 .
− 23 3

243
5.6. Trigonometric Ratios on the Unit Circle www.ck12.org

Quadrants in a Unit Circle

Another way to approach these exact value problems is to use the reference angles and the special right triangles.
The benefit of this method is that there is no need to memorize the entire unit circle. If you memorize the special
right triangles, can determine reference angles and know where the ratios are positive and negative you can put the
pieces together to get the ratios. Looking at the unit circle above, we see that all of the ratios are positive in Quadrant
I, sine is the only positive ratio in Quadrant II, tangent is the only positive ratio in Quadrant III and cosine is the only
positive ratio in Quadrant IV.
Keeping this diagram in mind will help you remember where cosine, sine and tangent are positive and negative. You
can also use the pneumonic device - All Students Take Calculus, or ASTC, to recall which is positive (all the others
would be negative) in which quadrant.
The coordinates on the vertices will help you determine the ratios for the multiples of 90◦ .

Now, let’s find the exact values for the following trigonometric functions using the alternative method.

1. cos 120◦

First, we need to determine in which quadrant the angles lies. Since 120◦ is between 90◦ and 180◦ it will lie in
Quadrant II. Next, find the reference angle. Since we are in QII, we will subtract from 180◦ to get 60◦ . We can use
the reference angle to find the ratio, cos 60◦ = 12 . Since we are in QII where only sine is positive, cos 120◦ = − 12 .

2. sin 300◦

The angle √300◦ lies in QIV√and the reference angle is 60◦ . This means that our ratio will be negative. Since
sin 60◦ = 2 3 , sin 300◦ = − 2 3 .

Examples

Example 1

Earlier, you were asked to find the exact values of the following trigonometric ratios.

a. cos 495◦

First, we need to determine in which quadrant the angle lies. Since 495◦ − 360◦ = 135◦ is between 90◦ and 180◦ it
will lie in Quadrant II. Next, find the reference angle. Since we are in QII, we will subtract from 180◦ to get 45◦ .

244
www.ck12.org Chapter 5. Trigonometric Functions and Applications

We can use the reference angle to find the ratio, cos 45◦ = 2
√ 2 . Since we are in QII where only sine is positive,
cos 495◦ = − 2 . 2

b. tan 300◦

300◦ lies in QIV and


In problem #2 above we established that the angle √ ◦
√ the reference angle is 60 . This means that
◦ ◦
the tangent ratio will be negative. Since tan 60 = 3, tan 300 = − 3.
Find the exact trigonometric ratios. You may use either method.

Example 2

sin 405◦
√ √
405◦ has a reference angle of 45◦ in QI. sin 45◦ = 2 and since sine is positive in QI, sin 405 = 2 2 .
2 ◦

Example 3

cos 120◦
120◦ is coterminal to 60◦ in QII. cos 60◦ = 1
2 and since cosine is negative in QII, cos 120◦ = 12 .

245
5.7. Introduction to Trig Identities www.ck12.org

5.7 Introduction to Trig Identities

Learning Objectives

Here you’ll learn about the basic trigonometric identities and how to use them.
You are given a list of Trig Identities. One of those identities is cos(−θ) = cos θ. Prove this identity without
graphing.

Trigonometric Identities

Trigonometric identities are true for any value of x (as long as the value is in the domain). You have learned about
secant, cosecant, and cotangent, which are all reciprocal functions of sine, cosine and tangent. These functions can
be rewritten as the Reciprocal Identities because they are always true.
1 1 1
Reciprocal Identities: csc θ = sin θ sec θ = cos θ cot θ = tan θ
Other identities involve the tangent, variations on the Pythagorean Theorem, phase shifts, and negative angles. We
will discover them in this concept.
opposite sin θ
We know that tan θ = ad jacent . Let’s show that tan θ = cos θ . This is the Tangent Identity.
Whenever we are trying to verify, or prove, an identity, we start with the statement we are trying to prove and
sin θ
work towards the desired answer. In this case, we will start with tan θ = cos θ and show that it is equivalent to
opposite
tan θ = ad jacent . First, rewrite sine and cosine in terms of the ratios of the sides.

sin θ
tan θ =
cos θ
opposite
hypotenuse
= ad jacent
hypotenuse

Then, rewrite the complex fraction as a division problem and simplify.

opposite ad jacent
= ÷
hypotenuse hypotenuse
opposite ( hypotenuse
((((
(
= ·
hypotenuse
( (((( ad jacent
(
opposite
=
ad jacent

We now have what we wanted to prove and we are done. Once you verify an identity, you may use it to verify
other identities.
Now, let’s show that sin2 θ + cos2 θ = 1 is a true identity.
Change the sine and cosine in the equation into the ratios. In this problem, we will use y as the opposite side, x is
the adjacent side, and r is the hypotenuse (or radius), as in the unit circle.

246
www.ck12.org Chapter 5. Trigonometric Functions and Applications

sin2 θ + cos2 θ = 1
 y 2  x 2
+ =1
r r
y2 x2
+ =1
r2 r2
y2 + x2
=1
r2

Now, x2 + y2 = r2 from the Pythagorean Theorem. Substitute this in for the numerator of the fraction.

r2
=1
r2

This is one of the Pythagorean Identities and very useful.



Finally, let’s verify that sin π2 − θ = cos θ by using the graphs of the functions.

The function y = sin π2 − x is a phase shift of π2 of the sine curve.

The red function above is y = sin x and the blue is y = cos x. If we were to shift the sine curve π2 , it would overlap
perfectly with the cosine curve, thus proving this Cofunction Identity.

MEDIA
Click image to the left or use the URL below.
URL: http://www.ck12.org/flx/render/embeddedobject/91979

Examples

Example 1

Earlier, you were asked to prove the identity of cos(−θ) = cos θ without graphing.
First, recall that cos θ = x, where (x, y) is the endpoint of the terminal side of θ on the unit circle.
Now, if we have cos(−θ), what is its endpoint? Well, the negative sign tells us that the angle is rotated in a clockwise
direction, rather than the usual counter-clockwise. If we make this rotation, we see that cos(−θ) = x as well, as
illustrated in the following diagram.

247
5.7. Introduction to Trig Identities www.ck12.org

We know that x = x, so we can set the two expressions equal to one another.
cos θ = cos(−θ)
We can now flip this identity around to get:
cos(−θ) = cos θ

Example 2

Prove the Pythagorean Identity: 1 + tan2 θ = sec2 θ.


First, let’s use the Tangent Identity and the Reciprocal Identity to change tangent and secant in terms of sine and
cosine.

1 + tan2 θ = sec2 θ
sin2 θ 1
1+ =
cos2 θ cos2 θ

Now, change the 1 into a fraction with a base of cos2 θ and simplify.

sin2 θ 1
1+ =
cos2 θ cos2 θ
cos θ sin2 θ
2 1
+ =
cos2 θ cos2 θ cos2 θ
cos2 θ + sin2 θ 1
=
cos2 θ cos2 θ
1 1
=
cos2 θ cos2 θ

In the second to last step, we arrived at the original Pythagorean Identity sin2 θ + cos2 θ in the numerator of the
left-hand side. Therefore, we can substitute in 1 for this and the two sides of the equation are the same.

Example 3

Without graphing, show that sin(−θ) = − sin θ.


First, recall that sin θ = y, where (x, y) is the endpoint of the terminal side of θ on the unit circle.

248
www.ck12.org Chapter 5. Trigonometric Functions and Applications

Now, if we have sin(−θ), what is it’s endpoint? Well, the negative sign tells us that the angle is rotated in a clockwise
direction, rather than the usual counter-clockwise. By looking at the picture, we see that sin(−θ) = −y. Therefore,
if sin θ = y, then − sin θ = −y and combining the equations, we have sin(−θ) = − sin θ.

249
5.8. Laws of Sines and Cosines www.ck12.org

5.8 Laws of Sines and Cosines

Learning Objectives

Here you’ll learn how to solve for missing sides and angles in non-right triangles using the Laws of Sines and
Cosines.

Laws of Sines and Cosines

sin A sin B sinC


Law of Sines: If 4ABC has sides of length, a, b, and c, then a = b = c .
Looking at a triangle, the lengths a, b, and c are opposite the angles of the same letter.

Use Law of Sines when given:

• An angle and its opposite side.


• Any two angles and one side.

Two sides and the non-included angle.

MEDIA
Click image to the left or use the URL below.
URL: http://www.ck12.org/flx/render/embeddedobject/10312

Law of Cosines: If 4ABC has sides of length a, b, and c, then:

a2 = b2 + c2 − 2bc cos A
b2 = a2 + c2 − 2ac cos B
c2 = a2 + b2 − 2ab cosC

250
www.ck12.org Chapter 5. Trigonometric Functions and Applications

Even though there are three formulas, they are all very similar. First, notice that whatever angle is in the cosine, the
opposite side is on the other side of the equal sign.
Use Law of Cosines when given:

• Two sides and the included angle.


• All three sides.

MEDIA
Click image to the left or use the URL below.
URL: http://www.ck12.org/flx/render/embeddedobject/10313

Using the Law of Sines

1. Solve the triangle using the Law of Sines. Round decimal answers to the nearest tenth.

First, to find m6 A, we can use the Triangle Sum Theorem.

m6 A + 85◦ + 38◦ = 180◦


m6 A = 57◦

Now, use the Law of Sines to set up ratios for a and b.

sin 57◦ sin 85◦ sin 38◦


= =
a b 12

sin 57◦ sin 38◦ sin 85◦ sin 38◦


= =
a 12 b 12
a · sin 38 = 12 · sin 57◦

b · sin 38 = 12 · sin 85◦

12 · sin 57◦ 12 · sin 85◦


a= ≈ 16.4 b= ≈ 19.4
sin 38◦ sin 38◦

251
5.8. Laws of Sines and Cosines www.ck12.org

2. Solve the triangle using the Law of Sines. Round decimal answers to the nearest tenth.

Set up the ratio for 6 B using Law of Sines.

sin 95◦ sin B


=
27 16
27 · sin B = 16 · sin 95◦
16 · sin 95◦ 16 · sin 95◦
 
sin B = → sin−1 = 36.2◦
27 27

To find m6 C use the Triangle Sum Theorem. m6 C + 95◦ + 36.2◦ = 180◦ → m6 C = 48.8◦
sin 95◦ sin 48.8◦
To find c, use the Law of Sines again. 27 = c

c · sin 95◦ = 27 · sin 48.8◦


27 · sin 48.8◦
c= ≈ 20.4
sin 95◦

Using the Law of Cosines

Solve the triangle using Law of Cosines. Round your answers to the nearest hundredth.

Use the second equation to solve for 6 B.

b2 = 262 + 182 − 2(26)(18) cos 26◦


b2 = 1000 − 936 cos 26◦
b2 = 158.7288
b ≈ 12.60

To find m6 A or m6 C, you can use either the Law of Sines or Law of Cosines. Let’s use the Law of Sines.

252
www.ck12.org Chapter 5. Trigonometric Functions and Applications

sin 26◦ sin A


=
12.60 18
12.60 · sin A = 18 · sin 26◦
18 · sin 26◦
sin A =
12.60
18·sin 26◦
sin−1 ≈ 38.77◦ To find m6 C, use the Triangle Sum Theorem.

12.60

26◦ + 38.77◦ + m6 C = 180◦


m6 C = 115.23◦

MEDIA
Click image to the left or use the URL below.
URL: http://www.ck12.org/flx/render/embeddedobject/137601

Examples

Find the following angles in the triangle bloew. Round your answers to the nearest hundredth.

Example 1

m6 A
When you are given only the sides, you have to use the Law of Cosines to find one angle and then you can use the
Law of Sines to find another.

152 = 222 + 282 − 2(22)(28) cos A


225 = 1268 − 1232 cos A
−1043 = −1232 cos A
 
−1043 1043
= cos A → cos−1 ≈ 32.16◦
−1232 1232

253
5.8. Laws of Sines and Cosines www.ck12.org

Example 2

m6 B
Now that we have an angle and its opposite side, we can use the Law of Sines.

sin 32.16◦ sin B


=
15 22
15 · sin B = 22 · sin 32.16◦
22 · sin 32.16◦
sin B =
15

22·sin 32.16◦
sin−1 ≈ 51.32◦ .

15

Example 3

m6 C
To find m6 C, use the Triangle Sum Theorem.

32.16◦ + 51.32◦ + m6 C = 180◦


m6 C = 96.52◦

254
www.ck12.org Chapter 5. Trigonometric Functions and Applications

5.9 Ambiguity and 3D Trigonometry

Sine Law and Ambiguity

255
5.9. Ambiguity and 3D Trigonometry www.ck12.org

256
www.ck12.org Chapter 5. Trigonometric Functions and Applications

257
5.9. Ambiguity and 3D Trigonometry www.ck12.org

258
www.ck12.org Chapter 5. Trigonometric Functions and Applications

259
5.9. Ambiguity and 3D Trigonometry www.ck12.org

3D Problems by Using Trigonometry

260
www.ck12.org Chapter 5. Trigonometric Functions and Applications

261
5.9. Ambiguity and 3D Trigonometry www.ck12.org

262
www.ck12.org Chapter 5. Trigonometric Functions and Applications

263
5.9. Ambiguity and 3D Trigonometry www.ck12.org

264
www.ck12.org Chapter 5. Trigonometric Functions and Applications

5.10 Sinusoidal Functions

265
5.10. Sinusoidal Functions www.ck12.org

266
www.ck12.org Chapter 5. Trigonometric Functions and Applications

267
5.10. Sinusoidal Functions www.ck12.org

268
www.ck12.org Chapter 5. Trigonometric Functions and Applications

269
5.10. Sinusoidal Functions www.ck12.org

270
www.ck12.org Chapter 5. Trigonometric Functions and Applications

271
5.10. Sinusoidal Functions www.ck12.org

272
www.ck12.org Chapter 5. Trigonometric Functions and Applications

273
5.10. Sinusoidal Functions www.ck12.org

274
www.ck12.org Chapter 5. Trigonometric Functions and Applications

275
5.11. Transforming Trigonometric Functions www.ck12.org

5.11 Transforming Trigonometric Functions

276
www.ck12.org Chapter 5. Trigonometric Functions and Applications

277
5.11. Transforming Trigonometric Functions www.ck12.org

278
www.ck12.org Chapter 5. Trigonometric Functions and Applications

279
5.11. Transforming Trigonometric Functions www.ck12.org

280
www.ck12.org Chapter 5. Trigonometric Functions and Applications

281
5.11. Transforming Trigonometric Functions www.ck12.org

282
www.ck12.org Chapter 5. Trigonometric Functions and Applications

5.12 Modelling Sinusoidal Functions

283
5.12. Modelling Sinusoidal Functions www.ck12.org

284
www.ck12.org Chapter 5. Trigonometric Functions and Applications

285
5.12. Modelling Sinusoidal Functions www.ck12.org

286
www.ck12.org Chapter 5. Trigonometric Functions and Applications

287
5.13. Trigonometric Applications www.ck12.org

5.13 Trigonometric Applications

288
www.ck12.org Chapter 5. Trigonometric Functions and Applications

289
5.13. Trigonometric Applications www.ck12.org

290
www.ck12.org Chapter 5. Trigonometric Functions and Applications

291
5.14. References www.ck12.org

5.14 References

1. CK-12 Foundation;CK-12. CK-12 . CCSA


2. CK-12 Foundation;CK-12. CK-12 . CCSA
3. CK-12 Foundation;CK-12. CK-12 . CCSA;CC BY-NC 3.0
4. CK-12 Foundation. . CCSA;CC BY-NC-SA
5. CK-12 Foundation. . CCSA
6. CK-12. CK-12 .
7. CK-12;Gustavb. CK-12;Wikimedia .
8. CK-12;Gustavb. CK-12;Wikimedia .
9. ck12. ck12.org .
10. ck12. ck12.org .
11. ck12. ck12.org .
12. ck12. ck12.org .
13. Nelson. Nelson . CC-BY-SA
14. Nelson. Nelson . CC-BY-SA
15. Nelson. Nelson . CC-BY-SA
16. Nelson. Nelson . CC-BY-SA
17. Nelson. Nelson . CC-BY-SA
18. Nelson. Nelson . CC-BY-SA
19. Nelson. Nelson . CC-BY-SA
20. Nelson. Nelson . CC-BY-SA
21. Nelson. Nelson . CC-BY-SA
22. Nelson. Nelson . CC-BY-SA
23. Nelson. Nelson . CC-BY-SA
24. Nelson. Nelson . CC-BY-SA
25. Nelson. Nelson . CC-BY-SA
26. Nelson. Nelson . CC-BY-SA
27. Nelson. Nelson . CC-BY-SA
28. Nelson. Nelson . CC-BY-SA
29. Nelson. Nelson . CC-BY-SA
30. Nelson. Nelson . CC-BY-SA
31. Nelson. Nelson . CC-BY-SA
32. Nelson. Nelson . CC-BY-SA
33. Nelson. Nelson . CC-BY-SA
34. Nelson. Nelson . CC-BY-SA
35. Nelson. Nelson . CC-BY-SA
36. Nelson. Nelson . CC-BY-SA
37. Nelson. Nelson . CC-BY-SA
38. Nelson. Nelson . CC-BY-SA
39. Nelson. Nelson . CC-BY-SA
40. Nelson. Nelson . CC-BY-SA
41. Nelson. Nelson . CC-BY-SA
42. Nelson. Nelson . CC-BY-SA
43. Nelson. Nelson . CC-BY-SA
44. Nelson. Nelson . CC-BY-SA
45. Nelson. Nelson . CC-BY-SA

292
www.ck12.org Chapter 5. Trigonometric Functions and Applications

46. Nelson. Nelson . CC-BY-SA


47. Nelson. Nelson . CC-BY-SA
48. Nelson. Nelson . CC-BY-SA
49. Nelson. Nelson . CC-BY-SA
50. Nelson. Nelson . CC-BY-SA

293
www.ck12.org

C HAPTER
6 Discrete Functions
Chapter Outline
6.1 A RITHMETIC S EQUENCES AND S ERIES
6.2 G EOMETRIC S EQUENCES AND S ERIES
6.3 S IMPLE AND C OMPOUND I NTEREST
6.4 A NNUITIES (F UTURE VALUE )
6.5 A NNUITIES (P RESENT VALUE )
6.6 R EFERENCES

294
www.ck12.org Chapter 6. Discrete Functions

6.1 Arithmetic Sequences and Series

Objective
Identify arithmetic sequences, find the nth term rule for an arithmetic sequence and find the sum of a finite arithmetic
sequence.
Review Queue
Describe the pattern in each sequence below.

1. 3, 5, 7, 9, . . .
2. 1, 4, 8, 13, . . .
5
3. Find ∑ 7n + 3
n=1

Arithmetic Sequences and Finding the N[U+1D57]h Term Given the Common
Difference and a Term

Objective
Identify an arithmetic sequence and its common difference and write an nth term rule given the common difference
and a term.
Guidance
In this concept we will begin looking at a specific type of sequence called an arithmetic sequence. In an arithmetic
sequence the difference between any two consecutive terms is constant. This constant difference is called the
common difference. For example, question one in the Review Queue above is an arithmetic sequence. The
difference between the first and second terms is (5 − 3) = 2, the difference between the second and third terms
is (7 − 5) = 2 and so on. We can generalize this in the equation below:
an − an−1 = d, where an−1 and an represent two consecutive terms and d represents the common difference.
Since the same value, the common difference, d, is added to get each successive term in an arithmetic sequence we
can determine the value of any term from the first term and how many time we need to add d to get to the desired
term as illustrated below:
Given the sequence: 22, 19, 16, 13, . . . in which a1 = 22 and d = −3

a1 = 22 or 22 + (1 − 1)(−3) = 22 + 0 = 22
a2 = 19 or 22 + (2 − 1)(−3) = 22 + (−3) = 19
a3 = 16 or 22 + (3 − 1)(−3) = 22 + (−6) = 16
a4 = 13 or 22 + (4 − 1)(−3) = 22 + (−9) = 13
..
.
an = 22 + (n − 1)(−3)
an = 22 − 3n + 3
an = −3n + 25

295
6.1. Arithmetic Sequences and Series www.ck12.org

Now we can generalize this into a rule for the nth term of any arithmetic sequence:

an = a1 + (n − 1)d

Example A
Find the common difference and nth term rule for the arithmetic sequence: 2, 5, 8, 11 . . .
Solution: To find the common difference we subtract consecutive terms.

5−2 = 3
8 − 5 = 3 , thus the common difference is 3.
11 − 8 = 3

Now we can put our first term and common difference into the nth term rule discovered above and simplify the
expression.

an = 2 + (n − 1)(3)
= 2 + 3n − 3 , so an = 3n − 1.
= 3n − 1

Example B
Find the nth term rule and thus the 100th term for the arithmetic sequence in which a1 = −9 and d = 2.
Solution: We have what we need to plug into the rule:

an = −9 + (n − 1)(2)
= −9 + 2n − 2 , thus the nth term rule is an = 2n − 11.
= 2n − 11

Now to find the 100th term we can use our rule and replace n with 100: a100 = 2(100) − 11 = 200 − 11 = 189.
Example C
Find the nth term rule and thus the 100th term for the arithmetic sequence in which a3 = 8 and d = 7.
Solution: This one is a little less straightforward as we will have to first determine the first term from the term we
are given. To do this, we will replace an with a3 = 8 and use 3 for n in the formula to determine the unknown first
term as shown:

a1 + (3 − 1)(7) = 8
a1 + 2(7) = 8
a1 + 14 = 8
a1 = −6

Now that we have the first term and the common difference we can follow the same process used in the previous
example to complete the problem.

296
www.ck12.org Chapter 6. Discrete Functions

an = −6 + (n − 1)(7)
= −6 + 7n − 7 , thus an = 7n − 13.
= 7n − 13

Now we can find the 100th term: a100 = 7(100) − 13 = 687.


Guided Practice
1. Find the common difference and the nth term rule for the sequence: 5, −3, −11, . . .
2. Write the nth term rule and find the 45th term for the arithmetic sequence with a10 = 1 and d = −6.
3. Find the 62nd term for the arithmetic sequence with a1 = −7 and d = 32 .
Answers
1. The common difference is −3 − 5 = −8. Now an = 5 + (n − 1)(−8) = 5 − 8n + 8 = −8n + 13.
2. To find the first term:

a1 + (10 − 1)(−6) = 1
a1 − 54 = 1
a1 = 55

Find the nth term rule: an = 55 + (n − 1)(−6) = 55 − 6n + 6 = −6n + 61.


Finally, the 45th term: a45 = −6(45) + 61 = −209.
3. This time we will not simplify the nth term rule, we will just use the formula to find the 62rd term: a62 =
−7 + (62 − 1) 32 = −7 + 61 32 = − 14 183 169

2 + 2 = 2 .
Vocabulary

Arithmetic Sequence
A sequence in which the difference between any two consecutive terms is constant.

Common Difference
The value of the constant difference between any two consecutive terms in an arithmetic sequence.

Finding the N[U+1D57]h Term Given Two Terms

Objective
Write an nth term rule for an arithmetic sequence given any two terms in the sequence.
Guidance
In the last concept we were given the common difference directly or two consecutive terms from which we could
determine the common difference. In this concept we will find the common difference and write nth term rule given
any two terms in the sequence.
Example A
Find the common difference, first term and nth term rule for the arithmetic sequence in which a7 = 17 and a20 = 82.

297
6.1. Arithmetic Sequences and Series www.ck12.org

Solution: We will start by using the nth term rule for an arithmetic sequence to create two equations in two variables:
a7 = 17, so a1 + (7 − 1)d = 17 or more simply: a1 + 6d = 17
a20 = 82, so a1 + (20 − 1)d = 82 or more simply: a1 + 19d = 82
Solve the resulting system:

a1 + 6d = 17 a1 + 6d = 17
−1(a1 + 19d = 82) ⇒ −a1 − 19d = −82
− 13d = −65
d=5

, replacing d with 5 in one of the equations we get

a1 + 6(5) = 7
a1 + 30 = 17
a1 = −13

.
Using these values we can find the nth term rule:

an = −13 + (n − 1)(5)
an = −13 + 5n − 5
an = 5n − 18

Example B
Find the common difference, first term and nth term rule for the arithmetic sequence in which a11 = −13 and
a40 = −71.
Solution: Though this is exactly the same type of problem as Example A, we are going to use a different approach.
We discovered in the last concept that the nth term rule is really just using the first term and adding d to it n − 1 times
to find the nth term. We are going to use that idea to find the common difference. To get from the 11th term to the
40th term, the common difference is added 40 − 11 or 29 times. The difference in the term values is −71 − (−13)
or -58. What must be added 29 times to create a difference of -58? We can subtract the terms and divide by the
difference in term number to determine the common difference.

−71 − (−13) −71 + 13 −58


= = = −2. So d = −2.
40 − 11 29 29

Now we can use the common difference and one of the terms to find the first term as we did previously.

a1 + (11 − 1)(−2) = −13


a1 + (−20) = −13
a1 = 7

Writing the nth term rule we get: an = 7 + (n − 1)(−2) = 7 − 2n + 2 = −2n + 9.

298
www.ck12.org Chapter 6. Discrete Functions

More Guidance
Before we look at the final example for this concept, we are going to connect the nth term rule for an arithmetic
sequence to the equation of a line. Have you noticed that the simplified nth term rule, an = pn + q, where p and q
represent constants, looks a little like y = mx + b, the slope-intercept form of the equation of a line? Let’s explore
why this is the case using the arithmetic sequence 1, 4, 7, 10, . . . If we create points by letting the x - coordinate be
the term number and the y - coordinate be the term, we get the following points and can plot them in the coordinate
plane as shown below,

The points are: (1, 1), (2, 4), (3, 7), (4, 10)
Notice, that all of these points lie on the same line. This happens because for each increase of one in the term number
(x), the term value (y) increases by 3. This common difference is actually the slope of the line.
We can find the equation of this line using the slope, 3, and the point (1, 1) in the equation y = mx + b as follows:

1 = 3(1) + b
1 = 2 + b , so the equation of the line is y = 3x − 1
−1 = b

The nth term rule for the sequence is thus: an = 3n − 1.


Example C
Find the common difference, first term and nth term rule for the arithmetic sequence in which a10 = −50 and
a32 = −182.
Solution: This time we will use the concept that the terms in an arithmetic sequence are actually points on a line to
write an equation. In this case our points are (10, −50) and (32, −182). We can find the slope and the equation as
shown.

−182 − (−50) −132


m= = = −6
32 − 10 22

299
6.1. Arithmetic Sequences and Series www.ck12.org

Use the point (10, −50) so find the y-intercept:

−50 = −6(10) + b
−50 = −60 + b
10 = b

, so y = −6x + 10 and an = −6n + 10.


Guided Practice
1. Use the method in Example A to find the nth term rule for the arithmetic sequence with a6 = −13 and a15 = −40.
2. Use the method in Example B to find the nth term rule for the arithmetic sequence with a6 = 13 and a22 = 77.
3. Use the method in Example C to find the nth term rule for the arithmetic sequence with a7 = −75 and a25 = −273.
Answers
1. From a6 = −13 we get the equation a1 + (6 − 1)d = a1 + 5d = −13.
From a15 = −40 we get the equation a1 + (15 − 1)d = a1 + 14d = −40.
Use the two equations to solve for a1 and d:

a1 + 5d = −13 a1 + 5(−3) = −13


−a1 − 14d = 40 Use d to find a1 ⇒ a1 − 15 = −13.
− 9d = 27 a1 = 2
d = −3

Find the nth term rule: an = 2 + (n − 1)(−3) = 2 − 3n + 3 = −3n + 5.


77−13 64
2. The common difference is 22−6 = 16 = 4. The first term can be found using a6 = 13:

a1 + (6 − 1)(4) = 13
a1 + 20 = 13
a1 = −7

. Thus an = −7 + (n − 1)(4) = −7 + 4n − 4 = 4n − 11.


3. From a7 = −75 we get the point (7, −75). From a25 = −273 we get the point (25, −273). The slope between
these points is −273−(−75)
25−7 = −198
18 = −11. The y-intercept can be found next using the point (7, −75):

−75 = −11(7) + b
−75 = −77 + b
2=b

The final equation is y = −11x + 2 and the nth term rule is an = −11n + 2.

Finding the Sum of a Finite Arithmetic Series

Objective

300
www.ck12.org Chapter 6. Discrete Functions

Find the sum of an arithmetic series using the formula and the calculator.
Guidance
In the concept Series and Summation Notation we explored how to use the calculator to evaluate the sum of a
series. This method can be used to find the sum of an arithmetic series as well. However, in this concept we will
explore an algebraic method unique to arithmetic series. As we discussed earlier in the unit a series is simply the
sum of a sequence so an arithmetic series is a sum of an arithmetic sequence. Let’s look at an example to illustrate
this and develop a formula to find the sum of a finite arithmetic series.
Example A
Find the sum of the arithmetic series: 1 + 3 + 5 + 7 + 9 + 11 + . . . + 35 + 37 + 39
Solution: Now, while we could just add up all of the terms to get the sum, if we had to sum a large number of
terms that would be very time consuming. A famous German mathematician, Johann Carl Friedrich Gauss, used the
method described here to determine the sum of the first 100 integers in grade school. First, we can write out all the
numbers twice, in ascending and descending order, and observe that the sum of each pair of numbers is the same:

1 3 5 7 9 11 ... 35 37 39
39 37 35 33 31 29 ... 5 3 1
..
.
40 40 40 40 40 40 . . . 40 40 40

Notice that the sum of the corresponding terms in reverse order is always equal to 40, which is the sum of the first
and last terms in the sequence.
What Gauss realized was that this sum can be multiplied by the number of terms and then divided by two (since we
are actually summing the series twice here) to get the sum of the terms in the original sequence. For the problem he
was given in school, finding the sum of the first 100 integers, he was able to just use the first term, a1 = 1 , the last
term, an = 100, and the total number of terms, n = 100, in the following formula:

n (a1 + an ) 100 (1 + 100)


= = 5050
2 2

In our example we know the first and last terms but how many terms are there? We need to find n to use the formula
to find the sum of the series. We can use the first and last terms and the nth term to do this.

an = a1 + d(n − 1)
39 = 1 + 2(n − 1)
38 = 2(n − 1)
19 = n − 1
20 = n
20(1+39)
Now the sum is 2 = 400.
More Guidance-Proof of the Arithmetic Sum Formula
The rule for finding the nth term of an arithmetic sequence and properties of summations that were explored in the
problem set in the concept Series and Summation Notation can be used to prove the formula algebraically. First,
we will start with the nth term rule an = a1 + (n − 1)d. We need to find the sum of numerous nth terms (n of them to
be exact) so we will use the index, i, in a summation as shown below:

301
6.1. Arithmetic Sequences and Series www.ck12.org

n
∑ [a1 + (i − 1)d] Keep in mind that a1 and d are constants in this expression.
i=1
n n
We can separate this into two separate summations as shown: ∑ a1 + ∑ (i − 1)d
i=1 i=1
n
Expanding the first summation, ∑ a1 = a1 + a1 + a1 + . . . + a1 such that a1 is added to itself n times. We can simplify
i=1
this expression to a1 n.
In the second summation, d can be brought
 out in
 front of the summation and the difference inside can be split up
n n
as we did with the addition to get: d ∑ i − ∑ 1 . Using rules from the concept Series and Summation Notation,
i=1 i=1
n n
1
∑ i= 2 n(n + 1) and ∑ 1 = n. Putting it all together, we can write an expression without any summation symbols
i=1 i=1
and simplify.

 
1
a1 n + d n(n + 1) − n
2
1
= a1 n + dn(n + 1) − dn Distribute d
2
1 1
= n [2a1 + d(n + 1) − 2d] Factor out n
2 2
1
= n [2a1 + dn + d − 2d]
2
1
= n [2a1 + dn − d]
2
1
= n [2a1 + d(n − 1)] ← This version of the equation is very useful if you don’t know the nth term.
2
1
= n [a1 + (a1 + d(n − 1))]
2
1
= n(a1 + an )
2

Example B
Find the sum of the first 40 terms in the arithmetic series 35 + 31 + 27 + 23 + . . .
Solution: For this particular series we know the first term and the common difference, so let’s use the rule that
doesn’t require the nth term: 12 n [2a1 + d(n − 1)], where n = 40, d = −4 and a1 = 35.

1
(40) [2(35) + (−4)(40 − 1)] = 20 [70 − 156] = −1720
2

We could also find the nth term and use the rule 12 n(a1 + an ), where an = a1 + d(n − 1).
a40 = 35 + (−4)(40 − 1) = 35 − 156 = −121, so the sum is 21 (40)(35 − 121) = 20(−86) = −1720.
Example C
Given that in an arithmetic series a21 = 165 and a35 = 277, find the sum of terms 21 to 35.
Solution: This time we have the “first” and “last” terms of the series, but not the number of terms or the common
difference. Since our series starts with the 21st term and ends with the 35th term, there are 15 terms in this series.
Now we can use the rule to find the sum as shown.

302
www.ck12.org Chapter 6. Discrete Functions

1
(15)(165 + 277) = 3315
2

Example D
8
Find the sum of the arithmetic series ∑ (12 − 3i)
i=1
Solution: From the summation notation, we know that we need to sum 8 terms. We can use the expression 12 − 3i
to find the first and last terms as and the use the rule to find the sum.
First term: 12 − 3(1) = 9
Last term: 12 − 3(8) = −12
8
∑ (12 − 3i) = 21 (8)(9 − 12) = 4(−3) = −12.
i=1
We could use the calculator in this problem as well: sum(seq(12 − 3x, x, 1, 8)) = −12
Guided Practice
1. Find the sum of the series 87 + 79 + 71 + 63 + . . . + −105.
50
2. Find ∑ (3i − 90).
i=10
3. Find the sum of the first 30 terms in the series 1 + 6 + 11 + 16 + . . .
Answers
1. d = 8, so

−105 = 87 + (−8)(n − 1)
−192 = −8n + 8
−200 = −8n
n = 25

and then use the rule to find the sum is 12 (25)(87 − 105) = −225
2. 10th term is 3(10) − 90 = −60, 50th term is 3(50) − 90 = 60 and n = 50 − 10 + 1 = 41 (add 1 to include the 10th
term). The sum of the series is 12 (41)(−60 + 60) = 0. Note that the calculator is a great option for this problem:
sum(seq(3x − 90, x, 10, 50)) = 0.
3. d = 5, use the sum formula, 21 n(2a1 + d(n − 1)), to get 21 (30) [2(1) + 5(30 − 1)] = 15 [2 + 145] = 2205

303
6.2. Geometric Sequences and Series www.ck12.org

6.2 Geometric Sequences and Series

Objective
Identify geometric sequences, find the nth term rule for a geometric sequence and find the sum of a finite geometric
sequence.
Review Queue

1. Find the nth term rule for the arithmetic sequence with a8 = 1 and a32 = 13.
15
2. Find ∑ (2i + 7).
i=1
3. Find the sum of the series 9 + 7 + 5 + 3 + . . . + −37 + −39.

Geometric Sequences and Finding the N[U+1D57]h Term Given the Common
Ratio and the First Term

Objective
Identify a geometric sequence and its common ratio and write an nth term rule given the common ratio and a term.
Guidance
A geometric sequence is a sequence in which the ratio between any two consecutive terms, aan−1 n
, is constant. This
constant value is called the common ratio. Another way to think of this is that each term is multiplied by the same
value, the common ratio, to get the next term.
Example A
Consider the sequence 2, 6, 18, 54, . . .
Is this sequence geometric? If so, what is the common difference?
6 18 54
Solution: If we look at each pair of successive terms and evaluate the ratios, we get 2 = 6 = 18 = 3 which indicates
that the sequence is geometric and that the common ratio is 3.
More Guidance
Now let’s see if we can develop a general rule (nth term) for this sequence. Since we know that each term is multiplied
by 3 to get the next term, let’s rewrite each term as a product and see if there is a pattern.

a1 = 2
a2 = a1 (3) = 2(3) = 2(3)1
a3 = a2 (3) = 2(3)(3) = 2(3)2
a4 = a3 (3) = 2(3)(3)(3) = 2(3)3

This illustrates that the general rule is an = a1 (r)n−1 , where r is the common ratio. This even works for the first term
since a1 = 2(3)0 = 2(1) = 2.
Example B

304
www.ck12.org Chapter 6. Discrete Functions

Write a general rule for the geometric sequence 64, 32, 16, 8, . . .
Solution: From the general rule above we can see that we need to know two things: the first term and the common
ratio to write the general rule. The first term is 64 and we can find the common ratio by dividing a pair of successive
1 n−1
terms, 32 1 th

64 = 2 . The n term rule is thus an = 64 2 .
Example C
Find the nth term rule for the sequence 81, 54, 36, 24, . . . and hence find the 12th term.
2 n−1
Solution: The first term here is 81 and the common ratio, r, is 54 2 th

81 = 3 . The n term rule is an = 81 3 . Now we
th 2 12−1 2 11 2048
 
can find the 12 term a12 = 81 3 = 81 3 = 2187 . Use the graphing calculator for the last step and MATH
> Frac your answer to get the fraction. We could also use the calculator and the general rule to generate terms
seq(81(2/3)∧ (x − 1), x, 12, 12). Reminder: the seq( ) function can be found in the LIST (2nd STAT) Menu under
OPS. Be careful to make sure that the entire exponent is enclosed in parenthesis.
Example D
Randall deposits $1000 in a savings account which earns 3% interest per year. What is the value of his investment
after 15 years?
Solution: This is actually a geometric sequence. The value of the investment at the end of each year is the terms in the
sequence which corresponds to that year. Our common difference will be 1.03 (103%) because we are maintaining
the original amount (100%) and adding another 3%. Let’s look at the terms:

Year 1 2 3 ... 14 15
2 3 14
Value 1000(1.03) 1000(1.03) 1000(1.03) ... 1000(1.03) 1000(1.03)15

The general rule for the terms in this sequence (i.e. the value of the investment after a prescribed number of years)
is an = 1000(1.03)n . Note that since we are looking at the value at the end of a certain number of years and we will
get interest for all of those years, the exponent is n rather than (n − 1). The value at the end of five years can be
calculated using a15 = 1000(1.03)15 = 1557.967417 ≈ $1557.97
Guided Practice
1. Identify which of the following are geometric sequences. If the sequence is geometric, find the common ratio.
a. 5, 10, 15, 20, . . .
b. 1, 2, 4, 8, . . .
c. 243, 49, 7, 1, . . .
2. Find the general rule and the 20th term for the sequence 3, 6, 12, 24, . . .
3. Find the nth term rule and list terms 5 thru 11 using your calculator for the sequence −1024, 768, −432, −324, . . .
4. Find the value of a 10 year old car if the purchase price was $22,000 and it depreciates at a rate of 9% per year.
Answers
1. a. arithmetic
b. geometric, r = 2
1
c. geometric, r = 7
6
2. The first term is 3 and the common ratio is r = 3 = 2 so an = 3(2)n−1 .
The 20th term is a20 = 3(2)19 = 1, 572, 864.
768
n−1
3. The first term is -1024 and the common ratio is r = −1024 = − 34 so an = −1024 − 34 .

305
6.2. Geometric Sequences and Series www.ck12.org

Using the calculator sequence function to find the terms and MATH > Frac,
seq (−1024(−3/4)∧ (x − 1), x, 5, 11) = −324 243 − 729 2187
− 6561 19683
− 59049

4 16 256 256 1024
4. The first term (value of the car after 0 years) is $22,000. The common ratio is 1 − .09 or 0.91. The value of the car
after n years can be determined by an = 22, 000(0.91)n . For 10 years we get a10 = 22, 000(0.91)10 = 8567.154599 ≈
$8567.
Vocabulary

Geometric Sequence
A sequence in which the ratio of any two consecutive terms is constant.

Common Ratio
The value of the constant ratio between any two consecutive terms in a geometric sequence. Also, the value
by which you multiply a term in the sequence to get the next term.

Finding the N[U+1D57]h Term Given the Common Ratio and any Term or Two
Terms

Objective
Write an nth term rule for a geometric sequence given the common ratio and any term or any two terms in the
sequence.
Guidance
We will be using the general rule for the nth term in a geometric sequence and the given term(s) to determine the
first term and write a general rule to find any other term.
Example A
Consider the geometric sequence in which the common ratio is − 45 and a5 = 1280. Find the first term in the sequence
and write the general rule for the sequence.
Solution: We will start by using the term we know, the common ratio and the general rule, an = a1 rn−1 . By plugging
in the values we know, we can then solve for the first term, a1 .

 4
4
a5 = a1 −
5
 4
4
1280 = a1 −
5
1280
4 = a1
− 54
3125 = a1
n−1
Now, the nth term rule is an = 3125 − 45 .
Example B
1
Find the nth term rule for a sequence in which a1 = 16 and a7 = 4
1 1
Solution: Since a7 = 4 and we know the first term, we can write the equation 4 = 16r6 and solve for the common
ratio:

306
www.ck12.org Chapter 6. Discrete Functions

1
= 16r6
4
1
= r6
r 64
6 1 √
6
= r6
64
1
=r
2

1 n−1
The nth term rule is an = 16

2
Example C
Find the nth term rule for the geometric sequence in which a5 = 8 and a10 = 41 .
Solution: Using the same method at the previous example, we can solve for r and a1 . Then, write the general rule.
8
Equation 1: a5 = 8, so 8 = a1 r4 , solving for a1 we get a1 = r4
.
1
Equation 2: a10 = 41 , so 1
4 = a1 r9 , solving for a1 we get a1 = 4
r9
.

1
8 4
=
r4 r9
1
8r9 = r4
4
1 4
8r9 4r
=
8r4 8r4
1
r5 =
32

r
5 5 1
r5 =
32
1
r=
2

8 8
Thus, a1 = 4 = 1 = 81 · 16
1 = 128.
( 21 ) 16

3
The nth term rule is an =

8 (2)n−1 .
∗ Note: In solving the equation above for r we divided both sides by r4 . In general it is not advisable to divide both
sides of an equation by the variable because we may lose a possible solution, r = 0. However, in this case, r 6= 0
since it is the common ratio in a geometric sequence.
Guided Practice
1. Find the first term and the nth term rule for the geometric sequence given that r = − 21 and a6 = 3.
16
2. Find the common ratio and the nth term rule for the geometric sequence given that a1 = − 625 and a6 = − 52 .
3. Find the nth term rule for the geometric sequence in which a5 = 6 and a13 = 1536.
Answers
1. Use the known quantities in the general form for the nth term rule to find a1 .

307
6.2. Geometric Sequences and Series www.ck12.org

 5
1
3 = a1 −
2
     
32 1 32
− · 3 = a1 − · −
1 32 1
a1 = −96
n−1
Thus, an = −96 − 12
2. Again, substitute in the known quantities to solve for r.

 
5 16
− = − r5
2 625
 
5 625
− − = r5
2 16
3125
= r5
32

r
5 3125 5
= r5
32
5
r=
2
16 5 n−1

So, an = − 625 2
3. This time we have two unknowns, the first term and the common ratio. We will need to solve a system of equations
using both given terms.
6
Equation 1: a5 = 6, so 6 = a1 r4 , solving for a1 we get a1 = r4
.
1536
Equation 2: a13 = 1536, so 1536 = a1 r12 , solving for a1 we get a1 = r12
.
Now that both equations are solved for a1 we can set them equal to each other and solve for r.

6 1536
4
= 12
r r
6r12 = 1536r4
6r12 1536r4
=
6r4 6r4
8
r = 256
√8 √
8
r8 = 256
r=2

6 6
Now use r to find a1 : a1 = (24 )
= 16 = 38 .
3
The nth term rule is an =

8 (2)n−1 .

Finding the Sum of a Finite Geometric Series

Objective

308
www.ck12.org Chapter 6. Discrete Functions

Find the sum of a geometric series using the formula and the calculator.
Guidance
We have discussed in previous sections how to use the calculator to find the sum of any series provided we know the
nth term rule. For a geometric series, however, there is a specific rule that can be used to find the sum algebraically.
Let’s look at a finite geometric sequence and derive this rule.
Given an = a1 rn−1
The sum of the first n terms of a geometric sequence is: Sn = a1 + a1 r + a1 r2 + a1 r3 + . . . + a1 rn−2 + a1 rn−1
Now, factor out a1 to get a1 (1 + r2 + r3 + . . . + rn−2 + rn−1 ). If we isolate what is in the parenthesis and multiply this
sum by (1 − r) as shown below we can simplify the sum:

(1 − r)Sn = (1 − r)(1 + r + r2 + r3 + . . . + rn−2 + rn−1 )


= (1 + r + r2 + r3 + . . . + rn−2 + rn−1 − r − r2 − r3 − r4 − . . . − rn−1 − rn )
= (1 + r + r2 + r3 + . . . + rn−2 + rn−1 − r − r2 − r3 − r4 − . . . − rn−1 − rn )
= (1 − r)n

By multiplying the sum by 1 − r we were able to cancel out all of the middle terms. However, we have changed the
1−r
sum by a factor of 1 − r, so what we really need to do is multiply our sum by 1−r , or 1.
1−r a1 (1−rn )
a1 (1 + r2 + r3 + . . . + rn−2 + rn−1 ) 1−r = 1−r , which is the sum of a finite geometric series.
a1 (1−rn )
So, Sn = 1−r
Example A
1 n−1 .
Find the sum of the first ten terms of the geometric sequence an = 32 (−2) This could also be written as, “Find
10
1 n−1 .”
∑ 32 (−2)
n=1
1
Solution: Using the formula, a1 = 32 , r = −2, and n = 10.

1 10 1
32 (1 − (−2) ) 32 (1 − 1024) 341
S10 = = =−
1 − (−2) 3 32

We can also use the calculator as shown below.

341
sum(seq(1/32(−2)x−1 , x, 1, 10)) = −
32

Example B
Find the first term and the nth term rule for a geometric series in which the sum of the first 5 terms is 242 and the
common ratio is 3.
Solution: Plug in what we know to the formula for the sum and solve for the first term:

309
6.2. Geometric Sequences and Series www.ck12.org

a1 (1 − 35 )
242 =
1−3
a1 (−242)
242 =
−2
242 = 121a1
a1 = 2

The first term is 2 and an = 2(3)n−1 .


Example C
Charlie deposits $1000 on the first of each year into his investment account. The account grows at a rate of 8% per
year. How much money is in the account on the first day on the 11th year.
Solution: First, consider what is happening here on the first day of each year. On the first day of the first year,
$1000 is deposited. On the first day of the second year $1000 is deposited and the previously deposited $1000 earns
8% interest or grows by a factor of 1.08 (108%). On the first day of the third year another $1000 is deposited, the
previous year’s deposit earns 8% interest and the original deposit earns 8% interest for two years (we multiply by
1.082 ):

Sum Year 1 : 1000


Sum Year 2 : 1000 + 1000(1.08)
Sum Year 3 : 1000 + 1000(1.08) + 1000(1.08)2
Sum Year 4 : 1000 + 1000(1.08) + 1000(1.08)2 + 1000(1.08)3
..
.
Sum Year 11 : 1000 + 1000(1.08) + 1000(1.08)2 + 1000(1.08)3 + . . . + 1000(1.08)9 + 1000(1.08)10

∗There are 11 terms in this series because on the first day of the 11th year we make our final deposit and the original
deposit earns interest for 10 years.
This series is geometric. The first term is 1000, the common ratio is 1.08 and n = 11. Now we can calculate the sum
using the formula and determine the value of the investment account at the start of the 11th year.

1000 1 − 1.0811

s11 = = 16645.48746 ≈ $16, 645.49
1 − 1.08

Guided Practice
8
1. Evaluate ∑ 2(−3)n−1 .
n=3
215
2. If the sum of the first seven terms in a geometric series is 8 and r = − 12 , find the first term and the nth term rule.
3. Sam deposits $50 on the first of each month into an account which earns 0.5% interest each month. To the nearest
dollar, how much is in the account right after Sam makes his last deposit on the first day of the fifth year (the 49th
month).
Answers
1. Since we are asked to find the sum of the 3rd through 8th terms, we will consider a3 as the first term. The third
term is a3 = 2(−3)2 = 2(9) = 18. Since we are starting with term three, we will be summing 6 terms, a3 + a4 + a5 +

310
www.ck12.org Chapter 6. Discrete Functions

a6 + a7 + a8 , in total. We can use the rule for the sum of a geometric series now with a1 = 18, r = −3 and n = 6 to
find the sum:

8
18(1 − (−3)6 )
∑ 2(−3)n−1 = 1 − (−3)
= −3276
n=3

2. We can substitute what we know into the formula for the sum of a geometric series and solve for a1 .

  
1 7
215 a1 1 − − 2
= 1

8 1− −2
 
215 43
= a1
8 64
  
64 215
a1 = = 40
43 8

n−1
The nth term rule is an = 40 − 12
3. The deposits that Sam make and the interest earned on each deposit generate a geometric series,

S49 = 50 + 50(1.005)1 + 50(1.005)2 + 50(1.005)3 + . . . + 50(1.005)47 + 50(1.005)48 ,


↑ ↑
last deposit first deposit

Note that the first deposit earns interest for 48 months and the final deposit does not earn any interest. Now we can
find the sum using a1 = 50, r = 1.005 and n = 49.

50(1 − (1.005)49 )
S49 = ≈ $2768
(1 − 1.005)

311
6.3. Simple and Compound Interest www.ck12.org

6.3 Simple and Compound Interest

Learning Objectives

Here you will explore the difference between simple interest, interest compounded on a specific schedule, and
continuously compounded interest.
Suppose you are re-negotiating an allowance with your parents. Currently you are given $25 per week, but it is
the first of June, and you have started mowing the lawn and taking out the trash every week, and you think your
allowance should be increased.
Your father considers the situation and makes you the following offer:
"I tell you what, son. I will give you three options for your allowance, you tell me which you would like"
"Option A: You keep the $25 per week"
"Option B: You take $15 this week, then $16 next week, and so on. I’ll continue adding $1 per week until New
Year’s."
"Option C: I’ll give you 1 penny this week, and then double your allowance each week until the first of October,
then keep it at that rate."
Which option would you choose?

Simple and Compound Interest

Simple interest is interest which accrues based only on the principal of an investment or loan. The simple interest
is calculated as a percent of the principal.
Simple Interest: i = p · r · t.
Variable i is interest, p represents the principal amount, r represents the interest rate, and t represents the amount of
time the interest has been accruing. For example, say you borrow $2,000 from a family member, and you insist on
repaying with interest. You agree to pay 5% interest, and to pay the money back in 3 years.
The interest you will owe will be 2000(0.05)(3) = $300. This means that when you repay your loan, you will pay
$2300. Note that the interest you pay after 3 years is not 5% of the original loan, but 15%, as you paid 5% of $2000
each year for 3 years.
Now let’s consider an example in which interest is compounded. Say that you invest $2000 in a bank account, and
it earns 5% interest annually. How much is in the account after 3 years?
Compound interest: A(t) = p · (1 + r)t
Here, A(t) is the Amount in the account after a given time in years, principal is the initial investment, and rate is the
interest rate. Note that we use (1 + r) instead of just r, so we can find the entire amount in the account, not just the
interest paid.
A(t) = 2000 · (1.05)3
After three years, you will have $2315.25 in the account, which means that you will have earned $315.25 in interest.
Compounding results in more interest because the principal on which the interest is calculated increased each year.
Another way to look at it is that compounding creates more interest because you are earning interest on interest, and
not just on the principal.

312
www.ck12.org Chapter 6. Discrete Functions

MEDIA
Click image to the left or use the URL below.
URL: http://www.ck12.org/flx/render/embeddedobject/187523

Examples

Example 1

Earlier, you were asked which allowance option you would choose.
Assume you want to make the most money possible by the end of the year. Assume also that there are 24 weeks left.
Option A = 25 · 24 = $600 total
Option B = 15 + 16 + 17... + 39 = $609 total
Option C (assuming 16 weeks until Oct.) = 1 · (216 ) = $655.36 each week after Oct 1.
It is entirely possible that dear old dad didn’t take exponential growth seriously enough, he may need a second job!

Example 2

Use the formula for compound interest to determine the amount of money in an investment after 20 years, if you
invest $2000, and the interest rate is 5% compounded annually.
The investment will be worth $5306.60.
A(t) = P(1 + r)t
A(20) = 2000(1.05)20
A(20) = $5306.60

Example 3

How long will it take for $2000, invested at 5% compounded annually, to reach $7,000?
If we graph the function A(t) = 2000(1.05)t , we can see the values for any number of years.

313
6.3. Simple and Compound Interest www.ck12.org

If you graph this function using a graphing calculator, you can determine the value of the investment by tracing
along the function, or by pressing <TRACE> on your graphing calculator and then entering an x value.
You can also choose an investment value you would like to reach, and then determine the number of years it would
take to reach that amount. Find the intersection of the exponential function with the line y = 7000.

314
www.ck12.org Chapter 6. Discrete Functions

You can see here that the line and the curve intersect at a little less than x = 26. Therefore it would take almost 26
years for the investment to reach $7000.

Example 4

What is the value of an investment after 20 years, if you invest $2000, and the interest rate is 5% compounded
continuously?
The more often interest is compounded, the more it increases, but there is a limit. Each time you increase the number
of compoundings, you decrease the fraction of the annual interest that is applied to each compounding. Eventually,
the differences become so small as to be negligible. This is known as continuous compounding.
The function A(t) = Pert is the formula we use to calculate the amount of money when interest is continuously
compounded, rather than interest that is compounded at discrete intervals, such as monthly or quarterly.
A(t) = Pert
A(20) = 2000e.05(20)
A(20) = 2000e1
A(20) = $5436.56

315
6.3. Simple and Compound Interest www.ck12.org

Example 5

Compare the values of the investments shown in the table. If everything else is held constant, how does the
compounding influence the value of the investment?

TABLE 6.1:
Principal r n t
a. $4,000 .05 1 (annual) 8
b. $4,000 .05 4 (quarterly) 8
c. $4,000 .05 12 (monthly) 8
d. $4,000 .05 365 (daily) 8
e. $4,000 .05 8760 (hourly) 8

8n
Use the compound interest formula. For this example, the n is the quantity that changes: A(8) = 4000 1 + .05
n

TABLE 6.2:
Principal r n t A
a. $4,000 .05 1 (annual) 8 $5909.82
b. $4,000 .05 4 (quarterly) 8 $5952.52
c. $4,000 .05 12 (monthly) 8 $5962.34
d. $4,000 .05 365 (daily) 8 $5967.14
e. $4,000 .05 8760 (hourly) 8 $5967.29

8x
A graph of the function f (x) = 4000 1 + .05
x is shown below:

The graph seems to indicate that the function has a horizontal asymptote at $6000. However, if we zoom in, we can
see that the horizontal asymptote is closer to 5967.

316
www.ck12.org Chapter 6. Discrete Functions

What does this mean? This means that for the investment of $4000, at 5% interest, for 8 years, compounding more
and more frequently will never result in more than about $5968.00.

Example 6

Determine the value of each investment.

a. You invest $5000 in an account that gives 6% interest, compounded monthly. How much money do you have
after 10 years?

$5000, invested for 10 years at 6% interest, compounded monthly.


nt
A(t) = P 1 + nr
12·10
A(10) = 5000 1 + .0612

A(10) = 5000 (1.005)120


A(10) = $9096.98

b. You invest $10,000 in an account that gives 2.5% interest, compounded quarterly. How much money do you
have after 10 years?

$10000, invested for 10 years at 2.5% interest, compounded quarterly.


Quarterly compounding means that interest is compounded four times per year. So in the equation, n = 4.
nt
A(t) = P 1 + nr
4·10
A(10) = 6000 1 + .025
4
A(10) = 6000(1.00625)40
A(10) = $12, 830.30
In each example, the value of the investment after 10 years depends on three quantities: the principal of the
investment, the number of compoundings per year, and the interest rate.

Example 7

How long will it take $2000 to grow to $25,000 at a 5% interest rate?

317
6.3. Simple and Compound Interest www.ck12.org

It will take about 50 years:

TABLE 6.3:
A(t) = Pert
25,000 = 2000e.05(t)
12.5 = e.05(t) Divide both sides by 2000
ln 12.5 = ln e.05(t ) Take the ln of both sides
ln 12.5 = .05t ln e Use the power property of logs
ln 12.5 = .05t × 1 ln e = 1
ln 12.5 = 0.5t Isolate t
t = ln12.5
.05 ≈ 50.5

MEDIA
Click image to the left or use the URL below.
URL: http://www.ck12.org/flx/render/embeddedobject/187524

318
www.ck12.org Chapter 6. Discrete Functions

6.4 Annuities (Future Value)

Learning Objectives

Here you’ll learn how to compute future values of periodic payments.


Sally knows she can earn a nominal rate of 6% convertible monthly in a retirement account, and she decides she can
afford to save $1,500 from her paycheck every month. How can you use geometric series to simplify the calculation
of finding the future value of all these payments? How much money will Sally have saved in 30 years?

Annuity

An annuity is a series of equal payments that occur periodically. The word annuity comes from annual which means
yearly. You will start by working with payments that occur once at the end of each year and then delve deeper to
payments that occur monthly or any period.
Assume an investor saves R dollars at the end of each year for t years in an account that earns i interest per period.

• The first payment R will be in the bank account for t − 1 years and grow to be: R(1 + i)t−1
• The second payment R will be in the bank account for t − 2 years and grow to be: R(1 + i)t−2
• This pattern continues until the last payment of R that is deposited in the account right at t years, so it doesn’t
earn any interest at all.

The account balance at this point in the future (Future Value, FV ) is the sum of each individual FV of all the
payments:
FV = R + R(1 + i)1 + R(1 + i)2 + · · · + R(1 + i)t−2 + R(1 + i)t−1
Recall that a geometric series with initial value a and common ratio r with n terms has sum:
n
a + ar + ar2 + · · · + arn−1 = a · 1−r
1−r
So, a geometric series with starting value R and common ratio (1 + i) has sum:

1 − (1 + i)n
FV = R ·
1 − (1 + i)
1 − (1 + i)n
= R·
−i
(1 + i)n − 1
= R·
i

This formula describes the relationship between FV (the account balance in the future), R (the annual payment), n
(the number of years) and i (the interest per year).

319
6.4. Annuities (Future Value) www.ck12.org

MEDIA
Click image to the left or use the URL below.
URL: http://www.ck12.org/flx/render/embeddedobject/57222

The formula is extraordinarily flexible and will work even when payments occur monthly instead of yearly by
rethinking what, R, i and n mean. The resulting Future Value will still be correct. If R is monthly payments, then i
is the interest rate per month and n is the number of months.
Take an IRA (special type of savings account). If Lenny saves $5,000 a year at the end of each year for 35 years at
an interest rate of 4%, he can determine what his Future Value will be using the formula.
R = 5, 000, i = 0.04, n = 35, FV =?

(1 + i)n − 1
FV = R ·
i
(1 + 0.04)35 − 1
FV = 5, 000 ·
0.04
FV = $368, 281.12

MEDIA
Click image to the left or use the URL below.
URL: http://www.ck12.org/flx/render/embeddedobject/57224

Examples

Example 1

Earlier, you were given a problem where Sally wanted to know how much she will have if she can earn a nominal
6% interest rate compounded monthly in a retirement account where she decides to save $1500 from her paycheck
every month for thirty years.
0.06
FV =? , i = 12 = 0.005, n = 30 · 12 = 360, R = 1, 500

(1 + i)n − 1
FV = R ·
i
(1 + 0.005)360 − 1
FV = 1, 500 ·
0.005
FV ≈ 1, 506, 772.56

320
www.ck12.org Chapter 6. Discrete Functions

Example 2

How long does Mariah need to save if she wants to retire with a million dollars and saves $10,000 a year at 5%
interest?
FV = 1, 000, 000, R = 10, 000, i = 0.05, n =?

(1 + i)n − 1
FV = R ·
i
(1 + 0.05)n − 1
1, 000, 000 = 10, 000 ·
0.05
(1 + 0.05)n − 1
100 =
0.05
5 = (1 + 0.05)n − 1
6 = (1 + 0.05)n
ln 6
n= ≈ 36.7 years
ln 1.05

Example 3

How much will Peter need to save each month if he wants to buy an $8,000 car with cash in 5 years? He can earn a
nominal interest rate of 12% compounded monthly.
In this situation you will do all calculations in months instead of years. An adjustment in the interest rate and the
time is required and the answer needs to be clearly interpreted at the end.
0.12
FV = 8, 000, R =? , i = 12 = 0.01, n = 5 · 12 = 60

(1 + i)n − 1
FV = R ·
i
(1 + 0.01)60 − 1
8, 000 = R ·
0.01
8, 000 · 0.01
R= ≈ 97.96
(1 + 0.01)60 − 1

Peter will need to save about $97.96 every month.

Example 4

At the end of each quarter, Fermin makes a $200 deposit into a mutual fund. If his investment earns 8.1% interest
compounded quarterly, what will his annuity be worth in 15 years?
Quarterly means 4 times per year.
0.081
FV =? , R = 200, i = 4 , n = 60
60
(1+ 0.081
4 ) −1
FV = 200 · 0.081 ≈ $23, 008.71
4

321
6.4. Annuities (Future Value) www.ck12.org

Example 5

What interest rate compounded semi-annually is required to grow $25 semi-annual payments to $500 in 8 years?
FV = 500, R = 25, i =? , n = 8 · 2 = 16. Note that the calculation will be done in months. At the end you will
convert your answer to years.

(1 + i)n − 1
FV = R ·
i
(1 + i)16 − 1
500 = 25 ·
i
20i = (1 + i)16 − 1
0 = (1 + i)16 − 20i − 1

Using a graphing calculator, we find this equation has roots at i = 0 and i = 0.0290. Since i 6= 0, the semi-annual
interest rate is i = 0.0290 = 2.90% for a nominal annual interest rate of 5.80%.

322
www.ck12.org Chapter 6. Discrete Functions

6.5 Annuities (Present Value)

Learning Objectives

Here you’ll learn how to compute present values of equal periodic payments.
Many people buy houses they cannot afford. This causes major problems for both the banks and the people who
have their homes taken. In order to make wise choices when you buy a house, it is important to know how much you
can afford to pay each period and calculate a maximum loan amount.
Joanna knows she can afford to pay $12,000 a year for a house loan. Interest rates are 4.2% annually and most house
loans go for 30 years. What is the maximum loan she can afford? What will she end up paying after 30 years?

Annuities for Loans

The present value can be found from the future value using the regular compound growth formula:

PV (1 + i)n = FV
FV
PV =
(1 + i)n

You also know the future value of an annuity:


n −1
FV = R · (1+i)i
So by substitution, the formula for the present value of an annuity is:
n −1 n
(1+i) −1 −n
PV = R · (1+i)i 1
· (1+i)n = R · i(1+i)n = R ·
1−(1+i)
i
The present value of a series of equal payments R with interest rate i per period for n periods is:
−n
PV = R · 1−(1+i)
i
This formula can also be used to find out other information such as how much a regular payment should be and how
long it will take to pay off a loan.
Take a $1,000,000 house loan over 30 years with a nominal interest rate of 6% compounded monthly. You are not
given the monthly payments, R. To find R, solve for R in the formula given above.
PV = $1, 000, 000, R =? , i = 0.005, n = 360

1 − (1 + i)−n
PV = R ·
i
1 − (1 + 0.005)−360
1, 000, 000 = R ·
0.005
1, 000, 000 · 0.005
R= ≈ 5995.51
1 − (1 + 0.005)−360

It is remarkable that in order to pay off a $1,000,000 loan you will have to pay $5,995.51 a month, every month, for
thirty years. After 30 years, you will have made 360 payments of $5995.51, and therefore will have paid the bank

323
6.5. Annuities (Present Value) www.ck12.org

more than $2.1 million, more than twice the original loan amount. It is no wonder that people can get into trouble
taking on more debt than they can afford.

MEDIA
Click image to the left or use the URL below.
URL: http://www.ck12.org/flx/render/embeddedobject/57226

MEDIA
Click image to the left or use the URL below.
URL: http://www.ck12.org/flx/render/embeddedobject/57228

Examples

Example 1

Earlier, you were asked about how much Joanna can afford to take out in a loan. Joanna knows she can afford to pay
$12,000 a year to pay for a house loan. Interest rates are 4.2% annually and most house loans go for 30 years. What
is the maximum loan she can afford? What does she end up paying after 30 years? You can use the present value
formula to calculate the maximum loan:
−30
PV = 12, 000 · 1−(1+0.042)
0.042 ≈ $202, 556.98
For 30 years she will pay $12,000 a year. At the end of the 30 years she will have paid $12, 000 · 30 = $360, 000
total

Example 2

How long will it take to pay off a $20,000 car loan with a 6% annual interest rate compounded monthly if you pay
it off in monthly installments of $500? What about if you tried to pay it off in monthly installments of $100?
0.06
PV = $20, 000, R = $500, i = 12 = 0.005, n =?

1 − (1 + i)−n
PV = R ·
i
1 − (1 + 0.005)−n
20, 000 = 500 ·
0.005
0.2 = 1 − (1 + 0.005)−n
(1 + 0.005)−n = 0.8
ln 0.8
n=− ≈ 44.74 months
ln 1.005

324
www.ck12.org Chapter 6. Discrete Functions

For the $100 case, if you try to set up an equation and solve, there will be an error. This is because the interest on
$20,000 is exactly $100 and so every month the payment will go to only paying off the interest. If someone tries to
pay off less than $100, then the debt will grow.

Example 3

It saves money to pay off debt faster in order to save money on interest. As shown earlier, interest can more than
double the cost of a 30 year mortgage. This example shows how much money can be saved by paying off more than
the minimum.
Suppose a $300,000 loan has 6% interest convertible monthly with monthly payments over 30 years. What are the
monthly payments? How much time and money would be saved if the monthly payments were larger by a factor of
13
12 ? This is like making 13 payments a year instead of just 12. First you will calculate the monthly payments if 12
payments a year are made.

1 − (1 + i)−n
PV = R ·
i
1 − (1 + 0.005)−360
300, 000 = R ·
0.005
R = $1, 798.65

After 30 years, you will have paid $647,514.57, more than twice the original loan amount.
If instead the monthly payment was 13
12 · 1798.65 = 1948.54, you would pay off the loan faster. In order to find out
how much faster, you will make your unknown.

1 − (1 + i)−n
PV = R ·
i
1 − (1 + 0.005)−n
300, 000 = 1948.54 ·
0.005
0.7698 = 1 − (1 + 0.005)−n
(1 + 0.005)−n = 0.23019
ln 0.23019
n=− ≈ 294.5 months
ln 1.005
294.5 months is about 24.5 years. Paying fractionally more each month saved more than 5 years of payments.
294.5 months · $1, 948.54 = $573, 847.99
The loan ends up costing $573,847.99, which saves you more than $73,000 over the total cost if you had paid over
30 years.

Example 4

Mackenzie obtains a 15 year student loan for $160,000 with 6.8% interest. What will her yearly payments be?
PV = $160, 000, R =? , n = 15, i = 0.068

1 − (1 + 0.068)−15
160, 000 = R ·
0.068
R ≈ $17, 345.88

325
6.5. Annuities (Present Value) www.ck12.org

Example 5

How long will it take Francisco to pay off a $16,000 credit card bill with 19.9% APR if he pays $800 per month?
Note: APR in this case means nominal rate convertible monthly.
0.199
PV = $16, 000, R = $600, n =? , i = 12

−n
1 − 1 + 0.199
12
16, 000 = 600 · 0.199
12
n = 24.50 months

326
www.ck12.org Chapter 6. Discrete Functions

6.6 References

1. . . CC BY-NC
2. CK-12. CK-12 .
3. CK-12. CK-12 .

327
6.6. References www.ck12.org

328

You might also like